<<

Basic ICD-10-CM/PCS Coding

2013 Edition

Answer Key

Lou Ann Schraffenberger, MBA, RHIA, CCS, CCS-P, FAHIMA

Answer Key

The answer key includes the correct ICD-10-CM/PCS codes and the Alphabetic Index entry used to locate each code.

Chapter 1 4. Main term=Prolapse Introduction to ICD-10-CM M50.20 Prolapse—see Displacement, intervertebral disc, cervical 5. Main term=Stenosis Exercise 1.1 N88.2 Stenosis, endocervical—see Stenosis 1. N63 Mass, cervix 2. N13.30 Hydronephrosis (primary) 3. J34.2 Deviated, nasal septum Exercise 1.4 4. R59.0 Adenopathy, inguinal 1. A41.9 Sepsis NOS 5. I25.10 Disease, arteriosclerotic—see 2. A49.8 Infection, bacteroides NEC Disease, heart, ischemic, atherosclerotic. 3. I31.9 Pericarditis (with effusion) Arteriosclerotic heart—see Arteriosclerosis, 4. B08.3 Disease, fifth coronary (artery) Tabular List—B08.3—Erythema 6. G44.209 Headache, tension infectiosum [fifth disease] 7. K85.9 Pancreatitis (suppurative) 5. Dementia, with, Lewy bodies G31.83 8. K00.6 Eruption, tooth abnormal [F02.80] (premature) See the "Use additional code" note under 9. I33.0 Endocarditis, infectious category G31 10. I08.0 Endocarditis, mitral with aortic Use additional code to identify dementia (valve) disease, active or acute with behavioral disturbance (F02.81) Use additional code to identify dementia Exercise 1.2 without behavioral disturbance (F02.80) 1. Nonessential modifier = congenital Q67.8 Distortion (congenital) chest (wall) Exercise 1.5 2. Nonessential modifier=acute 1. Intrahepatic K57.32 (acute) intestine, large 2. Diabetes mellitus arising in pregnancy 3. Nonessential modifier = bleeding Gestational diabetes mellitus K64.4 Hemorrhoids external 3. Anorexia nervosa 4. Nonessential modifier=cardiac 4. Blackout, Fainting, Vasovagal attack R01.0 Murmur (cardiac) functional 5. Diverticulum of appendix 5. Nonessential modifier=chronic J32.0 Sinusitis (chronic) maxillary Exercise 1.6 1. I85.11 Varix, , in, cirrhosis of Exercise 1.3 , bleeding 1. Main term=Endomyometritis 2. N39.0 Infection, urinary (tract) N71.0 Endomyometritis—see Endometritis, Use additional code (B95–B97) to identify acute infectious agent 2. Main term=Metrorrhexis B96.20 Infection, Escherichia coli as cause N85.8 Metrorrhexis—see Rupture, uterus, of disease classified elsewhere nontraumatic 3. K26.0 Ulcer, duodenum, acute, with 3. Main term=Osteoarthrosis hemorrhage M19.019 Osteoarthrosis—see also 4. P61.2 Anemia, due to, prematurity Osteoarthritis, shoulder, M19.01- 5. Z04.1 Examination, following, motor (unspecified = M19.019) vehicle accident

Review Exercises: Chapter 1

1. K35.2 Appendicitis, acute, with perforation 2. J15.4 Pneumonia, streptococcal NEC 3. R07.2 Pain, chest, precordial 4. I26.09 Cor, pulmonale, acute 5. M19.071 Osteoarthrosis—see Osteoarthritis, primary, ankle, right 6. E05.20 Goiter, nodular, toxic 7. Q89.2 Extra—see Accessory, thyroid 8. K55.21 Angiodysplasia (colon) with bleeding 9. J20.9 Tracheobronchitis—see also Bronchitis, acute 10. I25.119 Disease, heart, arteriosclerotic—see Disease, heart, ischemic, atherosclerotic with angina pectoris—see Arteriosclerosis, coronary (artery) native vessel with angina pectoris 11. M32.14 Nephritis, due to, systemic lupus erythematosus 12. Z34.02 Prenatal, care, normal first pregnancy—see Pregnancy, normal, first, second trimester 13. S72.142A Fracture, femur, upper end, intertrochanteric, left The coder must see the Tabular List for assignment of the left side and the seventh character “A” for the closed fracture, initial encounter to identify the encounter. 14. A59.02 Prostatitis, trichomonal 15. I63.239 Occlusion, artery, carotid, with cerebral infarction I10 Hypertension (essential) 16. B17.11, Hepatitis, C, acute, with hepatic coma 17. C91.01, Leukemia, acute lymphoblastic, see Tabular List for sixth character for in remission 18. D3A.020, Tumor, carcinoid, benign, appendix 19. N40.1, Enlarged, prostate, with lower urinary tract symptoms (urinary obstruction) N13.8 verified in Tabular List under code N40.1 N13.8 Obstruction, urinary, specified. Inclusion notes under N13.8 state urinary obstruction due to specified cause and there is a “code first” note present to code the causal condition such as enlarge prostate Corrected from Alphabetic Index: N13.9, Obstruction, urinary 20. Z85.3, History, personal, malignant neoplasm (of), breast Chapter 2 3. Answer: 041L0KL Introduction to ICD-10-PCS Character Code Explanation Section 0 Medical and Surgical

Body 4 Lower Arteries Review Exercises: Chapter 2 Syste

1. Answer: 0DJO8ZZ m Character Code Explanation Root 1 Bypass Oper Section 0 Medical and Surgical ation Body D Gastrointestinal System Body Part L Femoral Artery, Left Syst em Approach 0 Open Root J Inspection Device K Nonautologous Tissue Ope Substitute ratio Qualifier L Popliteal Artery n Body Part 0 Upper Intestinal Tract INDEX: When consulting the Index, the main term Approach 8 Via Natural or Artificial Bypass, subterm Artery, Femoral produced Opening Endoscopic the root operation table of 041. According to ICD-10-PCS guideline B3.6a.Bypass Device Z No Device procedures: Bypass procedures are coded by Qualifier Z No Qualifier identifying the body part bypassed “from”

and the body part bypassed “to”. The fourth INDEX: Root Operation: Inspection. Index: character body part specifies the body part Esophagogastroduodenoscopy (0DJ08ZZ) bypassed from, and the qualifier specifies the

body part bypassed to. In this example, the In this example, the complete seven bypass was “from” the femoral artery “to” the character code is listed in the Index. The popliteal artery. A cadaver vein graft is the code still must be confirmed using the code device identified as nonautologous (from Tables. EGD is an inspection procedure another human than patient) tissue substitute. when no other procedures, such as a biopsy

or excision, are performed with the EGD. 4. Answer: 0UN24ZZ Body part inspected is the upper intestinal Character Code Explanation tract. Approach is through the mouth so “via Section 0 Medical and Surgical natural or artificial opening endoscopic” is the choice for this procedure. Body U Female Reproductive Syste System 2. Answer: 0HBU0ZZ m Character Code Explanation Root N Release Opera Section 0 Medical and Surgical tion Body System H Skin and Breast Body Part 2 Ovaries, Bilateral Root B Excision Approach 4 Percutaneous Endoscopic Oper ation Device Z No Device Body Part U Breast, Left Qualifier Z No Qualifier Approach 0 Open INDEX: Lysis see Release. Release, Ovaries, Device Z No Device Bilateral (0UN2) Definition of release in Qualifier Z No Qualifier ICD10-PCS is freeing a body part from an

abnormal physical constraint which also INDEX: When consulting the Index, the main term describes a procedure identified as lysis. Two has two subterms: see Excision, codes are required for this procedure as the Skin and Breast and see Resection, Skin and same root operation is performed on different Breast. Since only part of the breast was body parts as defined by distinct values of the removed, the root operation is Excision. body part character for the root operation

“release.” is an approach that is 6. Answer: 0TP98DZ percutaneous endoscopic Character Code Explanation Section 0 Medical and Surgical Answer: 0UN74ZZ Body System T Urinary System Character Code Explanation Root P Removal Section 0 Medical and Surgical Operation Body U Female Reproductive Body Part 9 Ureter System System Approach 8 Via Natural or Artificial Root N Release Opening Endoscopic Operation Device D Intraluminal Device Body Part 7 Fallopian Tubes, Bilateral Qualifier Z No Qualifier Approach 4 Percutaneous Endoscopic Device Z No Device INDEX: Root Operation: Removal. Index: Qualifier Z No Qualifier Removal of device from, Ureter (0TP9) The objective of the procedure was to take the stent INDEX: Lysis see Release. Release, Fallopian out of the ureter which matches the definition of Tubes (0UN7) “removal” to take out of off a device from a body part. The approach is by cystoscopy or via natural or 5. Answer: 0SG10A1 artificial opening (urethra) endoscopic. A stent is an Character Code Explanation intra-luminal device which are devices placed inside Section 0 Medical and Surgical tubular body parts. Body System S Lower Joints Root G Fusion 7. Answer: 0XMJ0ZZ Operation Character Code Explanation Body Part 1 Lumbar Vertebral Joints, 2 Section 0 Medical and Surgical or more Body X Anatomical Region, Upper Approach 0 Open System Extremities Device A Interbody Fusion Device Root M Reattachment Qualifier 1 Posterior Approach, Operation Posterior Column Body Part J Hand, Right Approach 0 Open INDEX: Root operation is fusion. Index: Fusion, Device Z No Device lumbar vertebrae 2 or more (0SG1). According to Qualifier Z No Qualifier the ICD-10-PCS guideline for fusion procedures of the spine, B3.10a: The body part coded for a spinal INDEX: Root Operation: Reattachment. Index: vertebral joint(s) rendered immobile by a spinal Reattachment, Hand, Right (0XMJ0ZZ) Index fusion procedure is classified by the level of the provides a specific code for this operation. The only spine (e.g. thoracic). There are distinct body part variable on the Table is the body part being values for a single vertebral joint and for multiple reattached. vertebral joints at each spinal level. According to guideline B3.10.c if an interbody fusion device is 8. Answer: 0W9G3ZX used to render the joint immobile (alone or Character Code Explanation containing other material like bone graft), the Section 0 Medical and Surgical procedure is coded with the device value Interbody Body W Anatomical Regions, Fusion Device. The qualifier identifies the System General combination of the approach and the column. Root 9 Drainage Operation Body Part G Peritoneal Cavity Approach 3 Percutaneous Device Z No Device Qualifier X Diagnostic

INDEX: Root Operation: Drainage Index: , Peritoneal Cavity see Drainage, Peritoneal Cavity (0W9G) The Index includes the six characters leaving the coder to select the qualifier procedure of paracentesis to send the coder directly to identify the type of transplant. The ICD-10-PCS to the correct coding Table. The approach is Reference Manual describes transplantation as described as percutaneous and the procedure is “allogeneic” for a human donor, “syngeneic” for identified as “diagnostic” so that the qualifier “X” is identical twin donor, and “zooplastic” for animal as used. the source of the organ.

9. Answer: 0Y6M0Z9 11. Answer: 0DQE0ZZ Character Code Explanation Character Code Explanation Section 0 Medical and Surgical Section 0 Medical and Surgical Body Y Anatomical Region, Lower Body D Gastrointestinal System System Extremities System Root 6 Detachment Root Q Repair Operation Operation Body Part M Foot, Right Body Part E Approach 0 Open Approach 0 Open Device Z No Device Device Z No Device Qualifier 9 Partial 1st Ray Qualifier Z No Qualifier

INDEX: —see Detachment. Root INDEX: Root Operation: Repair. Index: Repair, Operation: Detachment. Index: Detachment, Foot, Intestine, Large (0DQE) The term “repair” is defined Right (0Y6M0Z). The Index gives the coder as restoring to the extent possible, a body part to its everything except character 7 for the qualifier. The normal anatomic structure and function. Often the choices for the qualifiers listed are complete, term “repair” involves a suture repair which needs to complete 1st thought 5th ray, partial and partial 1st be confirmed in the operative report. The Index thought 5th ray. Complete is defined as amputation includes the term “Suture, laceration repair, see through the carpometacarpal (hand) or through Repair.” If the operative report described the tarsal-metatarsal (foot). Partial is amputation location more precisely, the body part character anywhere along the shaft or head of the metacarpal could change. (hand) or metatarsal (foot). The fingers or toes are 1st thought 5th rays. In this example, the right big toe is the 1st ray. Transmetatarsal is a partial amputation.

The definition of the detachment qualifiers are included in the ICD-10-PCS Reference Manual, which can be found as a download at http://www.cms.gov/Medicare/Coding/ICD10/2013- ICD-10-PCS-GEMs.html

10. Answer: 0TY10Z0 Character Code Explanation Section 0 Medical and Surgical Body T Urinary System System Root Y Transplantation Operation Body Part 1 Kidney, Left Approach 0 Open Device Z No Device Qualifier 0 Allogeneic

INDEX: Root Operation: Transplantation. Index: Transplant, Kidney, Left (0TY10Z) The Index gives 12. Answer: 0QSG0ZZ 14. Answer: 02703ZZ Character Code Explanation Character Code Explanation Section 0 Medical and Surgical Section 0 Medical and Surgical Body System Q Lower Bones Body 2 Heart and Great Vessels Root S Reposition System Operation Root 7 Dilation Body Part G Tibia, Right Operation Approach 0 Open Body Part 0 Coronary Artery, One Site Device Z No Device Approach 3 Percutaneous Qualifier Z No Qualifier Device Z No Device Qualifier Z No Qualifier INDEX: Root Operation: Reposition. Index: Reduction, fracture, see reposition. Reposition, INDEX: Root Operation: Dilation. Index: Tibia, Right (0QSG). There is no device used for Angioplasty—see Dilation, Heart and Great Vessels. this question because no fixation device is included Index: Dilation, artery, coronary, one site (0270) in the procedure title. The device character would Also the option “PTCA” see Dilation, Heart and change depending on the type of fixation device use, Great Vessels (027) The coding of a PTCA if any. procedure depends on coronary artery lesion sites (See guideline B4.4) involved (body part), the 13. Answer: 02RG08Z approach (usually percutaneous) and whether or not Character Code Explanation a stent (and what type) is used for the device Section 0 Medical and Surgical character. Bifurcation for a qualifier option is to Body 2 Heart and Great Vessels identify when a procedure is performed at site of a System vessel’s bifurcation. Root R Replacement Operation Body Part G Mitral Valve Approach 0 Open Device 8 Zooplastic Tissue Qualifier Z No Qualifier

INDEX: Root Operation: Replacement. Index: Replacement, Valve, Mitral (02RG) The coder needs to complete the code with the approach (open), the device (porcine which is animal or zooplastic tissue) and the default “Z” for no qualifier .

15. Answer: 0D5N8ZZ 17. Answer: 0SWD0JZ Character Code Explanation Character Code Explanation Section 0 Medical and Surgical Section 0 Medical and Surgical Body D Gastrointestinal System Body System S Lower Joints System Root W Revision Root 5 Destruction Operation Operation Body Part D Knee Joint, Left Body Part N Approach 0 Open Approach 8 Via Natural or Artificial Device J Synthetic Substitute Opening Endoscopic Qualifier Z No Qualifier Device Z No Device Qualifier Z No Qualifier INDEX: Root Operation: Revision. Index: Revision of device in, joint, knee, left (0SWD)Coder INDEX: Root Operation: Destruction. Index: needs to complete 5th character for approach (open), Fulguration—see Destruction. Destruction, colon, 6th character for device (joint prosthesis are metal or sigmoid (0D5N.) Usually, a procedure like this ceramic, which are synthetic substitutes for the joint) example is titled with fulguration of and no option for qualifier polyp. The coder must complete the code with the approach (endoscopic via natural opening to reach a colonic polyp) and there is no option for a device or a qualifier.

16. Answer: 05CD0ZZ Character Code Explanation Section 0 Medical and Surgical Body System 5 Upper Veins Root C Extirpation Operation Body Part D Cephalic Vein, Right Approach 0 Open Device Z No Device Qualifier Z No Qualifier

INDEX: Root Operation: Extirpation. Thrombectomy-see Extirpation. Index: Extirpation, Vein, Cephalic, Right (05CD) Again, the coder must complete the coder with the approach (open=by incision) but no options for device or qualifier.

18. Answer: 0UDB8ZX 20. Answer: 0D848ZZ Character Code Explanation Character Code Explanation Section 0 Medical and Surgical Section 0 Medical and Surgical Body U Female Reproductive Body D Gastrointestinal System System System System Root D Extraction Root B Division Operation Operation Body Part B Endometrium Body Part 4 Esophagogastric Junction Approach 8 Via Natural or Artificial Approach 8 Via Natural or Artificial Opening Endoscopic Opening Endoscopic Device Z No Device Device Z No Device Qualifier X Diagnostic Qualifier Z No Qualifier

INDEX: Root Operation: Extraction. Index: INDEX: Root Operation: Division. Index: Curettage—see Excision, see Extraction The Esophagotomy see Division, Esophagogastric objective of the procedure, D&C, is a curettage of Junction (0D84). The root operation of division is the endometrium. A curettage is not an excision defined as cutting into a body part without draining procedure. It is a pulling or stripping out or off all of fluids and/or gases from the body part in order to a body part by the use of force which is the separate or transect a body part. An definition of extraction. Extraction, Endometrium esophagomyotomy is a procedure to cut into the (0UDB). Hysteroscopy for the approach is an esophagus muscle. The approach, an EGD, is endoscopic procedure through a natural opening to endoscopic procedure through a natural orifice. reach the endometrium. Qualifier X used as the There is no option for a device or qualifier. procedure is described as diagnostic. 21. Answer: 0HXKXZZ 19. Answer: 0TF6XZZ Character Code Explanation Character Code Explanation Section 0 Medical and Surgical Section 0 Medical and Surgical Body H Skin and Breast Body T Urinary System System System Root X Transfer Root F Fragmentation Operation Operation Body Part K Skin, Right Lower Leg Body Part 6 Ureter, Right Approach X External Approach X External Device Z No Device Device Z No Device Qualifier Z No Qualifier Qualifier Z No Qualifier INDEX: Root Operation: Transfer. Index: INDEX: Root Operation: Fragmentation. Index: Transfer, skin, lower leg, right (0HXKXZZ) Index see Fragmentation, Ureter, Right (0TF6) gives the coder the seven characters to verify in the Body part involved is the right ureter. An alternate code Table. Any procedure done on skin is an index entry—Extracorporeal shockwave external procedure. lithotripsy—see Fragmentation. Extracorporeal shock wave lithotripsy (ESWL) is the most common 22. Answer: 02VQ0CZ type of lithotripsy. "Extracorporeal" means outside Character Code Explanation the body. Lithotripsy is an external procedure as the Section 0 Medical and Surgical shockwaves enter through the body, not through an Body 2 Heart and Great Vessels incision or orifice. Approach is therefore external. System The code is completed using the default Z for no Root V Restriction device and no qualifier. Operation Body Part Q Pulmonary Artery, Right Approach 0 Open Device C Extraluminal Device Qualifier Z No Qualifier

INDEX: Root Operation: Restriction. Index: is nonautologous or other human tissue. The Banding—see Restriction, Artery, Pulmonary, Right Qualifier identifies that a vagina is being created. (02VQ.) The root operation “restriction” is defined as partially closing an orifice or the lumen of a 25. Answer: 0H0V0JZ tubular body part. A banding procedure puts a Character Code Explanation device on a tubular body part to partially close the Section 0 Medical and Surgical lumen. Approach is open as a thoracotomy. Device Body H Skin and Breast is specified in procedure title as an extraluminal System device that is the actual banding of the vessel. Root 0 Alteration Extraluminal—outside the tube/vessel.23. Operation Body Part V Breast, Bilateral 23. Answer: 04LE3DT Approach 0 Open Character Code Explanation Device J Synthetic Substitute Section 0 Medical and Surgical Qualifier Z Qualifier Body 4 Lower Arteries System INDEX: Root Operation: Alteration. Coder must Root L Occlusion recognize this is a cosmetic procedure, therefore, the Operation root operation is alteration. Index: Alteration, breast, Body Part E Internal Iliac Artery, Right bilateral (0H0V) The silicone implants are the Approach 3 Percutaneous device which is a synthetic substitute. Device D Intraluminal Device Qualifier T Uterine Artery, Right 26. Answer: 00HV3MZ Character Code Explanation INDEX: Root Operation: Occlusion. Index: Section 0 Medical and Surgical Embolization—see Occlusion, artery, internal iliac, Body 0 Central Nervous System right uterine artery, right (04LE) There is no entry in System the Index for artery, uterine. Using the Body Part Root H Insertion Key, the uterine artery is used as the internal iliac Operation artery for the body part character. The Device Key is Body Part V Spinal Cord used to identify that embolization coil is an Approach 3 Percutaneous intraluminal device for the device character. The Device M Neurostimulator Lead Qualifier “T” identifies the uterine artery as the site Qualifier Z No Qualifier of the procedure. INDEX: Root Operation: Insertion. Index: Insertion 24. Answer: 0W4M0K0 of device in, spinal cord (00HV). The device is a Character Code Explanation neurostimulator which is inserted into lumbar spinal Section 0 Medical and Surgical cord by percutaneous approach. Body W Anatomical Regions, General System 27. Answer: 0UUG0JZ Root 4 Creation Character Code Explanation Operation Section 0 Medical and Surgical Body Part M Perineum, Male Body U Female Reproductive System Approach 0 Open System Device K Nonautologous Tissue Root U Supplement Substitute Operation Qualifier 0 Vagina Body Part G Vagina Approach 0 Open INDEX: Creation. The coder must know the Device J Synthetic Substitute definition of the root operations so that the root Qualifier Z No Qualifier operation of “creation” is accessed in the Index. Index: Creation, male, should be used as it is a male INDEX: Root Operation: Supplement. Index: patient having the procedure. The body part of the Colporrhaphy see Repair, Vagina (0UQG) When male where the procedure is performed is the reviewing this table, coder will note there is no perineum. The device of tissue bank donor material device character to identify the mesh used. So this is not the correct root operation. The definition of 30. Answer: 0W3G0ZZ supplement is putting in or on biological or synthetic Character Code Explanation material that physically reinforces and/or augments Section 0 Medical and Surgical the function of a portion of a body part. The Body W Anatomical Regions, insertion of mesh is a “supplement” procedure that System General reinforces the structure of the body part. The Index Root 3 Control entry to be used is Supplement, vagina (0UUG). Operation Mesh is a synthetic product. Body Part G Peritoneal Cavity Approach 0 Open 28. Answer: 0B21XFZ Device Z No Device Character Code Explanation Qualifier Z No Qualifier Section 0 Medical and Surgical Body B Respiratory System INDEX: Root Operation: Control. The procedure System is identified as a control of postoperative bleeding. Root 2 Change Index: Control, postprocedural bleeding in, Operation peritoneal cavity (0W3G). The approach is open as Body Part 1 Trachea described by . The body part where the Approach X External procedure is performed is the peritoneal cavity Device F Tracheostomy Device where the bleeding occurred. Qualifier Z No Qualifier

INDEX: Root Operation: Change. Index: Exchange see Change device in, trachea (0B21). An exchange procedure is removing and reinserting the same or similar device into the same location. A tracheostomy tube exchange involved a device in the trachea. The approach is external the tracheostomy opening is accessed directly at the skin level.

29. Answer: 00K00ZZ Character Code Explanation Section 0 Medical and Surgical Body 0 Central Nervous System System Root K Map Operation Body Part 0 Brain Approach 0 Open Device Z No Device Qualifier Z No Qualifier

INDEX: Root Operation: Map. Index: Map, brain (00K0). The approach is open as described by the craniotomy. There is no device left in the body after the procedure and no qualifier listed on the code table.

Chapter 3 two diagnoses equally meet the definition of Introduction to the principal diagnosis.

Uniform Hospital Discharge Data Set 10 . Principal diagnosis could be either acute and Official ICD-10-CM Coding pancreatitis or acute cholangitis as both are Guidelines unconfirmed diagnosis and could explain the patient’s symptoms. Because there are no specific Review Exercises: Chapter 3 symptoms noted, either acute pancreatitis or acute cholangitis may be listed as the principal diagnosis. 1. To establish a minimum common core of data to be collected on individual acute care short term 11 . Principal diagnosis is left lower quadrant hospital discharges in Medicare and Medicaid abdominal pain. The diagnoses of ruptured ovarian programs. UHDDS sought to improve the uniformity cyst and acute salpingitis are coded as additional and comparability of hospital discharge data. All diagnoses. non-outpatient settings including acute care, short term care, long term care, and psychiatric hospitals, 12 . Principal diagnosis is osteoarthritis of knee. home health agencies, rehabilitation facilities, and Additional codes for hypertensive heart disease and nursing homes. code for surgical or other procedure not carried out because of contraindication, would be assigned. 2. The condition established after study to be chiefly responsible for occasioning the admission of 13 . Principal diagnosis is postoperative wound the patient to the hospital for care. infection. An additional diagnosis for diverticulitis is assigned. 3. Conditions that coexist at the time of admission, that develop subsequently, or that affect the treatment 14 . Principal diagnosis is viral pneumonia. received and/or the length of stay. Diagnoses are to be excluded that relate to an earlier episode that has 15 . Principal diagnosis is status asthmaticus no bearing on the current hospital stay. 16 . Principal diagnosis is orthostatic 4. . A complication is an additional diagnosis that hypotension. An additional diagnosis code for describes a condition arising after the beginning of cataract is also assigned. The procedure of the the hospital observation and treatment and then cataract extraction is also coded. modifying the course of the patient’s illness or the medical care required. A comorbidity is an additional 17 . The other diagnoses of cholelithiasis and diagnosis that describes a preexisting condition that type II diabetes are coded. The history of pneumonia because of its presence with a specific principal and status post bunionectomy are unrelated to this diagnosis will likely cause an increase in the patient’s hospital stay, are historical events, and therefore not length of stay in the hospital. coded.

5 . Grand total of 22. 18 . The other diagnoses of hypertension and 18 Diagnosis codes + 3 E codes + 1 admitting benign prostatic hypertrophy are coded. No other diagnosis code diagnoses codes for the findings from the laboratory reports should be assigned without asking the 6 . 6 procedure codes physician if the abnormal findings are significant.

7 . Principal diagnosis = seizure 19 . In addition to the gastritis, both the acute duodenitis and acute pancreatitis should be coded. 8 . Principal diagnosis could be either acute pyelonephritis or acute cystitis as there are inter- 20 . The urinary retention would be reported with related conditions (same ICD-9-CM chapter) and “N” for no, condition not present on admission. both were treated during the hospital stay.

9 . Principal diagnosis could be either acute exacerbation of COPD or acute low back pain as the Chapter 4 occurring in the esophagus and causing an Certain Infectious and Parasitic esophagitis condition.

Diseases 11. PROCEDURE: Insertion of multilumen central venous catheter into the right subclavian vein for Review Exercises: Chapter 4 intravenous infusion by percutaneous approach Character Code Explanation 1. N39.0, Infection, urinary tract Section 0 Medical and Surgical B96.4, Infection, bacterial, NOS, as cause of Body 5 Upper Veins disease classified elsewhere, Proteus System (mirabilis) Root H Insertion The “Use additional code’ note under N39.0 Operation instructs the coder to an additional code Body Part 5 Subclavian Vein, Right (B95–B97) to identify the infectious agent. Approach 3 Percutaneous

Device 3 Infusion Device 2. A04.7, Colitis, Clostridium difficile Qualifier Z No Qualifier Z16.24, Resistance, to multiple drugs

(MDRO) antibiotics INDEX: Insertion of device in, vein, subclavian, ICD-10-CM provides a code to identify drug right 05H5 resistant organisms (Z16). The “use

additional code” note is found at the 12. PROCEDURE: and beginning of Chapter 1. small- of 50 cm portion of the

jejunum with side-to-side, functional end-to-end 3. A02.9, Poisoning, food, bacterial—see sewn anastomosis of the jejunum. The patient has Intoxication, foodborne, due to Salmonella peritonitis and a twisted nonviable small bowel. Another entry is Poisoning, food, due to, Salmonella. Character Code Explanation Food poisoning is classified to Chapter 1, Section 0 Medical and Surgical Certain infectious and parasitic disease Body D Gastrointestinal System (A00–B99). If gastroenteritis is documented, System then the code would change to A02.0. Root B Excision Operation 4. A56.11, Salpingitis, chlamydial Body Part A Jejunum Approach 0 Open 5. B20, AIDS Device Z No Device C46.0 Sarcoma, Kaposi's, skin (multiple) Qualifier Z No Qualifier

6. A41.81 Sepsis, Enterococcus INDEX: Excision, jejunum 0DBA. The physician K57.40 Diverticulitis, intestine, large, states resection but according to the definition of the with , with perforation root operations resection and excision, this operation is an excision because only a portion of the small 7. A41.3 Sepsis, Haemophilus influenzae bowel/jejunum. The approach is open as stated by R65.21 Sepsis, severe, with septic shock exploratory laparotomy. Anastomosis should not be N17.9 Failure, renal acute assigned separately. New coding guideline in 2013. B3.1b Components of a procedure specified in the 8. A37.01 Whooping cough, due to root operation definition and explanation are not Bordetella, pertussis, with pneumonia coded separately. Procedural steps necessary to reach the operative site and close the operative site, 9. B16,2 Hepatitis, B, acute, with, hepatic including anastomosis of a tubular body part, are coma also not coded separately. Example: Resection of a joint as part of a joint replacement procedure is 10. B37.81, Esophagitis, candidal included in the root operation definition of A combination code exists in ICD-10-CM to Replacement and is not coded separately. identify the myotic condition of candidiasis Laparotomy performed to reach the site of an open is not coded separately. In a resection of sigmoid colon with anastomosis of 14. PROCEDURE: Low anterior sigmoid colon (30 to , the anastomosis is not coded separately. cm) open resection with end-to-end anastomosis of sigmoid to sigmoid colon. 13. PROCEDURE: Insertion of venous access Character Code Explanation device/port percutaneously into the subclavian vein Section 0 Medical and Surgical advanced to the superior vena cava with a pocket for Body D Gastrointestinal System the port placed in the subcutaneous tissue of the System chest wall for chemotherapy to treat colon Root B Excision carcinoma. An incision is made to create the pocket. Operation Character Code Explanation Body Part N Sigmoid Colon Section 0 Medical and Surgical Approach 0 Open Body 2 Heart and Great Vessels Device Z No Device System Qualifier Z No Qualifier Root H Insertion Operation INDEX: Excision, colon, sigmoid 0DBN. The Body Part V Superior Vena Cava procedure is an excision because only 30 cm of Approach 3 Percutaneous sigmoid colon was removed. Anastomosis should Device 3 Infusion Device not be assigned separately. New coding guideline in Qualifier Z No Qualifier 2013. This procedure is an excision with colostomy creation. Guideline B3.1b INDEX: Insertion of device into vena cava, superior 02HV. The infusion device catheter is inserted into Components of a procedure specified in the root the superior vena cava by percutaneous approach operation definition and explanation are not coded with the device remaining in the vessel separately. Procedural steps necessary to reach the operative site and close the operative site, including 13. PROCEDURE: Insertion of VAD portion of anastomosis of a tubular body part, are also not procedure described above coded separately. Example: Resection of a joint as Character Code Explanation part of a joint replacement procedure is included in Section 0 Medical and Surgical the root operation definition of Replacement and is Body J Subcutaneous Tissue and not coded separately. Laparotomy performed to System Fascia reach the site of an open liver biopsy is not coded Root H Insertion separately. In a resection of sigmoid colon with Operation anastomosis of descending colon to rectum, the Body Part 6 Subcutaneous Tissue and anastomosis is not coded separately. Fascia, Chest Approach 0 Open 15. PROCEDURE: Removal of implanted infusion Device X Vascular Access Device port from the subcutaneous tissue in patient’s chest Qualifier Z No Qualifier by incision following completion of infusion therapy Character Code Explanation INDEX: Insertion of device into subcutaneous Section 0 Medical and Surgical tissue, chest 0JH6. The venous access port is placed Body J Subcutaneous Tissue and in the subcutaneous tissue in the chest wall. This is System Fascia an insertion of device left in place in the Root P Removal subcutaneous tissue. The coder should not use the Operation Index entry of “insertion of device, in chest wall” Body Part T Subcutaneous Tissue and because the chest wall is a different anatomic deeper Fascia, Trunk location.. The approach is open because an incision Approach 0 Open is made. The device is not specified as a “reservoir” Device 3 Infusion Device so the device “X” is chosen for the VAD. Qualifier Z No Qualifier INDEX: Removal of device from, subcutaneous tissue and fascia, trunk 0JPT (Chest) by incision or open approach. An implanted infusion port is an infusion device that is different from a venous access device or a reservoir Chapter 5 is not coded from the Neoplasm Table, but rather indexed under the term Neoplasms Melanoma. The specific type is type A spindle th cell that is indexed under spindle cell. The 5 Review Exercises character is identified in the Tabular List for the

right ciliary body of the eye 1. C34.31 Carcinoma, see also Neoplasm, by

site, malignant. Refer to Neoplasm Table, by site 5. G89.3 Pain, chronic, neoplasm related (lung), lower lobe, malignant, primary site C79.51 Carcinoma, see also Neoplasm, by C77.1 Refer to Neoplasm Table, by site, site, malignant. Refer to Neoplasm Table, by site lymph gland, intrathoracic, malignant, , bone, vertebrae, malignant, secondary site secondary site. C34.02 Refer to Neoplasm Table, by site, C79.31 Refer to Neoplasm Table, by site, lung, main bronchus (left) , malignant, primary brain, malignant, secondary site. site C79.51 Refer to Neoplasm Table, by site,

bone, rib, malignant, secondary site. ICD-10-CM coding guideline I.6.b.5.states that

when the reason for the encounter is for The primary site is the small cell carcinoma of neoplasm-related pain control or pain the right lower lobe of the lung. The management, the pain code may be assigned as intrathoracic lymph nodes, brain, and rib are the first-listed diagnosis. The underlying secondary sites. Index the term Carcinoma neoplasms should be reported as additional because the histological term is documented. diagnoses. This refers you to the Neoplasm Table, by site,

malignant. It is correct to list each metastatic 6. C78.7 Refer to Neoplasm Table, by site, site. liver, malignant, secondary site

Z85.038 History, personal, malignant 2. D3A.021 Carcinoid, see Tumor, carcinoid, neoplasm (of), colon benign, cecum. Z90.49 Absence (of) (organ or part)

(complete or partial), intestine, large (acquired) When indexing carcinoid, the note directs to Z92.21 History, chemotherapy for Tumor. It is not necessary to use the Neoplasm neoplastic condition Table to code this tumor. Under carcinoid, there Z93.3 Colostomy, status is a differentiation between benign or malignant,

with specific sites listed. Benign carcinoid The reason for this encounter is the metastatic tumors fall into category D3A, Benign liver cancer. The colon cancer was previously neuroendocrine tumors. The following notes are excised with no further treatment directed at that present: Code also any associated multiple site, therefore, it is coded as history of colon endocrine neoplasia [MEN] syndromes; and Use cancer. Because the patient had a previous colon additional code to identify any associate excision , a code for the acquired absence of the endocrine syndrome, such as: carcinoid large intestine is also coded. There is also a code syndrome (E34.0). available for history of chemotherapy and for the

presence (status) of a colostomy. . 3. C93.91 Leukemia, monocytic (subacute)

in remission 7. Z51.11 Chemotherapy (session) (for), Monocytic leukemia as stated in this exercise cancer is not specified so an unspecified code is C25.0 Carcinoma, see also Neoplasm, by used. The fifth character of 1 is assigned for site, malignant. Refer to Neoplasm Table, by the status of being in remission. The term site, , head , malignant, primary site subacute is a nonessential modifier that has Z90.411 Absence, pancreas, acquired, partial no influence on coding the condition.

The reason for the encounter (chemotherapy) is 4 C69.41 Melanoma, spindle cell, type A the first listed diagnosis. Coding guidelines (right) 1.C.2.e.2 describes the coding of encounters

solely for administration of chemotherapy. The neoplasm is coded as current (even though it was excised) because the patient is still receiving 11. PROCEDURE: Ultrasound probe-guided chemotherapy. The acquired absence of the prostate needle biopsy via rectum. One needle core pancreas may be coded. biopsy submitted for diagnostic evaluation. Character Code Explanation 8. Z51.0 Admission for, radiation therapy Section 0 Medical and Surgical (antineoplastic) Body V Male Reproductive System C61 Carcinoma, see also Neoplasm, by System site, malignant. Refer to Neoplasm Table, by Root B Excision site, prostate , malignant, primary site Operation Body Part 0 Prostate The reason for the encounter (radiation therapy) Approach 7 Via Natural or Artificial is the first listed diagnosis. The neoplasm is Opening coded as current as the tumor has not been Device Z No Device excised and the patient is receiving radiation Qualifier X Diagnostic therapy. Coding guideline I.C.2.e.2 describes the coding of encounters solely for administration of INDEX: Biopsy see Excision with qualifier radiation therapy diagnostic. Excision, prostate 0VB0. The needle biopsy is done to obtain tissue for pathological 9. C78.01 Metastatic, cancer, to specific site— examination for a definitive diagnosis. If multiple see Neoplasm, secondary by site prostate biopsies were performed, the code Neoplasm, lung, (right) , malignant secondary 0VB07ZX would be assigned for each biopsy taken Z85.528 History, personal, malignant according to ICD-10-PCS guideline B3.2b to code neoplasm, kidney multiple procedures when the same root operation is Z92.21 History, personal, chemotherapy for repeated at different body sites that are included in neoplastic condition the same body part value (prostate.) The approach Z92.3 History, personal, radiation therapy performed through the rectum is assigned the approach of “via natural or artificial opening.” The reason for the visit is evaluation of the metastatic carcinoma of the lung which is the PROCEDURE: Ultrasound portion of procedure first-listed code. History of kidney cancer which described above was the primary site was coded as a secondary Character Code Explanation diagnosis. Also coded was the patient’s history Section B Imaging of receiving chemotherapy and radiation therapy Body V Male Reproductive System

System 10. C7A.022 Tumor, carcinoid, malignant, Root Type 4 Excision Body Part 9 Prostate and seminal vesicles E34.0 Syndrome, carcinoid Contrast Z Via Natural or Artificial The patient was seen during this visit for the Opening malignant carcinoid tumor in the ascending Device Z None colon. In addition the patient was treated for the Qualifier Z None carcinoid syndrome that is a result of the INDEX: Ultrasonography, prostate and seminal carcinoid tumor. A “use additional code” note to vesicles BV49ZZZ identify any associated endocrine syndrome, such as: carcinoid syndrome (E34.0) appears under category C7A, Malignant neuroendocrine tumors.

12. PROCEDURE: Right breast with 14. PROCEDURE: Tube Thoracostomy—chest tube sentinel lymph node biopsy, right axilla insertion by incision—for drainage of malignant Character Code Explanation pleural effusion from right side of pleural cavity Section 0 Medical and Surgical Character Code Explanation Body H Skin and Breast Section 0 Medical and Surgical System Body W Anatomical Regions, General Root B Excision System Operation Root 9 Drainage Body Part T Breast, Right Operation Approach 0 Open Body Part 9 Pleural Cavity, Right Device Z No Device Approach 0 Open Qualifier Z No Qualifier Device 0 Drainage Device Qualifier Z No Qualifier INDEX: Lumpectomy see Excision, breast, right 0HBT. Lumpectomy is an open procedure The INDEX: Thoracostomy tube see Drainage Device, lumpectomy is a therapeutic procedure to remove a pleural cavity 0W99. The objective of this procedure tumor within the breast. is to drain fluid to remove the effusion through a drainage device, in this case, the chest tube. PROCEDURE: Sentinel node biopsy, right axilla Character Code Explanation 15. PROCEDURE: Rigid Bronchoscopy with YAG Section 0 Medical and Surgical laser photoresection for the destruction of lesion in Body 7 Lymphatic and Hemic the right main bronchus System Systems Character Code Explanation Root B Excision Section 0 Medical and Surgical Operation Body B Respiratory System Body Part 5 Lymphatic, Right Axillary System Approach 0 Open Root 5 Destruction Device Z No Device Operation Qualifier X Diagnostic Body Part 3 Main Bronchus, Right INDEX: Biopsy, see Excision, lymphatic, axillary, Approach 8 Via Natural or Artificial right 07B5. Sentinel node biopsies are open Opening Endoscopic procedures. The qualifier X is used to identify the Device Z No Device excision as a biopsy. A sentinel node biopsy is done Qualifier Z No Qualifier to obtain tissue for pathological examination to determine if disease is present. INDEX: Destruction, bronchus, main, right 0B53. Laser photoresection is a destruction procedure. 13. PROCEDURE: Open resection and removal of Approach is by bronchoscopy. A the left lobe of the liver due to metastasis from colon bronchoscopy is involves visualization of the carcinoma respiratory system by entering through the pharynx Character Code Explanation or nasopharynx which is a natural opening using an Section 0 Medical and Surgical endoscopic device. Body F Hepatobiliary System and System Pancreas Root T Resection Operation Body Part 2 Liver, Left Lobe Approach 0 Open Device Z No Device Qualifier Z No Qualifier

INDEX: Resection, liver, left lobe 0FT2 This procedure is a resection by definition because the entire body part, left lobe of liver, was removed

Chapter 6 13. D57.419 Thalassemia, sickle-cell—see Diseases of the Blood and Blood- Disease, sickle-cell, thalassemia, with crisis

Forming Organs and Certain 14. N18.6 Disease, end stage renal Disorders Involving the Immune D63.1 Anemia, due to (in) (with) chronic Mechanism kidney disease Code first underlying chronic kidney disease Review Exercises: Chapter 6 (CKD) (N18.-) 1. D50.0 Anemia, iron deficiency, secondary to blood loss (chronic). 15. D70.9, Neutropenic fever This condition may also be referred to Use additional code for any associated fever posthemorrhagic anemia (chronic.) (R50.81) R50.81, Fever presenting with conditions 2. D68.0, Von Willebrand's disease or classified elsewhere syndrome. Disease, von Willebrand 16. PROCEDURE: Transfusion via peripheral vein, 3. D73.2 Splenomegaly, congestive, chronic Blood Platelets, Nonautologous donor blood Character Code Explanation 4. D55.8, Anemia, hemolytic, nonspherocytic, Section 3 Administration congenital Physiological System 0 Circulatory The important term is here nonspherocytic. A Root Operation 2 Transfusion different disease is spherocytic anemia that is Body System/Region 3 Peripheral included in the Index as anemia, Approach 3 Percutaneous spherocytic—Spherocytosis, D58.0 Substance R Platelets Qualifier 1 Nonautologous 5. D69.3, Purpura, thrombocytopenia, idiopathic This condition may be documented as INDEX: Transfusion, Vein, Peripheral, Blood, immune thrombocytopenic purpura or Platelets, 3023 A transfusion is completed by a hemorrhagic purpura percutaneous puncture into a peripheral vein for infusion. 6. D61.09, Anemia, Fanconi's This condition may also be referred to as 17. PROCEDURE: Therapeutic Plasmapheresis, constitutional aplastic anema Single Session Character Code Explanation 7. D65, Coagulation, intravascular—See also Section 6 Extracorporeal Therapies Defibrination syndrome Body System A Physiological Systems This condition may commonly be Root 5 Pheresis documented as diffuse or disseminated Operation intravascular coagulation abbreviated as DIC. Body System 5 Circulatory Duration 0 Single 8. D62, Anemia, blood loss, acute Qualifier Z No Qualifier Qualifier 3 Plasma 9. D56.1, Anemia, Cooley's This condition may also be documented as INDEX: Plasmapheresis, 6A550Z3. The coder must beta thalassemia major or thalassemia major be careful not to confuse the main term plateletpheresis, therapeutic with plasmapheresis. It 10. D75.1, Polycythemia due to high altitude is important to validate the code obtained in the Index in the code table as done here. The qualifier 11. D55.0, Anemia, glucose-6-phosphate distinguishes between substances that can be dehydrogenate infused.

12. . D75.82, Thrombocytopenia, heparin induced (HIT)

18. PROCEDURE: Bone Marrow Needle Extraction Biopsy, Iliac Character Code Explanation Section 0 Medical and Surgical Body System 7 Lymphatic and Hemi Systems Root Operation D Extraction Body Part R Bone Marrow, Iliac Approach 3 Percutaneous Device Z No Device Qualifier X Diagnostic

INDEX: Extraction, Bone Marrow, Iliac, 07DR Bone marrow biopsies are not coded to excisions as there is no cutting but instead involve pulling out tissue from the bone. Therefore, bone marrow biopsies or aspirations are extractions with qualifier of X as these are diagnostic procedures.

19. PROCEDURE: Laparoscopic total splenectomy Character Code Explanation Section 0 Medical and Surgical Body System 7 Lymphatic and Hemic Systems Root Operation T Resection Body Part P Spleen Approach 4 Percutaneous Endoscopic Device Z No Device Qualifier Z No Qualifier

INDEX: Splenectomy, see Excision or Resection, Lymphatic and Hemic Systems, total splenectomy would be Resection, 07T. A laparoscopic approach is a percutaneous endoscopy as the trocars and ports used in a laparoscopic are placed in the body percutaneously so that the device of the laparoscope can be inserted.

20. PROCEDURE: Lymph Node Open Biopsy by Excision, Right Axilla Character Code Explanation Section 0 Medical and Surgical Body System 7 Lymphatic and Hemic Systems Root Operation B Excision Body Part 5 Lymphatic, Right Axillary Approach 0 Open Device Z No Device Qualifier X Diagnostic

INDEX: Biopsy see Excision with Qualifier, Excision, Lymphatic, Axillary, Right 07B5

Chapter 7 hyperglycemia (uncontrolled) Endocrine, Nutritional, and Metabolic 9. Diabetic proliferative retinopathy in a patient Diseases with controlled type 1 diabetes E10.359 Diabetes, with, retinopathy, Review Exercise: Chapter 7 proliferative

1. Type 2 diabetic with nephropathy due to the 10. Overweight adult with a body mass index diabetes (BMI) of 26.5 E11.21 Diabetes, type 2, with, E66.3 Overweight nephropathy Z68.26 Body, mass index, adult, 26.0- 26.9 2. Toxic diffuse goiter with thyrotoxic storm E05.01 Goiter, toxic—see 11. Syndrome of inappropriate secretion of Hyperthyroidism, with goiter (diffuse), with antidiuretic hormone (SIADH) thyroid storm E22.2 Syndrome, inappropriate secretion of antidiuretic hormone 3. Cushing’s syndrome E24.9 Syndrome, Cushing's 12. Hypoglycemia in type 1 diabetes with coma E10.641 Diabetes, type 1, with, 4. Hypokalemia hypoglycemia, with coma E87.6 Hypokalemia 13. Postsurgical hypothyroidism 5. Cystic fibrosis with pulmonary manifestations E89.0 Hypothyroidism, postsurgical E84.0 Fibrosis, cystic, with, pulmonary manifestations 14. Folic acid deficiency E53.8 Deficiency, folic acid 6. Uncontrolled (hyperglycemia) type 2 diabetes mellitus; mild degree malnutrition 15. Partial androgen insensitivity syndrome E11.65 Diabetes, type 2, with, E34.52 Syndrome, androgen hyperglycemia insensitivity, partial E44.1 Malnutrition, degree, mild There is no combination code for diabetes and 16. PROCEDURE: Open total thyroidectomy malnutrition, nor is there a stated cause-and- Character Code Explanation effect relationship between diabetes and Section 0 Medical and Surgical malnutrition There are no ICD-10-CM codes Body G Endocrine System that state “uncontrolled” diabetes but instead System the uncontrolled status is identified as Root T Resection diabetes with hyperglycemia. Uncontrolled Operation diabetes means the patient has elevated Body Part K Thyroid Gland glucose levels. In this example, there is no Approach 0 Open stated relationship between the diabetes and Device Z No Device the malnutrition so the condition is coded Qualifier Z No Qualifier separately.

INDEX: Thyroidectomy, see Resection, Endocrine 7. Panhypopituitarism System, thyroid gland, 0GTK. Total thyroidectomy E23.0 Panhypopituitarism is removal of entire thyroid.

8. Lower extremity ulcer on skin of left heel secondary to brittle diabetes mellitus, type 1, uncontrolled E10.621 Diabetes, type 1, with foot ulcer L97.429 Ulcer, heel - see Ulcer, lower limb, heel, left E10.65 Diabetes, type 1, with, 17. PROCEDURE: Partial left lobectomy, thyroid 20. PROCEDURE: Stereotactic gamma beam gland, open radiosurgery, parathyroid gland tumor Character Code Explanation Character Code Explanation Section 0 Medical and Surgical Section D Radiation Oncology Body G Endocrine System Body System G Endocrine System System Root Type 2 Stereotactic Radiosurgery Root B Excision Treatment 4 Parathyroid Glands Operation Site Body Part G Thyroid Gland Lobe, Left Modality J Stereotactic Gamma Beam Approach 0 Open Qualifier Radiosurgery Device Z No Device Isotope Z None Qualifier Z No Qualifier Qualifier Z None INDEX: Stereotactic Radiosurgery, Gamma Beam, INDEX: Lobectomy, excision, endocrine system Gland, Parathyroid, DG24JZZ 0GB. A thyroid lobe is a body part, so a partial Stereotactic radiosurgery is a destruction procedure excision of a body part is an excision of the parathyroid gland tumor using gamma beams.

18. PROCEDURE: Right carotid body biopsy, open, by excision Character Code Explanation Section 0 Medical and Surgical Body G Endocrine System System Root B Excision Operation Body Part 7 Carotid Body, Right Approach 0 Open Device Z No Device Qualifier X Diagnostic

INDEX: Biopsy, see Excision with qualifier diagnostic, carotid body, right, 0GB7

19. PROCEDURE: Laparoscopic partial left adrenalectomy Character Code Explanation Section 0 Medical and Surgical Body G Endocrine System System Root B Excision Operation Body Part 2 Adrenal Gland, Left Approach 4 Percutaneous Endoscopic Device Z No Device Qualifier Z No Device

INDEX: Adrenalectomy, see Excision, Endocrine System, see Gland, Adrenal, left, 0G32. A partial adrenalectomy would be an excision as the total gland is not removed. The laparoscopic approach is performed through the skin with the introduction of trocars and ports percutaneously to allow the laparoscopy or endoscopic procedure to be performed. Chapter 8 8. F31.62 Disorder, bipolar, current episode, Mental, Behavioral and mixed, moderate

Neurodevelopmental Disorders 9. F41.8 Anxiety depression

Review Exercises: Chapter 8 10. F91.3 Disorder, conduct, oppositional defiance 1. F12.280 Addiction, drug—see Dependence, drug, cannabis, with anxiety disorder 11. F90.1 Disorder, attention-deficit hyperactivity (adult), hyperactive type 2. F20.5 Schizophrenia, chronic undifferentiated or Schizophrenia, 12. I67.2 Arteriosclerosis, cerebral undifferentiated, chronic (Rationale: the Index F01.50 Dementia, arteriosclerotic—see must be trusted in this example. The title and Dementia, vascular, Code first note under inclusion terms for ICD-10-CM code F20.5 is category F01, Vascular dementia—Code first residual schizophrenia and does not include the underlying physiological condition or sequelae terminology included in the diagnosis provided. of cerebrovascular disease. But the Index clearly sends the coder to code F20.5 which is the correct code.) 13. F53 Depression, postpartum

3. F33.1 Depression, seasonal—see Disorder, 14. F80.81 Stuttering, childhood onset depressive, recurrent, current episode, moderate (Rationale: Another example of the coder 15. F80.1 Dysphasia, developmental, trusting the Index. The Index clearly refers the expressive coder from the entry of depression, seasonal to F71 Disabilities, intellectual, moderate disorder, depressive, recurrent as this is the Under section F70–F79, Intellectual Disabilities, nature of seasonal depression. The Index entry is a note to code first any associated physical or of disorder, depressive, recurrent directs the development disorders coder for the correct code. The category F33 includes the inclusion term of recurrent episodes 16. F68.12 Munchhausen’s syndrome—see of seasonal depressive disorder under the Disorder, factitious, with predominantly category heading) physical symptoms

4. F10.121 Alcoholic, intoxication (acute), with 17. F11.23 Addiction, heroin—see delirium Dependence, drug, opioid, with, withdrawal Y90.7 Index to External Causes, Blood alcohol level, 200 to 239 mg/100ml 18. F60.3 Disorder, personality, explosive (Rationale: Category F10, Alcohol related disorders, has the “use additional code note” to 19. F17.210 Smoker—see Dependence, drug, use a code for the blood alcohol level, if nicotine, cigarettes applicable. The coder must access the ICD-10- CM Index to External Causes because a blood 20. F14.182 Hypersomnia, due to, cocaine, abuse alcohol level is an external cause of a condition Another entry in the Index that can be uses is and not itself a disease. The main term is “blood “Abuse, cocaine, with sleep disorder.” alcohol level” with subterms for the various measurements of the blood alcohol level in the patient.

5. F43.12 Disorder, post-traumatic stress, chronic

6. F55.3 Abuse, steroids

7. F50.01 Anorexia, nervosa, restricting type 21. PROCEDURE: Individual cognitive 24. PROCEDURE: Cognitive-behavioral group psychotherapy for mental health treatment counseling for substance abuse treatment Character Code Explanation Character Code Explanation Section G Mental Health Section Section H Substance Abuse Treatment Body Z None Body Z None System System Root Type 5 Individual Psychotherapy Root Type 4 Group counseling Type 2 Cognitive Type 2 Cognitive-behavioral Qualifier Qualifier Qualifier Z None Qualifier Z None Qualifier Z None Qualifier Z None Qualifier Z None Qualifier Z None

INDEX: Psychotherapy, individual, mental health, INDEX: Counseling, group, cognitive-behavioral cognitive, GZ52ZZZ HZ42ZZZ

22. PROCEDURE: Electroconvulsive therapy, 25. PROCEDURE: Drug detoxification treatment bilateral, single seizure for substance abuse Character Code Explanation Character Code Explanation Section G Mental Health Section Section H Substance Abuse Treatment Body Z None Body Z None System System Root Type B Electroconvulsive Therapy Root Type 2 Detoxification services Type 2 Bilateral-single seizure Type Z None Qualifier Qualifier Qualifier Z None Qualifier Z None Qualifier Z None Qualifier Z None Qualifier Z None Qualifier Z None

INDEX: Electroconvulsive therapy, bilateral single INDEX: Detoxification services, for substance abuse seizure, GZB2ZZZ HZ2ZZZZ

23. PROCEDURE: Intellectual and psychoeducational psychological test Character Code Explanation Section G Mental Health Section Body Z None System Root Type 1 Psychological Tests Type 2 Intellectual and Qualifier psychoeducational Qualifier Z None Qualifier Z None Qualifier Z None

INDEX: Psychological tests, intellectual and psychoeducational, GZ12ZZZ

Chapter 9 A "use additional code" note appears under code Diseases of the Nervous System G90.4 to use an additional code for the cause, as In this case the urinary tract Infection. There is

also a note under code N39.0 to use additional Review Exercises: Chapter 9 code (B95–B97) to identify infectious agent. In

this example, there is no mention of the 1. G30.0 Alzheimer’s disease or sclerosis, see infectious agent so no code is applied. Disease, Alzheimer’s, early onset, with behavioral disturbance 6. Idiopathic normal pressure hydrocephalus F02.81 Dementia, in Alzheimer’s disease, G91.2 Hydrocephalus, normal pressure. see Disease, Alzheimer’s There is no Index entry for idiopathic under There is mandatory sequencing for these codes. hydrocephalus but when the Tabular List is The etiology (Alzheimer’s disease) is sequenced referenced, the title of code G91.2 includes first and the manifestation (dementia) is (Idiopathic) normal pressure hydrocephalus so sequenced second. The Index provides the the term idiopathic in parentheses means the following documentation: Alzheimer’s, early presence or absence of the term does not impact onset, without behavioral disturbance G30.0 the code assignment. [F02.81]. The use of the brackets in the Index indicates manifestation codes. Further the note 7. G45.9 Attack, attacks, transient ischemic in the Tabular at the G30 category states to use (TIA) an additional code to identify dementia without E11.40 Diabetes, diabetic, (mellitus) (sugar) behavioral disturbance (F02.81) At the F02 type 2, with, neuropathy category, the note states to code first the G43.119 Migraine, classical—see Migraine, underlying physiological condition. with aura Migraine, with aura, intractable 2. G40.319 Epilepsy, juvenile myoclonic—see The TIA is the first listed diagnoses as it was the epilepsy, generalized, idiopathic. Epilepsy, reason for the encounter. The migraine is generalized, idiopathic, intractable, without documented as classical. In ICD-10-CM, status epilepticus classical migraine is classified to with aura.

3. Episodic cluster headache, not described as 8. G70.01 Myasthenia., gravis, in crisis intractable The Index entries for myasthenia is G44.019, Headache, cluster, episodic, not straightforward. Another description of intractable myasthenia gravis that could code to G70.01 The Index entry is straightforward. The G44.019 would be myasthenia gravis with acute is the default code for an episodic cluster exacerbation or simply with exacerbation headache that has no documentation of whether the headache is intractable or not. This would be 9. G31.2 Encephalopathy, alcoholic a reasonable query for the physician as It would F10.20 Alcoholism contribute to the best coding for the condition. “Code also associate alcoholism (F10.-)” appears under code G31.2. This direction is not 4. Chronic Migraine without aura, not intractable provided in the Index and demonstrates the with status migrainous requirement to review the Tabular List for G43.701 Migraine, chronic, not intractable, complete coding instructions. with status migrainous The Index entries for migraine must be followed 10. G90.523 Syndrome, pain, complex regional I, closely for such entries as "with" and "without" lower limb G90.52-Index entries include aura, persistent or chronic, intractable or not syndrome, complex regional pain—see intractable, with or without status migrainous. Syndrome, pain, complex regional with additional subterms for lower limb, specified site 5. Autonomic dysreflexia due to urinary tract NEC and upper limb. infections G90.4 Dysreflexia, autonomic 11. Intracranial subdural abscess due to methicillin N39.0 Infection, urinary (tract) resistant Staphylococcus aureus G06.0 Abscess, intracranial or Abscess, weakness, muscle atrophy and possibly fatigue subdural, brain. There is a use additional code and muscle pain. This code is not used to (B95–B97) to identify the infectious organism. identify polio as the late effect of a current B95.62 Infection, staphylococcal, as cause of residual condition as is the intent of code B91. disease classified elsewhere, aureus, methicillin resistant 16. PROCEDURE: Diagnostic lumbar spinal puncture 12. Carpal tunnel syndrome and tarsal tunnel Character Code Explanation syndrome, both on left side Section 0 Medical and Surgical G56.02 Syndrome, carpal tunnel, left upper limb Body 0 Central Nervous System G57.52 Syndrome, tarsal tunnel, left lower limb System The Index entries are straightforward. No site is Root 9 Drainage included in the Index, such as upper limb or Operation lower limb. The Index can be trusted as carpal Body Part U Spinal Canal tunnel syndrome only occurs in the upper limb Approach 3 Percutaneous and tarsal tunnel syndrome only occurs in the Device Z No Device lower limb. Qualifier X Diagnostic

13. Spastic diplegic cerebral palsy INDEX: Puncture, see Drainage, (lumbar) spinal G80.1 Palsy, cerebral, spastic, diplegic canal, 009U. Diagnostic procedure uses qualifier X, The Index entries for spastic cerebral palsy and procedure is percutaneous with needle passing spastic diplegic cerebral palsy both code to through skin into lumbar spinal canal to obtain G80.1 spinal fluid for analysis. No device is left in the body for a diagnostic lumbar puncture 14. Dementia with Parkinsonism G31.83 Dementia, with Parkinsonism 17. PROCEDURE: Transposition/reposition radial G31.83, [F02.80] nerve, open, right arm F02.80 Dementia in other diseases Character Code Explanation classified elsewhere without behavioral Section 0 Medical and Surgical disturbance Body 1 Peripheral Nervous System The Index must be followed closely for this System condition. Under main term Dementia, the coder Root S Reposition must note the connecting term “with” directly Operation under the main term and two entries exist that Body Part 6 Radial Nerve produce different codes: Parkinson’s Disease G20 and Parkinsonism G31.83. Another Approach 0 Open potential error would be if the coder accessed Device Z No Device the main term Dementia “in” Parkinson’s Qualifier Z No Qualifier Disease G20, [F02.80]. If the coder accessed “Parkinsonism” code G20 would be found. INDEX: Transposition see Reposition, Nerve, radial When the coder accessed the Tabular List with 01S6. The approach is stated as open and there is no G20, an important Excludes1 note is found: option for a device or qualifier value for the Dementia with Parkinsonism (G31.83). In this procedure. example, the coder cannot exchange the term Parkinson’s Disease with Parkinsonism.

15. Postpolio Syndrome G14 Syndrome, postpolio (myelitic) The Index entry is straightforward to code G14. In the Tabular List, there is an Excludes1 note under code G14 that excludes code G14 being used with sequelae of poliomyelitis. Postpolio syndrome is a current condition in a patient who had poliomyelitis in the past but after a stable period of time, the patient develops new muscle 18. PROCEDURE: Ulnar nerve release by fasciotomy, left arm Character Code Explanation Section 0 Medical and Surgical Body 1 Peripheral Nervous System System Root N Release Operation Body Part 4 Ulnar Nerve Approach 0 Open Device Z No Device Qualifier Z No Qualifier

INDEX: Release, Nerve, Ulnar 01N4. Fasciotomy is an open approach

19 . PROCEDURE: Cerebral ventricular-peritoneal shunt using synthetic shunt material for shunt creation via open technique Character Code Explanation Section 0 Medical and Surgical Body 0 Central Nervous System System Root 1 Bypass Operation Body Part 6 Cerebral Ventricle Approach 0 Open Device J Synthetic Substitute Qualifier 6 Peritoneal Cavity

INDEX: Shunt creation, see Bypass, cerebral ventricles 0016. Draft coding guideline B3.6a states the procedure is coded by identifying the body part bypassed to and the body part bypassed from. The fourth character body part specifies the body part bypassed from and the qualifier specifies the body part bypassed to.

20. PROCEDURE: Open neurorrhaphy, left tibial nerve Character Code Explanation Section 0 Medical and Surgical Body 1 Peripheral Nervous System System Root Q Repair Operation Body Part G Tibial Nerve Approach 0 Open Device Z No Device Qualifier Z No Qualifier

INDEX: Neurorrhaphy see Repair peripheral nerve (tibial) 01QG. The procedure is identified as an open procedure repairing the tibial nerve. Chapter 10 Because of the numerous codes for retinal Diseases of the Eye and Adnexa detachment, the coder must follow the Index carefully and refer to the Tabular List to find the

code for the right, left or it the condition is Review Exercise: Chapter 10 bilateral.

8. Acute chemical conjunctivitis, right eye 1. H35.361 Drusen, macular—see H10.1 Conjunctivitis, acute, chemical Degeneration, macula, drusen, right Main term is conjunctivitis, acute, chemical. The main term in the Index is drusen, macular There is a "see also" note to locate corrosion, directs the coder to the main term degeneration cornea in the Index. Corrosion, cornea gives because a drusen is a degenerative condition the code T26.6-. However, no mention of

corrosion is included in the diagnosis 2. H52.213 Astigmatism, irregular statement. Plus under by both codes H10.1 The main term, astigmatism, has three choices and T26.6 is the direction to "code first (T51– for types, that is, irregular, regular and T65) to identify chemical and intent, meaning unspecified there had to be a chemical that caused the

conjunctivitis, probably a substance was 3. H26.012 Cataract, presenile, cortical splashed into the eye. The coder must review The presenile description of the cataract directs the record again because the coding cannot be the coder to infantile and juvenile cataracts with completed without the information about the specific codes for cortical type. chemical involved.

4. H02.811 Foreign body, retained, eyelid, right, 9. Bilateral retinopathy of prematurity, stage 2 upper H35.133 Retinopathy, of prematurity, stage 2, Z18.10 Retained, foreign body fragments, bilateral metal The Index entries for this condition is A “use additional code” to identify the type of straightforward, the Tabular must be used to retained foreign body (Z18.1-) is the reason for find the sixth character for the bilateral nature the second code to describe the foreign body of the condition fragments as metal.

10. Monocular exotropia with V pattern, left 5. H40.1231 Glaucoma, low tension—see eye Glaucoma, open angle, primary, low tension, H50.132 Exotropia—see Strabismus, mild, bilateral divergent concomitant, monocular, with, V Many of the codes for glaucoma require a pattern, left eye. seventh character to identify the stage of The Index entries direct the code to glaucoma. Given the various terminology used strabismus after the condition of exotropia is by physician for glaucoma conditions, the coder first used. The strabismus index entries must must follow the Index to Diseases and Injuries in be followed closely and the laterality of the ICD-10-CM carefully to identify the appropriate eye is used for the sixth character in this code. example.

6. H21.81 Syndrome, floppy iris 11. Low vision, visual impairment category T44.6X5D There is an instruction under code two, both eyes H21.81 to “Use additional code” for adverse H54.2 Low, vision effect, if applicable, to identify drug (T36–T50) The Index entry gives a default code H54.2 with fifth or sixth character (5). Because it was for low vision. The other entry under low, known the patient was taking Flomax, the coder vision, addresses one eye, left or right, with locates the drug in the Table of Drugs and comparison to the other eye. No Index entry Chemical, Flomax, adverse effect to get the T44 under low vision address the terminology of code. visual impairment or category. When the

Tabular List is accessed code H54.2 is "Low 7. H33.012 Detachment, retina wit retinal break vision, both eye" with an inclusion term single, left eye beneath it that states "visual impairment categories 1 or 2 in both eyes." There is a "code first any associated underlying cause of 16. Procedure: Removal of foreign body (glass) the blindness" note under category H54, from left cornea Blindness and low vision. In this example, Character Code Explanation there is no other diagnosis mentioned and this Section 0 Medical and surgical patient has low vision, not blindness. There is Body System 8 Eye a table in the Tabular List that includes the Root C Extirpation definitions of visual impairment categories. Operation Body Part 9 Cornea, left 12. H59.012 Keratopathy, bullous (aphakic), Approach X External following cataract Device Z No Device Qualifier Z No Qualifier Bullous keratopathy, or corneal edema, is often sequelae of cataract extraction. In ICD-10-CM, INDEX: Extirpation (removal of foreign body) codes for both keratopathy and keratopathy due cornea left 08C9XZZ to cataract surgery are provided. These codes are further subdivided by laterality. 17. Procedure: Transnasal endoscopy for dilation and stent placement In left lacrimal duct 13. Chronic iridocyclitis and cataract with Character Code Explanation neovascularization, right eye Section 0 Medical and surgical H20.11 Iridocyclitis, chronic, right Body System 8 Eye Under subcategory H20.1-, there is a note Root 7 Dilation "use additional code for any associated Operation cataract (H26.21-) Body Part Y Lacrimal duct, left H26.211 Cataract, with neovascularization—see Cataract, Approach 8 Via Natural or Artificial complicated, with neovascularization, Opening Endoscopic H26.21- Device D Intraluminal Duct Under category H26.2-, there is a code to Qualifier Z No Qualifier "code also associated condition, such as: chronic iridocyclitis (H20.1-) INDEX: Dilation, duct, lacrimal, left 087Y Neither of these notes provide a mandatory first listed code; the first-listed or principal 18.Procedure: Penetrating keratoplasty of right diagnosis code could be either condition cornea with donor matched cornea for transplant, depending on the circumstances of the visit or percutaneous approach admission as the phrase "code first" is not Character Code Explanation included Section 0 Medical and surgical Body 8 Eye 14. H04.331 Canaliculitis (lacrimal) (acute) right System eye Root R Replacement Operation 15. H35.039 Retinopathy, hypertensive Body Part 8 Cornea, right (unspecified as to which eye or both) Approach 3 Percutaneous I10 Hypertension (essential) Device K Nonautologous Tissue Substitute Qualifier Z No Qualifier INDEX: Keratoplasty is a replacement of a cornea. Index: Replacement, cornea 08R8. Donor tissue is from a matched cornea 19.Procedure: Cataract extraction by phacoemulsification, left eye, with prosthetic lens immediate insertion Character Code Explanation Section 0 Medical and surgical Body 8 Eye System Root R Replacement Operation Body Part K Lens, left Approach 3 Percutaneous Device J Synthetic substitute Qualifier Z No Qualifier

INDEX: Phacoemulsification, lens, with IOL implantation, see Replacement, eye 08R. It is important to refer to the PCS Tables to construct the code, even when an entry in the Index gives a complete seven character code in the Index such as Replacement, lens left 08RK30Z—need to confirm in Tabular, especially device. The sixth character must be “J” for the synthetic substitute, the prosthetic lens used in the eye.

20. Lamellar keratoplasty, supplement onlay type, right cornea, using autograft Character Code Explanation Section 0 Medical and surgical Body System 8 Eye Root U Supplement Operation Body Part 8 Cornea, right Approach X External Device 7 Autologous Tissue Substitute Qualifier Z No Device

INDEX: Keratoplasty, lamellar is a supplement type procedure, replacing part of the cornea—see Supplement, eye 08U or Supplement, cornea, 08U8

Chapter 11 intraoperative and postprocedural complications Diseases of the Ear and Mastoid available. Subcategory H95.3 provides codes for accidental puncture and laceration of the ear and Process mastoid process when a procedure on the ear and mastoid process was being performed Review Exercise: Chapter 11 (H95.31) and for accidental puncture and laceration of ear and mastoid process during 1. H65.23 Otitis, media, serous—see Otitis, other procedures. media, nonsuppurative, chronic, serous, bilateral 6. H93.13 Tinnitus—see subcategory H93.1, bilateral 2. H71.92 Cholesteatoma (ear)(middle) left 7. A46 Otitis, externa, in, erysipelas 3. H65.02 Otitis, media (hemorrhagic) H62.41 Otitis, externa in other diseases, (staphylococcal) (streptococcal) acute, subacute, right ear serous—see Otitis, media, nonsuppurative, acute, serous. Otitis media, nonsuppurative, 8. H83.3X3 Trauma, acoustic see subcategory acute or subacute, serous H83.3 for bilateral H72.821 Perforation, perforated (nontraumatic) (of), tympanum, tympanic 9. H60.541 Otitis, externa, acute, eczematoid, (membrane) (persistent post-traumatic) right (postinflammatory), total Otitis media has an expansion of codes in ICD- 10. H66.011 Otitis, media, suppurative, acute, 10-CM to classify these conditions. Laterality is with rupture of ear drum, right also part of the classification in ICD-10-CM. In category H65, distinction is made between 11. H70.01 Abscess, mastoid—see recurrent infections. A note is present stating Mastoiditis, acute, subperiosteal, right ear that an additional code for any associated perforated tympanic membrane should be coded 12. H66.42 Otitis, media, suppurative, left ear separately. It is then possible to show which H72.02 Perforation, tympanic (membrane), tympanic membrane is perforated by assigning central, left ear the correct code for right side. Under category H72, Perforation of tympanic membrane is the instructional note: Code first 4. H81.02 Vertigo, Ménière’s—see any associated otitis media (H65.-, H66.1-, subcategory H81.0 H66.2-, H66.3-, H66.4-, H66.9-, H67.-) The Index provides the category and the Tabular provides the specific laterality. Ménière’s 13. H93.11 Tinnitus—see subcategory H93.1 disease involves the inner ear and symptoms are right ear vertigo, tinnitus, and a feeling of fullness or H92.02 Otalgia - see category H92.0 left pressure in the ear. ear

5. H80.03 Otosclerosis (general) involving 14. H68.021 Salpingitis, eustachian (tube), oval window, nonobliterative chronic, right ear H90.0 Loss (of), hearing—see also Deafness. Deafness, conductive, bilateral 15. H95.112 Complication, ear, H95.31 Complication(s), ear procedure, postoperative—see Complication, laceration—see Complications, intraoperative, postmastoidectomy, Inflammation, chronic, puncture or laceration, ear. Complication(s) left ear intraoperative, puncture or laceration (accidental) (unintentional) (of) ear, during procedure on ear and mastoid process The otosclerosis is listed first since it is the underlying condition causing the hearing loss, and absent any sequencing instruction in the classification system. Note that there are 16. Procedure: Stapedectomy (removal of entire 19. Bilateral myringotomy and placement of tubes stapes), right ear behind the tympanic membrane: Character Code Explanation Section 0 Medical and Surgical Two codes required with two coding table Body System 9 Ear, Nose, Sinus explanations Root T Resection Character Code Explanation Operation Section 0 Medical and Surgical Body Part 9 Auditory Ossicle, Right Body System 9 Ear, Nose, Sinus Approach 0 Open Root 9 Drainage Device Z No Device Operation Qualifier Z No Qualifier Body Part 8 Tympanic Membrane, Left Approach 7 Via Natural or Artificial INDEX: Stapes—use auditory ossicle, right. Opening Stapedectomy, see Resection, ear, middle, right Device 0 Drainage Device 09T Qualifier Z No Qualifier

17. Procedure: Removal of impacted foreign body INDEX: Myringotomy, see Drainage, Ear, (bead) left external auditory canal through the ear Tympanic membrane (Left 0998) canal Character Code Explanation Character Code Explanation Section 0 Medical and Surgical Section 0 Medical and Surgical Body System 9 Ear, Nose, Sinus Body System 9 Ear, Nose, Sinus Root 9 Drainage Root C Extirpation Operation Operation Body Part 7 Tympanic Membrane, Right Body Part 4 External Auditory Canal, Approach 7 Via Natural or Artificial Left Opening Approach 7 Via Natural or Artificial Device 0 Drainage Device Opening Qualifier Z No Qualifier Device Z No Device Qualifier Z No Qualifier INDEX: Myringotomy, see Drainage, Ear, Tympanic membrane (Right 0997) INDEX: Removal of foreign body is not in the PCS Index. The coder must know the 20. Excision of cholesteatoma, left middle ear definition of root operations to—see Character Code Explanation Extirpation, ear, external auditory, canal, left Section 0 Medical and Surgical 09C4 (through ear canal) Body System 9 Ear, Nose, Sinus Root B Excision 18. Procedure: Endoscopic partial nasal Operation turbinectomy Body Part 6 Middle Ear, Left Character Code Explanation Approach 0 Open Section 0 Medical and Surgical Device Z No Device Body System 9 Ear, Nose, Sinus Qualifier Z No Qualifier Root B Excision INDEX: Excision, ear, middle, left 09B60Z Operation Body Part L Nasal Turbinate Approach 8 Via Natural or Artificial Opening Endoscopic Device Z No Device Qualifier Z No Qualifier INDEX: Removal of foreign body—see Extirpation, ear, external auditory, canal, left 09C4 (through ear canal)

Chapter 12 B95.61 Infection, bacterial, as cause of Diseases of the disease classified elsewhere, Staphylococcus aureus Circulatory System I47.2 Tachycardia, ventricular

Review Exercises 10. Arteriosclerotic coronary artery disease with unstable angina, no history of coronary artery 1. Acute cerebral infarction with left nondominant bypass surgery hemiparesis and dysphasia I25.110 Arteriosclerosis, coronary (artery), I63.9 Infarction, cerebral native vessel, with, angina pectoris, unstable G81.94 Hemiparesis—see Hemiplegia, unspecified, affecting left nondominant side 11. Arteriosclerosis of the right lower extremity R47.02 Dysphasia native arteries with rest pain These are current manifestations of an acute I70.221 Arteriosclerosis, extremities (native infarction. Code such as I69.321, dysphasia arteries), leg, right, with, rest pain following cerebrovascular disease, cerebral infarction, would not be correct as the I69 12. End-stage renal disease (ESRD) with category are sequel of cerebral infarction and hypertension coded only after the acute phase of the infarction I12.0 Disease, renal, end-stage, due to is treated. hypertension N18.6 Disease, renal, end-stage 2. Acute pericardial effusion ICD-10-CM presumes a cause-and-effect I30.9 Effusion, pericardial—see relationship when the diagnoses of ESRD and Pericarditis, acute hypertension are present in the same patient so that hypertensive chronic kidney disease is 3. Chronic atrial fibrillation; Essential hypertension coded. I48.2 Fibrillation, atrial, chronic I10 Hypertension (essential) 13. Inflamed varicose veins of the right lower extremity with development of calf ulcer 4. Coronary artery disease in autologous vein I83.212 Varix, leg, right, ulcer, calf, with bypass graft inflammation I25.810 Disease, coronary (artery)—see Disease, heart, ischemic, atherosclerotic—see 14. Occlusive disease of iliac artery right side. Arteriosclerosis, coronary (artery), bypass graft, I74.5 Occlusion, artery, iliac autologous vein 15. PROCEDURE: Coronary artery bypass graft 5. Venous thrombosis of greater saphenous vein, (CABG) using 4 saphenous veins for right leg aortocoronary bypass I82.811 Thrombosis, vein, saphenous Character Code Explanation (greater) vein, right Section 0 Medical and Surgical Body 2 Heart and Great Vessels 6. Thoracic aortic aneurysm System I71.2 Aneurysm, aortic, thoracic Root 1 Bypass Operation 7. Mitral valve insufficiency Body Part 3 Coronary Artery, Four or I34.0 Insufficiency, mitral (valve) More Sites Approach 0 Open 8. Acute myocardial infarction (STEMI) of Device 9 Autologous Venous Tissue posterolateral wall Qualifier W Aorta I21.29 Infarction, myocardial, ST

elevation, posterior INDEX: Bypass, artery, coronary, four or more sites

0213 9. Subacute bacterial endocarditis secondary to

Staphylococcus aureus; ventricular tachycardia

I33.0 Endocarditis, bacterial PROCEDURE Cardiopulmonary Bypass 18. PROCEDURE Percutaneous insertion of central Character Code Explanation venous catheter infusion device, subclavian, left Section 5 Extracorporeal Assistance Character Code Explanation and Performance Section 0 Medical and Surgical Body A Physiological Systems Body 5 Upper Veins System System Root 1 Performance Root H Insertion Operation Operation Body Part 2 Cardiac Body Part 6 Subclavian Vein, Left Approach 2 Continuous Approach 3 Percutaneous Device 1 Output Device 3 Infusion Device Qualifier Z No Qualifier Qualifier Z No Qualifier

INDEX: Bypass, cardiopulmonary 5A1221Z INDEX: Insertion of device in, vein, subclavian, left 05H6 16. PROCEDURE Replacement of Mitral Valve using porcine graft 19. PROCEDURE Ablation, right atrium, Character Code Explanation percutaneous (MAZE procedure) Section 0 Medical and Surgical Character Code Explanation Body 2 Heart and Great Vessels Section 0 Medical and Surgical System Body 2 Heart and Great Vessels Root R Replacement System Operation Root 5 Destruction Body Part G Mitral Valve Operation Approach 0 Open Body Part 6 Atrium, Right Device 8 Zooplastic Tissue Approach 3 Percutaneous Qualifier Z No Qualifier Device Z No Device Qualifier Z No Qualifier INDEX: Replacement, valve, mitral 02RG INDEX: Ablation, see Destruction, atrium, right 17. PROCEDURE Percutaneous Transluminal 0256 Coronary Angioplasty, 2 vessels, no stents Character Code Explanation 20. PROCEDURE PTCA, via femoral approach, 2 Section 0 Medical and Surgical vessels with insertion of drug eluting stent into same Body 2 Heart and Great Vessels two vessels System Character Code Explanation Root 7 Dilation Section 0 Medical and Surgical Operation Body 2 Heart and Great Vessels Body Part 1 Coronary Artery, Two Sites System Approach 3 Percutaneous Root 7 Dilation Device Z No Device Operation Qualifier Z No Qualifier Body Part 1 Coronary Artery, Two Sites Approach 3 Percutaneous INDEX: Angioplasty, see Dilation, Heart and Great Device 4 Intraluminal Device, Drug- Vessels 027 eluting Qualifier Z No Qualifier

INDEX: PTCA see Dilation, Heart and Great Vessels 027

Chapter 13 This diagnosis does not specify the type of Diseases of the Respiratory System influenza responsible for the pneumonia, hence, the use of category J11 code for influenza due to

unidentified influenza virus Review Exercises

9. J30.1 Fever, hay, due to, pollen, any plant or tree 1. J14 Pneumonia, Hemophilus influenzae Excludes1 note exists to prevent this code from (broncho) (lobar) being used with allergic rhinitis with asthma or The H. influenzae pneumonia is coded to J14. unspecified rhinitis The symptoms are not coded because they are

inherent in the pneumonia code. 10. J38.02 Paralysis, vocal cords, bilateral

Codes exist for unilateral, bilateral and 2. J44.1 Disease, diseased, pulmonary, unspecified vocal cord paralysis. The term chronic obstructive, with exacerbation (acute) “complete” vocal cord paralysis is not indexed, F17.200 Dependence (on) (syndrome), physician query or documentation search must tobacco—see Dependence, drug, nicotine be performed to identify the meaning. Use The acute respiratory insufficiency is a symptom additional code to identify exposure to or use of that is an integral part of the COPD and is not tobacco note if it was documented coded.

11. J93.0 Pneumothorax, tension 3. J45.51 Asthma, asthmatic (bronchial) Tension pneumothorax is specifically indexed (catarrh) (spasmodic), persistent, severe, with which is a different code of spontaneous exacerbation (acute) pneumothorax. Excludes1 note exist with There are categories of the three degrees of category J93 pneumothorax to prevent coding of persistent asthma, with the ability to identify these pneumothorax conditions with other with or without exacerbation and status specified types of pneumothorax asthmaticus.

12. J37.1 Laryngotracheitis, chronic 4. J35.3 Hypertrophy, tonsils, with adenoids An excludes2 note exists with code J37.1 that There is an Excludes1 note under code J35.3 that allows coding for acute laryngotracheitis or hypertrophy of tonsils and adenoids with acute tracheitis. tonsillitis and adenoiditis is coded to J35.03,

chronic tonsillitis and adenoiditis. 13. J95.811 Pneumothorax, postprocedural

Unlike other postprocedural conditions coded in 5. J41.0 Bronchitis, chronic, simple this department this code does not require the There is an Excludes1 note under code J42, identity of the procedure such as respiratory chronic bronchitis NOS. The term phrase system procedure or other procedure chronic “simple” bronchitis is the diagnosis that

leads to code J41.0. 14. J60, Black, lung

Other terminology for the same condition is 6. J43.1 Emphysema, panlobular coalworkers’ pneumoconiosis or lung. If the physician documents the use or exposure Excludes1 note exists to prevent coding of the to tobacco, there is a “use additional code” to same condition with tuberculosis. identify the conditions related to tobacco with

emphysema

7. J13 Pneumonia, Streptococcus pneumoniae J85.1 Abscess, lung, with pneumonia Under category J13, there is a note to code also associated lung abscess, if applicable, J85.1. Under category J85.1, there is a code also the type of pneumonia note

8. J11.00 Influenza with pneumonia 15. PROCEDURE: Tracheostomy tube exchange 18. PROCEDURE: Bronchoscopic excision of Character Code Explanation lesion of right upper lobe of lung Section 0 Medical and Surgical Character Code Explanation Body B Respiratory System Section 0 Medical and Surgical System Body B Respiratory System Root 2 Change System Operation Root B Excision Body Part 1 Trachea Operation Approach X External Body Part C Upper Lung Lobe, Right Device F Tracheostomy Device Approach 8 Via Natural or Artificial Qualifier Z No Qualifier Opening Endoscopic Device Z No Device INDEX: Tracheostomy Device, change device in, Qualifier Z No Qualifier Trachea 0B21XFZ INDEX: Excision, lung, upper lobe, 0BBC. This is 16. PROCEDURE: Thoracotomy with exploration an excision, not stated as a biopsy of right pleural cavity Character Code Explanation 19. PROCEDURE: Mechanical ventilation for 36 Section 0 Medical and Surgical consecutive hours following endotracheal tube Body W Anatomical Regions, General intubation System Character Code Explanation Root J Inspection Section 5 Extracorporeal Assistance Operation and Performance Body Part 9 Pleural Cavity, Right Body A Physiologic Systems Approach 0 Open (thoracotomy) System Device Z No Device Root 1 Performance Qualifier Z No Qualifier Operation Body Part 9 Respiratory INDEX: Exploration, See Inspection, pleural cavity, Approach 4 24-96 Consecutive hours right 0WJ90ZZ Device 5 Ventilation Qualifier Z No Qualifier 17. PROCEDURE: Laryngoscopy with Endoscopic biopsy of larynx by excision INDEX: Mechanical ventilation, see Performance, Character Code Explanation Respiratory 5A19 Section 0 Medical and Surgical Mechanical ventilation is coded to the extracorporeal Body C Mouth and Throat assistance and performance section. Insertion of the System endotracheal tube as part of a mechanical ventilation Root B Excision procedure is not coded as a separate device insertion Operation procedure, because it is merely the interface between Body Part S Larynx the patient and the equipment used to perform the Approach 8 Via Natural or Artificial procedure, rather than an end in itself. Opening Endoscopic Device Z No Device Qualifier X Diagnostic (biopsy)

INDEX: Biopsy , See Excision with qualifier Diagnostic, Larynx 0CBS. Endoscopy is laryngoscopy which is inserted through mouth into throat

20. PROCEDURE: Thoracotomy with resection of right lower lobe of lung Character Code Explanation Section 0 Medical and Surgical Body B Respiratory System System Root T Resection Operation Body Part F Lower Lung Lobe, Right Approach 0 Open Device Z No Device Qualifier Z No Qualifier

INDEX: Resection, lung, lower lobe, right 0BTF

Chapter 14 ICD-10-CM coding guidelines (II.F.) state when Diseases of the Digestive System original treatment plan is not carried out, the principal diagnosis is the condition after study

which occasioned the admission to the hospital, Review Exercises even though treatment may not have been carried

out due to unforeseen circumstances. In this 1. K40.41 , hernial, (acquired) patient, the unforeseen circumstance that caused (recurrent), inguinal (direct) (external) the surgery to be canceled was the heart failure (funicular) (indirect) (internal) (oblique) and hypertension which is a medical (scrotal) (sliding), unilateral, with, gangrene contraindication. Main term is “canceled” for the (and obstruction), recurrent code to identify the procedure was canceled. When coding , ICD-10-CM provides

specificity by type, laterality, with or without 7. K94.02 Infection, colostomy obstruction and recurrence. L03.311 Cellulitis (diffuse) (phlegmonous)

(septic) (suppurative), abdominal wall 2. K25.0 Ulcer, ulcerated, ulcerating, B95.2 Infection, infected, infective, ulceration, ulcerative, gastric—see Ulcer, bacterial, as cause of disease classified (eroded) (peptic) (round), acute, with, elsewhere, enterococcus hemorrhage The infection of the is sequenced Gastric ulcers are subdivided by severity and first. The note under K94.02 states to use an then further subdivided by hemorrhage and/or additional code to specify type of infection, such perforation. as cellulitis of abdominal wall. The organism

(Enterococcus) is also coded per instructional 3. K80.33 Choledocholithiasis (common duct) note which appears directly under the section (hepatic duct)—see Calculus, bile duct “Infections of the Skin and Subcutaneous Tissue (common) (hepatic), with, cholangitis, acute, (L00–L08)." The note states “Use additional with, obstruction code (B95–B97) to identify infectious agent." ICD-10-CM has provided a combination code

for bile duct calculus with cholangitis. 8. K70.11 Hepatitis, alcoholic (chronic) with

ascites 4. K13.21 Leukoplakia, mouth and F10.20 Alcoholism (chronic) Leukoplakia, tongue The Alphabetic Index must be followed closely ICD-10-CM coding guidelines (I.B.12) specify a for alcoholic hepatitis. Under category K70 is a diagnosis code may be reported only once for an note to “use additional code to identify alcohol encounter. The Index to Diseases and Injuries in abuse and dependence (F10.-) Alcoholism is in ICD-10-CM contains two separate entries for the Alphabetic Index have a nonessential leukoplakia of mouth and leukoplakia of tongue modifier of chronic. The code F10.20 includes but the code is only used once according to the the term “uncomplicated” but the Index clearly coding guideline. directs the coder to this code without the same

terminology and should be trusted. 5. K26.4 Ulcer, duodenum, chronic, with,

hemorrhage 9. K80.12 Cholecystitis, acute, with, chronic D50.0 Anemia, blood loss (chronic) cholecystitis, with calculus Two separate conditions are coded with The Alphabetic Index must be followed closely individual codes. The sequence of the codes for this combination of conditions related to would depend on the circumstance of the gallbladder inflammation and calculus. admission or the reason for the outpatient visit. Cholecystitis, acute with chronic followed by

“with gallbladder calculus” identifies the code. 6. K43.2 Hernia, incisional Another Index entry of cholecystitis with I10 Hypertension calculus, stone in gallbladder – see Calculus, I50.33 Failure, heart, diastolic, acute, and gallbladder with cholecystitis will lead to the chronic same code. Z53.09 Canceled procedures, because of,

contraindication 10. K58.0 Irritable, bowel (syndrome) with diarrhea Category K58, Irritable bowel syndrome, 16. Colonoscopy with excision of colon polyp includes irritable colon and spastic colon. descending colon Character Code Explanation 11. K62.1 Polyp, rectum Section 0 Medical and Surgical The code K62.1 is followed by an Excludes1 Body D Gastrointestinal System note that adenomatous polyp code D12.8 to state System that the two codes are mutually exclusive for the Root B Excision same polyp Operation Body Part M Descending Colon 12. K85.9 Pancreatitis, acute Approach 8 Via Natural or Artificial K86.1 Pancreatitis, chronic Opening Endoscopic Two different category codes are used for the Device Z No Device two conditions. K85 classifies acute pancreatitis. Qualifier Z No Qualifier K86 classifies “other diseases of pancreas” with a specific code for chronic pancreatitis. INDEX: Excision, colon, descending 0DBM. Approach for a colonoscopy is by endoscopy via 13. K91.2 Malabsorption, postgastrectomy or natural opening. Malabsorption, syndrome, postsurgical or Syndrome, malabsorption, postsurgical 17. Laparotomy with resection of a portion of the The code for this condition can be found with small intestine two entries under malabsorption or under the Character Code Explanation term syndrome for code K92.1, Postsurgical Section 0 Medical and Surgical malabsorption, not elsewhere classified. Specific Body D Gastrointestinal System conditions of malabsorption osteomalacia and System osteoporosis are includes in the Excludes1 note Root B Excision that follows. Operation

Body Part 8 Small Intestine 14. K57.00 Diverticulitis, small, with abscess, perforation or peritonitis Approach 0 Open A combination code exists for the diverticulitis, Device Z No Device abscess and perforation. The main term Qualifier Z No Qualifier “diverticulitis” for large intestine includes subterms for with abscess and perforation. The INDEX: Resection by definition in ICD-10-PCS is code K63.1 for perforation of intestine includes the cutting out or off without replacement of ALL of an Excludes1 code for perforation of intestine a body part. This operation title includes phrase with diverticular disease to specify that K63.1 is “portion” of small intestine. Instead the coder should not used with category K57.00 use the root operation of excision. Excision, intestine, small, 0DBB 15. Laparoscopic Character Code Explanation 18. Open cholecystectomy with open Section 0 Medical and Surgical choledocholithotomy Body F Hepatobiliary System and Character Code Explanation System Pancreas Section 0 Medical and Surgical Root T Resection Body F Hepatobiliary System and Operation System Pancreas Body Part 4 Gallbladder Root T Resection Approach 4 Percutaneous Endoscopic Operation Device Z No Device Body Part 4 Gallbladder Qualifier Z No Qualifier Approach 0 Open Device Z No Device INDEX: Cholecystectomy, see Resection, Qualifier Z No Qualifier gallbladder. A cholecystectomy is the removal of the entire gallbladder unless otherwise specified. INDEX: Cholecystectomy, see Resection, Resection, gallbladder 0FT4 gallbladder 0FT4

Character Code Explanation Section 0 Medical and Surgical Body F Hepatobiliary System and System Pancreas Root C Extirpation Operation Body Part 9 Common Bile Duct Approach 0 Open Device Z No Device Qualifier Z No Qualifier

INDEX: Choledocholithotomy, see Extirpation, Duct, Common Bile 0FC9

19. Percutaneous needle biopsy of liver Character Code Explanation Section 0 Medical and Surgical Body F Hepatobiliary System and System Pancreas Root B Excision Operation Body Part 0 Liver Approach 3 Percutaneous Device Z No Device Qualifier X Diagnostic

INDEX: Biopsy, see Excision with qualifier diagnostic. Excision, liver 0FB0

20. Left inguinal herniorrhaphy with mesh (synthetic material) Character Code Explanation Section 0 Medical and Surgical Body Y Anatomical Region, Lower System Extremities Root U Supplement Operation Body Part 6 Inguinal Region, Left Approach 0 Open Device J Synthetic Substitute Qualifier Z No Qualifier

INDEX: Herniorrhaphy with synthetic substance (mesh) see Supplement, anatomical regions, lower extremities (includes inguinal) 0YU. Not stated as laparoscopic so coded as open.

Chapter 15 H01.114 Dermatitis (eczematous), eyelid, Diseases of Skin and Subcutaneous contact—see Dermatitis, eyelid, allergic, left, upper Tissue H01.111 Dermatitis (eczematous), eyelid, contact—see Dermatitis, eyelid, allergic, right, Review Exercises upper L70.0 Acne, cystic 1. L57.0 Keratosis, actinic The reason for this encounter was the contact X32.xxxA (Index to external causes) Exposure, dermatitis due to the use of new eye cosmetics. sunlight There are several different Index terms for the A use additional code note is present to identify dermatitis. This was documented as irritant the source of the ultraviolet radiation (W89, contact dermatitis, but not allergic, so Index X32) appears under category L57, Skin changes contact, irritant, due to cosmetics, L24.3. Careful due to chronic exposure to nonionizing radiation review of the record and Index is indicated. In addition, there is reference to a specific site 2. L73.2 Hidradenitis (axillaris) (suppurative) (upper eyelids) having a separate classification. Under L24, there is an Excludes2 note for 3. L74.513 Hyperhidrosis, localized, primary, dermatitis of eyelid (H01.1-). This means that if soles both conditions are present, both codes may be assigned. The cystic acne is assigned as a 4. L89.154 Ulcer, pressure, stage 4, sacral secondary condition since it was also treated region during the encounter. One code combines the two facts about the pressure ulcer—the site and the stage. There is 8. L03.221 Cellulitis (diffuse) (phlegmonous) no laterality for the sacrum as it is one site. (septic) (suppurative), neck (region) F11.10 Abuse, drug, morphine type 5. L97.211 Ulcer, trophic—see Ulcer, skin, (opioids) —see Abuse, drug, opioid. lower limb (calf) —see Ulcer, lower limb, calf, Z72.89 Behavior, drug seeking right, skin breakdown only Alphabetic Index entries allow for 6. L27.1 Dermatitis due to drugs and straightforward coding of these conditions. The medicaments, (generalized) (internal use), term “abuse” was used for this patient and the localized skin eruption Index for abuse, drug, morphine, directs the T46.4x5A Table of Drug and Chemicals, coder to the entry for abuse, drug, opioid. In Ramipril, Adverse Effect, initial encounter addition, ICD-10-CM provides a code for drug I10 Hypertension, hypertensive (accelerated) seeking behavior using the main term behavior. (benign) (essential) (idiopathic) (malignant) (systemic) 9. L05.01 Cyst, pilonidal, with abscess The reason, after study, for this encounter is the Main term is cyst, type is pilonidal with dermatitis which is an adverse effect to the associated condition of abscess Ramipril. An instructional note in the Tabular under category L27.1 states “use additional code 10. L51.0 Erythema, multiforme (major), for adverse effect, if applicable, to identify drug nonbullous (T36–T50)” to identify drug or substance. Main term is erythema, multiforme. Major is a Following this instruction note, the T46.4X5A is nonessential modifier. The subterm nonbullous sequenced second. The seventh character of is required for coding T46.4X5A indicates this is the initial encounter (A) for this condition. Documentation states 11. L03.125 Lymphadenitis, acute, lower limb, localized dermatitis, and there is a specific code right for that. This documentation does not indicate L03.126 Lymphadenitis, acute, lower limb, long term use of the drug since it was recently left started. Two codes are required as individual codes exist for right and lower limbs. 7. L24.3 Dermatitis (eczematous), contact, irritant, due to, cosmetics 12. L51.3 Syndrome, Stevens-Johnson, toxic 16. PROCEDURE: Excisional debridement of epidermal overlap syndrome subcutaneous tissue and fascia, buttock by open L49.1 Exfoliation, due to erythematous approach conditions according to extent of body surface, Character Code Explanation 10 to 19 percent of body surface Section 0 Medical and Surgical A use additional code note follows category Body J Subcutaneous Tissue and heading for L51, Erythema multiforme. There is System Fascia no mention of associated manifestations or Root B Excision associated adverse drug effect with this Operation diagnostic statement. There is a use additional Body Part 9 Subcutaneous Tissue and code to identify percentage of skin exfoliation Fascia, Buttock using the main term exfoliation. Approach 0 Open Device Z No Device 13. L89.622 Ulcer, pressure, stage 2, heel, left Qualifier Z No Qualifier In ICD-10-CM, pressure ulcers are classified by site and by stage in one combination code. The INDEX: Debridement, excisional—see Excision, Main Index term is, ulcer, and subterm(s) subcutaneous tissue and fascia, buttock pressure, stage and site. 17. PROCEDURE: Skin biopsy, left cheek (face) 14. L40.54 Psoriasis, arthropathic, juvenile Character Code Explanation The index entry psoriasis is the main term, Section 0 Medical and Surgical subterm arthropathic with an indented term Body H Skin and Breast juvenile. There is no note to use an additional System code to further specify the arthropathy. Root B Excision

Operation 15. PROCEDURE: Incision and drainage of abscess Body Part 1 Skin, Face of pilonidal cyst, lower back Approach X External Character Code Explanation Device Z No Device Section 0 Medical and Surgical Qualifier X Diagnostic Body H Skin and Breast System INDEX: Biopsy—see Excision with qualifier Root 9 Drainage Diagnostic. Excision, skin, face 0HB1XZ Operation Seventh character of X is added to identify the Body Part 6 Skin, Back excision as diagnostic because it was a biopsy. Approach X External

Device Z No Device 18. PROCEDURE: Excision of skin lesion, left Qualifier Z No Qualifier cheek (face) Character Code Explanation INDEX: Incision, abscess—see Drainage, skin, back Section 0 Medical and Surgical 0H96. Abscess with a pilonidal cyst is located in the Body H Skin and Breast skin. System

Root B Excision Operation Body Part 1 Skin, Face Approach X External Device Z No Device Qualifier Z No Qualifier

INDEX: Excision, skin, face 0HB1XZ. Seventh character of Z is added to identify the excision that was not stated as a biopsy

19. PROCEDURE: Cryoablation of multiple skin lesions on chest Character Code Explanation Section 0 Medical and Surgical Body H Skin and Breast System Root 5 Destruction Operation Body Part 5 Skin, Chest Approach X External Device Z No Device Qualifier D Multiple

INDEX: Cryoablation—see Destruction, skin, chest 0H55XZ One code is used to code the entire procedure with seventh character of “D” because multiple skin lesions were destroyed

20. PROCEDURE: Cosmetic augmentation mammaplasty, bilateral with synthetic material implanted for the augmentation Character Code Explanation Section 0 Medical and Surgical Body H Skin and Breast System Root 0 Alteration Operation Body Part V Breast, Bilateral Approach O Open Device J Synthetic Tissue Qualifier Z No Qualifier

INDEX: Cosmetic procedures are the root operation “alteration” Mammaplasty—see Alteration, Breast, Bilateral 0H0V

Chapter 16 In ICD-10-CM, a combination code is utilized to Diseases of the Musculoskeletal System report osteoporosis with an associated pathological fracture. When identifying senile and Connective Tissue osteoporosis, the code book directs the coder to age-related osteoporosis. Review Exercises 5. M51.17 Displacement, intervertebral disc, 1. M00.861 Arthritis, arthritic (acute) (chronic) lumbosacral region, with, radiculopathy (nonpyogenic) (subacute), septic (any site except The main term is displacement, subterms spine)—see Arthritis, pyogenic or pyemic (any intervertebral disc and lumbosacral with site except spine), bacterial NEC, knee. Review subterms for neuritis, radiculitis, radiculopathy the Tabular for correct code assignment. or sciatica Most of the codes in this chapter have site and laterality designations. A note is available at 6. M16.0 Osteoarthritis, primary, hip, bilateral subcategory M00.8 stating to Use additional The main term is osteoarthritis in the Alpha code (B96) to identify bacteria. In this case, it Index. The type is described as “primary” which was not specified. is a subterm under osteoarthritis. The site of hip is indented under primary with bilateral indented 2. M08.071 Arthritis, arthritic (acute) (chronic) under hip. (nonpyogenic) (subacute), rheumatoid, juvenile (with or without rheumatoid factor), ankle. 7. M47.16 Spondylosis, with, myelopathy, Review the Tabular for assignment of laterality. lumbar region M08.072 Arthritis, arthritic (acute) (chronic) The main term is spondylosis with connecting (nonpyogenic) (subacute), rheumatoid, juvenile term “with” indented directly under the main (with or without rheumatoid factor), ankle. term. With “myelopathy” includes the different Review the Tabular for assignment of laterality. anatomic region where spondylosis can occur. For juvenile rheumatoid arthritis, there is not a The region of “lumbar” is indented under code to identify bilateral, therefore, both codes, myelopathy. to identify right and left, must be assigned. 8. M22.41 Chondromalacia, right patella 3. M84.551A Fracture, pathological (pathologic), Index entry of chondromalacia with subterm for due to neoplastic disease, femur patella is used for coding this condition. Another C79.51 Carcinoma (malignant), metastatic, subterm that can be used to locate this condition see Neoplasm, secondary. Refer to Neoplasm is knee, patella. The coder cannot substitute the Table, by site, bone, femur, secondary. subterm of “knee” for “patella.” Knee is a joint Z85.118 History, personal (of), malignant while patella is a bone and these are not neoplasm (of), lung interchangeable terms for this condition. Z92.3 History, personal (of), radiation therapy 9. M32.11 Lupus, erythematosus, systemic M84.551A correctly identifies the fracture in with endocarditis the shaft of the right femur. The seventh Index entry for lupus, systemic directs the coder character “A” is used as long as the patient is to lupus, erythematous, systemic. Because receiving active treatment for the fracture. endocarditis is present with the lupus, the Examples of active treatment are: surgical complete Index entry to follow is lupus, treatment, ER encounter, and evaluation and erythematous, systemic, with organ or system treatment by a new physician. The code Z92.3 involvement, endocarditis can be added to show history of radiation therapy if coding is performed to that degree. 10. M86.152 Osteomyelitis, acute, femur, left B95.61 Infection, staphylococcal, 4. M80.08xA Fracture, pathological (pathologic), unspecified site, as cause of disease classified due to osteoporosis, specified cause NEC—see elsewhere, aureus Osteoporosis, specified type NEC, with Main term is osteomyelitis. Indented under pathological fracture. Osteoporosis (female) osteomyelitis is acute. Under acute is the (male), senile—see Osteoporosis, age-related, subterms for the sites, in this question, femur. with current pathologic fracture, vertebra(e) Index gives the code of M86.15 which must be located in the Tabular to locate the code for left 15. PROCEDURE: Right hip replacement using femur, M86.152. Under category M86 is the uncemented metal prosthesis direction to use additional code note appears Character Code Explanation under category M86 states to use additional code Section 0 Medical and Surgical (B95–B97) to identify infectious agent. Coder Body S Lower Joints must be careful accessing the Index for the B95 System code using the main term, infection with Root R Replacement subterm “as cause of disease classified Operation elsewhere.” Body Part 9 Hip Joint, Right Approach 0 Open 11. M96.1 Syndrome, postlaminectomy Device 1 Synthetic Substitute, Metal This condition can be located under two entries Qualifier A Uncemented in the Index: Postlaminectomy syndrome INDEX: Replacement, hip, right 0SR9. A metal Syndrome, postlaminectomy prosthesis is a synthetic substitute with the choice of “1” as the value for the device. Because the method 12. M40.204 Kyphosis, thoracic region of securing the prosthesis in place is specified as M81.0 Osteoporosis, age related “uncemented” the seventh character or qualifier of Two codes required for condition. Kyphosis is “A” is used to describe the procedure. the main term with subterm thoracic region. There is entry for age-related osteoporosis under 16. PROCEDURE: Left knee replacement using the main term Kyphosis. Also there is no uncemented metal prosthesis instructional code under code M40 to use Character Code Explanation additional code, so the coder must realize that a Section 0 Medical and Surgical second code is needed for the osteoporosis. Body S Lower Joints Main term is osteoporosis, subterm, age-related System with code M81.0. No further entry is accessed Root R Replacement because there is no current pathologic fracture Operation present in this example. Body Part D Knee Joint, Left

13. M66.0 Cyst, Baker’s, ruptured Approach 0 Open Ruptured Baker’s cyst of knee Device J Synthetic Substitute Main term is cyst, subterm Baker’s, subterm Qualifier A Uncemented ruptured. Definition is included in the Includes Note under category M66, Spontaneous rupture INDEX: Replacement, knee, left 0SRD A metal of synovium and tendon that rupture is one that prosthesis is a synthetic substitute with the only occurs when a normal force is applied to tissues choice for a knee replacement device as “J” for the that are inferred to have less than normal device value. Because the method of securing the strength. The Excludes2 note state if the rupture prosthesis in place is specified as “uncemented” the occur where an abnormal force is applied to seventh character or qualifier of “A” is used to normal tissue—see injury of tendon by body describe the procedure. region. 17. PROCEDURE: Open Revision of left hip replacement metal prosthesis 14. M23.203 Derangement, knee, meniscus, due Character Code Explanation to old tear or injury, medial (right) Section 0 Medical and Surgical Alpha Index must be used carefully to identify Body S Lower Joints site (knee) and underlying cause (old tear or System injury) of specific site medial meniscus. The Root W Revision Index provides the code M23.20 does not Operation provide the sixth character to specify the right Body Part B Hip Joint, Left side so the Tabular must be used to identify the Approach O Open complete code of M23.203. Device J Synthetic Substitute Qualifier Z No Qualifier

INDEX: Revision of device in, joint, hip, left infection and insertion of spacer device in right hip 0SWB. When a physician uses the term “revision” for the next 8 weeks of antibiotic therapy the coder must confirm the procedure to code is Character Code Explanation consistent with the definition of “revision” which is Section 0 Medical and Surgical “correcting to the extent possible, a portion of a Body S Lower Joints malfunctioning device or the position of a displaced System device.” A revision procedure must involve a Root P Removal procedure on a device. A joint prosthesis is Operation identified with the device value of “J” for synthetic Body Part 9 Hip Joint, Right substitute. Approach 0 Open Device J Synthetic Substitute 18. PROCEDURE: Laminectomy of lumbosacral Qualifier Z No Qualifier disc L5-S1 Character Code Explanation INDEX: Removal of device from joint, hip, right Section 0 Medical and Surgical 0SP9. This is a two part procedure with separate root Body S Lower Joints operations used. “Removal” is the root operation System which is defined as “taking out or off a device from Root B Excision a body part.” This procedure was the removal of a Operation prosthesis. The next procedure is inserting another Body Part 4 Lumbosacral Disc device to take the place of the device removed. The Approach 0 Open root operation of inserting a spacer device meets the Device Z No Device definition of the root operation of “insertion” which Qualifier Z No Qualifier is putting in a non-biological appliance that monitors, assists, performs, or prevents a INDEX: Laminectomy, see Excision, lower joints physiological function but does not physically take 0SB. The coder must confirm the approach used for the place of a body part. the procedure but a laminectomy is typically an open procedure to excise a disc Character Code Explanation Section 0 Medical and Surgical 19. PROCEDURE: Arthroscopic partial medial Body S Lower Joints meniscectomy right knee System Character Code Explanation Root H Insertion Section 0 Medical and Surgical Operation Body S Lower Joints Body Part 9 Hip Joint, Right System Approach 0 Open Root B Excision Device 8 Spacer Operation Qualifier Z No Qualifier Body Part C Knee Joint, Right Approach 4 Percutaneous Endoscopic INDEX: Insertion of device (spacer) in joint, hip, Device Z No Device right 0SH9 The approach is identified as an Qualifier Z No Qualifier arthrotomy which is an open approach so the value of “0” is used as the 5th character. The device left in INDEX: Meniscectomy—see Excision, lower joint place is identified with the 6th character for the or resection, lower joint. Because the procedure is device specified as “8” for spacer. described as partial, the procedure would meet the definition of Excision so the entry of Excision, lower joints 0SB is correct. The arthroscopy part of the procedure title indicates the approach. The value of “4” is used for arthroscopy as it is a percutaneous endoscopic approach.

20. PROCEDURE: Arthrotomy with removal of right hip metal prosthesis due to internal joint Chapter 17 The prostate hypertrophy and urinary Diseases of the Genitourinary System obstruction are coded separately in ICD-10-CM. This note is available under subcategory N40.1:

Use additional code for associated symptoms, Review Exercises when specified: urinary obstruction (N13.8).

There is also a cross reference at code N13.8 1. N03.2 Syndrome, nephritic—see also stating to code, if applicable, any causal Nephritis. Nephritis, nephritic, chronic, with condition first, such as: enlarged prostate diffuse membranous glomerulonephritis. (N40.1). The sequencing of code N17.0 as the The indexing of this code requires close first-listed code is correct because there is an attention. If the term Syndrome, nephritic is instructional note under code N17.0 to code also used, there is a note at nephritic syndrome to see associated underlying condition. Nephritis. There are also terms for nephrotic

syndrome, which provides different codes. 5. N18.6 Disease, diseased, renal, end stage Nephrotic syndrome should not be used in the Z99.2 Dialysis, renal, status or Index for this question as this is a different Dependence, on, renal dialysis condition. The proteinuria and hematuria are The entry for N18.6, ESRD, in the Tabular symptoms and would not be coded. The Index includes the “use additional code to identify entry of nephritis, nephritic, chronic should be dialysis status Z99.2” The same dialysis status used with the connecting term with and diffuse code that identifies the patient is dependent on membranous glomerulonephritis indented hemodialysis or renal dialysis can be found in beneath it. the Index under the main term dialysis or main

term dependence. 2. N30.01 Cystitis (exudative) (hemorrhagic)

(septic) (suppurative), acute, with hematuria 6. N99.521 Complication, , urinary tract, B96.2 Escherichia (E.) coli, as cause of external, infection disease classified elsewhere The coder must convert the diagnosis of Suppurative is a nonessential modifier for “infection” to complication to find this code. cystitis, so it is included in the code. There is a Index entry for Infection, nephrostomy does not combination code for acute cystitis with exist. Entry for Infection, stoma does not exist. hematuria (N30.01). The frequent urination and Index entry for complication, nephrostomy lists pain are integral to the cystitis and not assigned “see” complication, stoma, urinary tract, codes. A note at category N30 states to use external. A subterm “infection” is listed under additional code to identify infectious agent external under this entry. (B95–B97). This code is never in the first

position. 7. N80.0 Endometriosis, uterus

N80.2 Endometriosis, fallopian tube 3. N92.1 Menometrorrhagia N80.1 Endometriosis, ovary The coder must follow the Index carefully and N84.1 Polyp, cervix, mucous not substitute the term “menorrhagia” for There are separate Index entries and separate menometrorrhagia. Menorrhagia is coded with codes to classify the condition when N92.0 which is excessive and frequent endometriosis is identified as occurring at menstruation with regular cycle. The diagnosis specific anatomic sites. There is now of menometrorrhagia is irregular intermenstrual combination code for multiple sites. The polyp bleeding. The similar spelling of the two of the cervix is also coded separately. conditions could cause a coding error

8. N87.1 Dysplasia, cervix, moderate 4. N17.0 Failure, failed, kidney, acute (see The main term of dysplasia and subterms of also Failure, renal, acute). Failure, renal, acute, cervix and moderate provide the code. Another with, tubular necrosis main term, CIN, refers the coder to see N40.1 Hypertrophy, prostate—see Neoplasia, intraepithelial, cervix, grade II, Enlargement, enlarged, prostate, with lower provides the same code. urinary tract symptoms (LUTS)

N13.8 Obstruction, urinary (moderate) 9. N60.11 Fibrocystic, disease, breast—see Mastopathy, cystic, right breast N60.12 Mastopathy, cystic, left breast subterm of “subacute” does not always appear The main term, fibrocystic, subterms disease and under all infectious diseases. In this event, the breast refers the coder to see Mastopathy, cystic diagnosis code for “acute” and “subacute” but it with code N60.1 provided. When the coder may not always be the same answer. looks up N60.1 in the Index, the coder finds there is no code for bilateral so two 15. N97.2 Infertility, female, due to, uterine codes are used for right and left breast anomaly The main term of Infertility, female has a 10. N20.1 Stone (see also calculus), no entry subterm of “due to” listed with uterine anomaly for ureter. Next main term accessed is Calculus, with code N97.2 which has the description of ureter with the default code of N20.1 when there female infertility of uterine origin. No additional is no documentation of present with calculus of codes are required as the specific uterine kidney. There is an Excludes1 note under anomaly is not stated. category N20: excludes calculus of kidney and ureter with hydronephrosis (N13.2) to state the 16. PROCEDURE: Transurethral resection of combination code should be used when both prostate (TURP) conditions exist together. Character Code Explanation Section 0 Medical and Surgical 11. Index entry of main term Cystocele, female, Body V Male Reproductive System midline, with prolapse of uterus, states see System Prolapse, uterus. There is no entry for prolapsed Root B Excision of uterus with cystocele but default code for Operation prolapsed uterus is N81.4. If the coder Body Part 0 Prostate disregards that entry and uses the main term of Approach 8 Via Natural or Artificial cystocele, female, midline, the code of N81.11 is Opening (transurethral) listed. When the code N81.11 is located in the endoscopic Tabular the coder should note the Excludes 1 Device Z No Device note under N81.1. It states Cystocele with Qualifier Z No Qualifier prolapsed of uterus should be coded in the range of N81.2–N81.4. Therefore, the default code of INDEX: TURP 0VB08ZZ—this procedure is an N81.4 should be used. excision or cutting out a portion of the prostate. Compare to Resection, prostate 0VT07ZZ—this 12. N10 Pyelonephritis, acute procedure is a resection or cutting out all of the B96.5 Infection, pseudomonas, as cause of prostate. The operative report was noted as stating disease classified elsewhere the procedure was an excision of the prostate and did When the main term of pyelonephritis, acute not remove the entire prostate. Usually a TURP is an identifies code N10 and the coder locates that excision of the prostate. code in the Tabular, the coder will find a Note appearing under code N10 to use additional code 17. PROCEDURE: Total abdominal hysterectomy (B95–B97) to identify infectious agent as cause Character Code Explanation of another disease Section 0 Medical and Surgical

Body U Female Reproductive System 13. N95.0 Bleeding, postmenopausal System N95.1 Menopause, symptomatic Root T Resection There are two conditions stated in this diagnostic Operation statement. There is a use additional code note Body Part 9 Uterus following N95.1 to code associated symptoms, but no symptoms are specified. If the actual Approach 0 Open symptoms had been documented, other codes Device Z No Device would be added. Qualifier Z No Qualifier

14. N70.03 Salpingo-oophoritis, subacute INDEX: Hysterectomy—See Resection, uterus The main term is salpingo-oophoritis with 0UT9 subterm of “subacute.” The coder must be Four organs are removed during a total abdominal certain to locate the subterm “acute.” The hysterectomy with bilateral salpingo-oophorectomy: uterus, cervix, fallopian tubes and ovaries. Because 18. PROCEDURE Total left laparoscopic each organ is a body part in the table for “0UT” with nephrectomy a unique value for uterus (9), cervix (C), bilateral Character Code Explanation fallopian tubes (7) and bilateral ovaries (2), four Section 0 Medical and Surgical separate ICD-10-PCS procedure codes must be used Body T Urinary System to describe the entire procedure. The approach for System the removal of the four body parts is “open” as a Root T Resection laparoscopic approach is not stated. Operation Body Part 1 Kidney, Left 17. PROCEDURE: Removal of cervix with Approach 4 Percutaneous endoscopic abdominal hysterectomy Device Z No Device Character Code Explanation Qualifier Z No Qualifier Section 0 Medical and Surgical Body U Female Reproductive System INDEX: Nephrectomy—see Resection Urinary System System 0TT The Index refers the coder to the root Root T Resection operation of resection because a nephrectomy is the Operation removal of the entire kidney. The approach is Body Part C Cervix percutaneous endoscopic for the laparoscopy. Approach 0 Open Device Z No Device 19. PROCEDURE Anterior colporrhaphy by vaginal Qualifier Z No Qualifier approach Character Code Explanation INDEX: Resection, cervix 0UTC Section 0 Medical and Surgical Body U Female Reproductive System 17. PROCEDURE: Salpingectomy, bilateral System Character Code Explanation Root Q Repair Section 0 Medical and Surgical Operation Body U Female Reproductive System Body Part G Vagina System Approach 7 Via Natural or Artificial Root T Resection Opening Operation Device Z No Device Body Part 7 Fallopian Tubes, Bilateral Qualifier Z No Qualifier Approach 0 Open Device Z No Device INDEX: Colporrhaphy—see Repair, Vagina 0UQG. Qualifier Z No Qualifier Colporrhaphy is a repair of the vagina and without further specification the root operation of repair is INDEX: Salpingectomy, see Resection 0UT used. The approach through the vagina is through a natural opening. 17. PROCEDURE: Oophorectomy, bilateral Character Code Explanation Section 0 Medical and Surgical Body U Female Reproductive System System Root T Resection Operation Body Part 2 Ovaries, Bilateral Approach 0 Open Device Z No Device Qualifier Z No Qualifier

INDEX: Oophorectomy, see Resection 0UT

20. PROCEDURE Lithotripsy (ESWL) to destroy right renal pelvis small calculus Character Code Explanation Section 0 Medical and Surgical Body T Urinary System System Root F Fragmentation Operation Body Part 3 Kidney Pelvis, Right Approach X External Device Z No Device Qualifier Z No Qualifier

INDEX: Lithotripsy see Fragmentation, renal pelvis, right 0TF3 ESWL are high-energy sound waves that pass through the body without injuring it and break the stone into small pieces. These small pieces move through the urinary tract and out of the body through the natural urination process. The definition of fragmentation is to break solid matter in a body part into pieces.

Chapter 18 3. O91.22 Mastitis (acute) (diffuse) Pregnancy, Childbirth, and the (nonpuerperal) (subacute), obstetric (interstitial) (nonpurulent), associated with, puerperium Puerperium In this case, the mastitis is not classified in a pregnancy or delivery complication; however, Review Exercises further indentation in the Index provides the specificity of a postpartum complication. 1. O13.2 Pregnancy (childbirth) (labor) (puerperium), complicated by, hypertension— 4. Delivery of single liveborn infant, full-term, see Hypertension, complicating, pregnancy, vaginal delivery by cephalic presentation, 40 gestational (pregnancy induced) (transient) weeks of gestation. (without proteinuria). Review the Tabular for O80 Delivery, normal complete code assignment. Z37.0 Outcome of delivery, single, O09.522 Pregnancy (childbirth) (labor) liveborn (puerperium), complicated by, elderly, Z3A.40 Pregnancy, weeks of gestation, 40 multigravida. Review the Tabular for complete weeks code assignment The coder must know the definition of a normal Z3A.26 Pregnancy, weeks of gestation, 26 delivery to determine this scenario would weeks require the main term of delivery with subterm The range of codes is further subdivided by the of normal. The weeks of pregnancy and the trimester for the current encounter. The note at outcome of delivery codes are required to be the beginning of Chapter 15 defines the second coded for all deliveries. trimester as 14 weeks 0 days to less than 28 weeks 0 days. The Index does not provide 5. Normal full-term vaginal delivery by cephalic complete codes; therefore, it is necessary to presentation, 38 weeks of gestation, elderly review the Tabular for complete code multigravida with gestational diabetes that is assignment. diet controlled, single liveborn infant O24.420 Pregnancy, complicated by, 2. O21.0 Pregnancy (childbirth) (labor) diabetes, gestational—see Diabetes, gestational, (puerperium), complicated by, hyperemesis in childbirth, diet controlled (gravidarum) (mild)—see also Hyperemesis, O09.523 Pregnancy, complicated by, elderly, gravidarum (mild) multigravida O23.42 Pregnancy (childbirth) (labor) Z37.0 Outcome of delivery, single, (puerperium), complicated by, infection(s), liveborn urinary (tract). Review the Tabular for complete Z3A.38 Pregnancy, weeks of gestation, 38 code assignment. weeks B96.20 Infection, infected, infective Whenever delivery occurs during the current (opportunistic), bacterial NOS, as cause of admission, and there is an “in childbirth” option disease classified elsewhere, Escherichia coli [E. for the obstetric complication being coded, the coli] “in childbirth” code should be assigned. Either Z3A.16 Pregnancy, weeks of gestation, 16 code O24 or O09 could be principal diagnosis. weeks When a delivery occurs, the principal diagnosis The hyperemesis gravidarum code for this case should correspond to the main circumstances or is specific to weeks of gestation “. . . starting complication of the delivery. The author before the end of the 20th week of gestation.” suspects the diabetes would require more care Note that there are different options for finding than the elderly status. The weeks of gestation this code in the Index. The UTI code does not and the outcome of delivery codes are required require a secondary code for the UTI (as for all deliveries. previously seen in ICD-9-CM) because specificity is found in the code, but there is a 6. Full-term vaginal delivery, complicated by “use additional code” note to identify the second degree perineal laceration, 39 weeks of organism. pregnancy, twin pregnancy, dichorionic/diamniotic, both liveborn infants. O70.1 Delivery, complicated by, laceration, perineum, second degree O30.043 Pregnancy, twin, 10. Pregnancy delivered, single liveborn, vaginal dichorionic/diamniotic, third trimester delivery following prolonged second stage of Z37.2 Outcome of delivery, twins, both labor, 38 weeks of gestation liveborn O63.1 Delivery, complicated by, prolonged Z3A.39 Pregnancy, weeks of gestation, 39 labor, second stage weeks Z37.0 Outcome of delivery, single, Either code O70.1 or O30.043 could be principal liveborn diagnosis based on the circumstances of the Z3A.38 Pregnancy, weeks of gestation, 38 individual record. The weeks of gestation and weeks. the outcome of delivery codes are required for all deliveries. 11. Ectopic pregnancy, tubal, 10 weeks gestation O00.1 Pregnancy, ectopic, tubal 7. Postpartum office visit, 5 days after discharge, Z3A.10 Pregnancy, weeks of gestation, 10 with partial lactation failure weeks O92.4 Failure, lactation, partial Main term is pregnancy with subterms, ectopic. The main term is failure, subterms lactation and There is no mention of complications. partial. This is a postpartum visit with a code According to the note at the beginning of representing a complication primarily related to Chapter 15, an additional code from category the puerperium. No codes are required for weeks Z3A, weeks of gestation, is used to identify the of pregnancy or outcome of delivery because a specific week of the pregnancy. The main term delivery did not occur during this visit. to locate the Z3A code is pregnancy, weeks of gestation, 10 weeks 8. False labor with Braxton Hicks contractions, 32 weeks of pregnancy, undelivered 12. Spontaneous incomplete abortion, 11 weeks O47.03 False, labor, before 37 completed O03.4 Abortion, incomplete (spontaneous) weeks of gestation, third trimester Z3A.11 Pregnancy, weeks of gestation, 11 Z3A.32 Pregnancy, weeks of gestation, 32 weeks weeks The main term is false, with subterms labor and Main term is abortion, incomplete before 37 completed week of gestation which is (spontaneous). There was no mention of in the third trimester that starts at 28 weeks. complication of the spontaneous incomplete Braxton Hicks contractions is included in the abortion. According to the note at the beginning code for false labor. No codes are required for of Chapter 15, an additional code from category outcome of delivery because the patient is Z3A, weeks of gestation, is used to identify the undelivered. However, weeks of gestation are specific week of the pregnancy. The main term required. to locate the Z3A code is pregnancy, weeks of gestation, 11 weeks 9. Office visit for pregnant female, 19 weeks of gestation, with cervical incompetence 13. Induced abortion, complicated by excessive complicating pregnant. Surgical consent signed hemorrhage, 8 weeks for cervical cerclage procedure to be performed O04.6 Abortion, induced, complicated by, the following day at the ambulatory surgery hemorrhage (excessive) center. Z3A.08 Pregnancy, weeks of gestation, 8 O34.32 Pregnancy, complicated by, weeks incompetent cervix, second trimester Main term is abortion, induced, complicated by, Z3A.19 Pregnancy, weeks of gestation, 19 hemorrhage (excessive). According to the note weeks at the beginning of Chapter 15, an additional The main term is pregnancy, with subterms code from category Z3A, weeks of gestation, is complicated by and incompetent cervix. Another used to identify the specific week of the main term that could be used is incompetence, pregnancy. The main term to locate the Z3A cervix, in pregnancy. Fifth character is 2 for code is pregnancy, weeks of gestation, 8 weeks second trimester which includes 19 weeks. Code for the weeks of gestation is required. 14. Mild pre-eclampsia in pregnancy, second trimester, 26 weeks, undelivered O14.02 Pregnancy, complicated by, pre- “products of conception” to identify the focus of the eclampsia, mild procedure is on delivering the infant. The approach Z3A.26 Pregnancy, weeks of gestation, 26 is open as an incision is made to perform a cesarean weeks delivery and is the only option on the code table. Main term is pregnancy with subterm There is no device left in the body. The qualifier complicated by, with subterm pre-eclampsia, identifies the type of cesarean delivery performed: mild. The Tabular is used to identify the classical, low cervical or extraperitoneal. appropriate fifth character to identify the trimester, which is second trimester for this 17. PROCEDURE: Manually assisted delivery question. According to the note at the beginning Character Code Explanation of Chapter 15, an additional code from category Section 1 Obstetrics Z3A, weeks of gestation, is used to identify the Body 0 Pregnancy specific week of the pregnancy. The main term System to locate the Z3A code is pregnancy, weeks of Root E Delivery gestation, 26 weeks Operation Body Part 0 Products of Conception 15. Pregnancy, 32 weeks, placenta previa without Approach X External hemorrhage Device Z No Device O44.13 Pregnancy, complicated by, placenta Qualifier Z No Qualifier previa, without hemorrhage Z3A.32 Pregnancy, weeks of gestation, 32 INDEX: Delivery, manually assisted 10E0XZZ weeks The main term is delivery, with subterm manually Main term is pregnancy with subterm assisted. There is only one option for characters 4 complicated by placenta previa, without through 7 on the table for “10E” The Index provides hemorrhage. The Tabular is used to identify the the complete code. appropriate fifth character to identify the trimester, which is third trimester for this 18. PROCEDURE: Vacuum assisted delivery question. According to the note at the beginning Character Code Explanation of Chapter 15, an additional code from category Section 1 Obstetrics Z3A, weeks of gestation, is used to identify the Body 0 Pregnancy specific week of the pregnancy. The main term System to locate the Z3A code is pregnancy, weeks of Root D Extraction gestation, 32 weeks Operation

Body Part 0 Products of Conception 16. PROCEDURE: Low cervical Cesarean Delivery Approach 7 Via Natural or Artificial Character Code Explanation Opening Section 1 Obstetrics Device Z No Device Body 0 Pregnancy Qualifier 6 Vacuum System

Root D Extraction INDEX: Delivery, vacuum assisted see Extraction, Operation Products of Conception 10D0. Body Part 0 Products of Conception The main term is delivery with subterm vacuum Approach 0 Open assisted. This entry refers the coder to the main term Device Z No Device “extraction” and subterm products of conception. Qualifier 1 Low Cervical The Index provides the complete code for extraction, products of conception, vacuum 10D07Z6. On the INDEX: Delivery, Cesarean see Extraction, Products table “10E”, the qualifier identifies the method of of Conception 10D0. Extraction, products of extraction or instrumentation used to perform the conception, low cervical 10D00Z1. The main term is delivery: low forceps, mid forceps, high forceps, delivery, with subterm Cesarean. That entry directs vacuum, internal version or other. the coder to the main term “extraction” with subterm of products of conception. The root operation is “extraction” of the infant from the uterus. The body part for all codes in the Obstetrics section of codes is 19. PROCEDURE: Induced abortion by laminaria Character Code Explanation Section 1 Obstetrics Body 0 Pregnancy System Root A Abortion Operation Body Part 0 Products of Conception Approach 7 Via Natural or Artificial Opening Device Z No Device Qualifier 6 Laminaria

INDEX: Abortion, laminaria 10A07ZW The main term is abortion with subterm laminaria. The Index provides the complete 7 character code. The only option for the approach to perform an abortion using laminaria is via natural opening (vagina.) The qualifier identifies the method of the abortion: vacuum, laminaria, abortifacient or an option for no qualifier when the method cannot be identified.

20. PROCEDURE: Treatment of incomplete spontaneous abortion by dilation and curettage (extraction) of retained products of conception Character Code Explanation Section 1 Obstetrics Body 0 Pregnancy System Root D Extraction Operation Body Part 1 Products of Conception, Retained Approach 7 Via Natural or Artificial Opening Device Z No Device Qualifier Z No Qualifier

INDEX: Curettage—see Extraction. Extraction, products of conception, retained 10D1 The main term curettage refers the coder to the main term “extraction” with subterm products of conception for the retained products of conception. The fact the abortion is described as “incomplete” means it is an incomplete abortion. The body part for this type of extraction is to identify the products of conception are retained. The D&C is performed through a natural opening (vagina) for the approach. There are no options for a device or qualifier.

Chapter 19 completed weeks (31 weeks, 0 days through 31 Certain Conditions Originating in the weeks, 6 days) P74.1 Newborn (infant) (liveborn) Perinatal Period (singleton), dehydration There is no documentation of withdrawal, which Review Exercises would be coded P96.1. Following sequencing according to the guidelines, the code for birth 1. P36.2 Newborn, (infant) (liveborn) weight is sequenced before the code for (singleton) sepsis (congenital) due to gestational age. In indexing the premature Staphylococcus, aureus newborn, note that “preterm infant” is not an The Z38 category is not assigned, because the option under the term Newborn. It is indexed birth episode did not occur at this encounter. under Preterm infant, newborn. Code A41.0 is incorrect because this encounter was within the 28 days after birth (perinatal 5. Full term newborn, Infant of diabetic mother period) and the newborn codes are to be used. syndrome. Baby was born by cesarean delivery See the Excludes1 note at category A41— in the hospital. Mother has pre-existing diabetes. Excludes1 neonatal (P36.-). This is the only Z38.01 Newborn, born in hospital, by code required because there is no mention of cesarean severe sepsis or organ dysfunction. And the P70.1 Infant, of diabetic mother P36.2 code identifies the organism, so no (syndrome of ) or Newborn, affected by additional code from category B95 is indicated. maternal, diabetes mellitus Baby was born in the hospital by cesarean 2. P59.9 Newborn (infant) (liveborn) delivery and was a full term gestation. Baby is (singleton), hyperbilirubinemia has infant of diabetic mother syndrome. There The birth did not occur at this encounter, so the are two main terms that can be used to access Z38 category is not assigned. the code for the condition: Infant, of diabetic Hyperbilirubinemia without mention of mother or newborn, affected by maternal, prematurity or specified cause is coded to P59.9. diabetes If prematurity was documented, there is a specific code to identify that condition (P59.0). 6. Newborn transferred to Children’s Hospital after birth at local community hospital. Reasons for 3. Newborn, full term, born in hospital, vaginal transfer are premature infant, 32 weeks birth with meconium peritonitistis gestation, birthweight of 1800 grams with grade Z38.00 Newborn, born in hospital 1 intraventricular hemorrhage P78.0 Newborn, affected by, meconium P52.0 Newborn, affected by, hemorrhage, peritonitis intraventricular, grade 1 Baby was born in the hospital by vaginal birth, P07.17 Low, birthweight, with weight of, full term gestation. The complication was 1750-1999 grams meconium peritonitis. Main term for the P07.35 Preterm, newborn, gestational age, principal diagnosis is newborn, born in hospital. 32 completed weeks Peritonitis can be accessed one of two ways: Coding for the services at the hospital where the newborn, affected by, meconium peritonitis or baby was received by transfer does not include a peritonitis, meconium. Z38 code for the newborn as it is only assigned at the hospital where the baby is born. There 4. Z38.01 Newborn (infant) (liveborn) were several reasons for transfer: premature (singleton), born in hospital, by cesarean infant, short gestation and low birth weight and P04.41 Newborn (infant) (liveborn) the intraventricular hemorrhage. The condition (singleton), affected by cocaine (crack) that meets the definition of principal diagnosis P07.14 Weight, 1000 to 2499 grams at birth would be selected. In this scenario the (low)—see Low, birthweight. Low, birthweight hemorrhage was selected as it would most likely (2499 grams or less) with weight of 1000 to be that condition. 1249 grams P07.34 Premature, newborn, less than 37 7. Full term infant with omphalitis with mild completed weeks—see Preterm newborn. hemorrhage, born in hospital by vaginal delivery Preterm, newborn (infant), gestational age 31 Z38.00 Newborn, born in hospital P38.1 Newborn, omphalitis, with mild coded. The principal diagnosis is the newborn hemorrhage twin status, born in hospital by cesarean The principal diagnosis is the newborn status. delivery. The cardiac condition of bradycardia The infection can be one of two ways: main term would be an additional code for this twin. Main of newborn, subterm omphalitis or main term term for the principal diagnosis is newborn with omphalitis, both with subterm of with mild subterms twin, born in hospital, by cesarean. hemorrhage. The cardiac condition’s code may be accessed in the Index under two options: newborn, affected 8. Premature infant, 35 weeks gestation, by, heart abnormalities, bradycardia or under the birthweight 2000 grams with stage 1 necrotizing main term bradycardia, neonatal. enterocolitis, born in hospital by vaginal delivery. 11. Full term infant, born in hospital, vaginal Z38.00 Newborn, born in hospital delivery. Meconium staining P77.1 Enterocolitis, necrotizing, in Z38.00 Newborn, born in hospital newborn, stage 1 P96.83 Staining, meconium (newborn) P07.18 Low, birthweight, with weight of, When the baby is born during the current 2000-2499 grams encounter, the newborn status code Z38.00 code P07.38 Preterm, newborn, gestation age, 35 is the principal diagnosis. An additional code for completed weeks the presence of meconium staining is used for When the baby is born during the current the finding. The main term used is staining with encounter, the newborn status code Z38.00 code subterm of meconium is the principal diagnosis. Additional codes are the enterocolitis using it as the main term with subterms of necrotizing, in newborn, and stage 12. Full term infant, born in hospital, vaginal 1. The premature status of the baby with the low delivery with respiratory distress syndrome, type birthweight and gestational age, with the birth I weight sequenced ahead of the gestational age. Z38.00 Newborn, born in hospital P22.0 Syndrome, respiratory, distress, 9. Single newborn, born in hospital, by Cesarean newborn (type I ) delivery; Birth injury of wound during When the baby is born during the current cesarean delivery encounter, the newborn status code Z38.00 code Z38.01 Newborn, born in hospital, by is the principal diagnosis. An additional code for cesarean the respiratory distress syndrome type I. Main P15.8 Birth, injury, scalpel wound term syndrome with subterm respiratory distress, When the baby is born during the current newborn (type I) is used. Another option would encounter, the newborn status code Z38.00 code be the main term distress, respiratory, newborn is the principal diagnosis. Additional codes are but there is no option for type I or type II and the enterocolitis using it as the main term with gives the code of P22.9 that is an unspecified subterms of necrotizing, in newborn, and stage type. This would not be specific enough for the 1. The premature status of the baby with the low stated condition birthweight and gestational age is also coded with the birth weight sequenced ahead of the 13. Full term infant, born in hospital, cesarean gestational age per the direction under category delivery, with transient neonatal neutropenia, P07. cause unknown Z38.01 Newborn, born in hospital, by 10. Twin newborn, born in hospital, by Cesarean cesarean delivery, full term; Newborn with neonatal P61.5 Neutropenia, neonatal, transitory bradycardia When the baby is born during the current Z38.31 Newborn, twin, born in hospital, by encounter, the newborn status code Z38.01 code cesarean is the principal diagnosis for the status of the P29.12 Newborn, affected by, heart rate newborn by cesarean delivery. An additional abnormalities, bradycardia code is used for the transient neonatal Bradycardia, neonatal neutropenia with the main term of neutropenia Each of the twin newborns would have an with subterms of neonatal and transitory. There individual record. Each twin’s record would be is no entry under “newborn” for the neutropenia. 16. PROCEDURE: Mechanical Ventilation, 112 14. 10 day old infant readmitted for sepsis due to E. consecutive hours following endotracheal intubation Coli Character Code Explanation P36.4 Newborn, sepsis, due to Escherichia Section 5 Extracorporeal Assistance coli and Performance Because this baby was not born during the Body A Physiological Systems current encounter, no code for newborn status System (Z38.0x) would be used. The principal diagnosis Root 1 Performance for sepsis would be accessed in the Index under Operation the main term, newborn, sepsis, due to Body 9 Respiratory Escherichia coli. Another option for coding System would be the use of the main term of “sepsis” Duration 5 Greater than 96 consecutive with subterms of newborn and due to hours Escherichia coli. No addition infection code Function 5 Ventilation from B96.2- would be needed as the infectious Qualifier Z No Qualifier organism is included in the newborn sepsis code. No code from A41 is required for the sepsis as INDEX: Mechanical ventilation—see Performance, the newborn sepsis codes are excluded from the Respiratory, greater than 96 consecutive hours, A41 codes. 5A1955Z Note: Mechanical ventilation is coded to the 15. 21 day old infant readmitted with neonatal extracorporeal assistance and performance section. urinary tract infection due to E. Coli bacteria Insertion of the endotracheal tube as part of a P39.3 Newborn, infection, urinary tract mechanical ventilation procedure is not coded as a B96.20 Infection, bacterial, as cause of separate device insertion procedure, because it is disease classified elsewhere, Escherichia coli merely the interface between the patient and the Because this baby was not born during the equipment used to perform the procedure, rather current encounter, no code for newborn status than an end in itself. On the other hand, insertion of (Z38.0x) would be used. The principal diagnosis an endotracheal tube in order to maintain an airway for the infection would be accessed in the Index in patients who are unconscious or unable to breathe under the main term, newborn, infection, urinary on their own is the central objective of the tract. Another option for coding would be the procedure. Therefore, insertion of an endotracheal use of the main term of “infection” with tube as an end in itself is coded to the root operation subterms of urinary and newborn. A note under INSERTION and the device ENDOTRACHEAL category B93 states to use an additional code to AIRWAY. Refer to Appendix C in the Reference identify organism or specific infection. The main Manual—page C.8–9 term infection, bacterial, as cause of disease classified elsewhere, Escherichia coli is used to 17. PROCEDURE: Insertion of intra-aortic balloon find the additional code of B96.20 for the pump (continuous) unspecified form of the E. Coli organism. Character Code Explanation

Section 5 Extracorporeal Assistance and Performance Physiological A Physiological Systems System Root Operation 0 Assistance Body System 2 Cardiac Duration 2 Continuous Function 1 Output Qualifier 0 Balloon Pump

INDEX: IABP (intra-aortic balloon pump) see Assistance, Cardiac 5A02. The intra-aortic balloon pump’s function is cardiac output performed continuously as a cardiac assist device.

18. PROCEDURE: CPAP (continuous positive 20. PROCEDURE: Percutaneous endoscopic airway pressure) 48 hours insertion of into jejunum Character Code Explanation Character Code Explanation Section 5 Extracorporeal Assistance Section 0 Medical and Surgical and Performance Body D Gastrointestinal System Physiological A Physiological Systems System System Root H Insertion Root Operation 0 Assistance Operation Body System 9 Respiratory Body Part A Jejunum Duration 4 24-96 consecutive hours Approach 4 Percutaneous endoscopic Function 5 Ventilation Device U Feeding Device Qualifier 7 Continuous Positive Qualifier Z No Qualifier Airway Pressure INDEX: Feeding device, insertion of device in, INDEX: CPAP (continuous positive airway jejunum 0DHA. Another Index entry to be used is pressure) see Assistance, respiratory, 24 to 96 “PEJ” for percutaneous endoscopic consecutive hours, continuous positive airway with the complete 7 character code included in the pressure 5A09457. CPAP is a respiratory or Index: 0DHA4UZ. The body part, jejunum, ventilation assistance procedure that provides identifies where the feeding tube is inserted. continues positive airway pressure. The duration by hours is the determining factor for selecting character 5 for duration.

19. PROCEDURE: Diagnostic audiology—hearing screening test using audiometer Character Code Explanation Section F Physical Rehabilitation and Diagnostic Airway Section 1 Diagnostic Audiology Qualifier Root Type 3 Hearing Assessment Body System Z None & Region Type 0 Hearing screening Qualifier Equipment 1 Audiometer Qualifier Z None

INDEX: Audiology, diagnostic, see Hearing assessment, diagnostic audiology F13, Hearing Assessment, F13Z. A screening hearing test can be found in the Index under audiology, diagnostic. It may also be found in the Index under “hearing assessment”. The fact the test is a screening procedure is important for selecting character 5 and the equipment used is the determining factor in selecting character 6.

Chapter 20 Encephalocele is classified in ICD-10-CM to Congenital Malformations, five possible codes. An encephalocele is defined as a congenital malformation in which brain Deformations and Chromosomal tissue protruding through a skull defect. The Abnormalities main term is encephalocele, with subterms for the specific locations, for example, frontal, Review Exercises nasofrontal, occipital or other specified site. .

1. Spina bifida, lumbar region, without 6. Q37.4 Cleft, (congenital) lip (unilateral), hydrocephalus bilateral, with cleft palate, hard with soft Q05.7 Spina bifida, lumbar Careful review of the documentation is The main term spina bifida has subterms indicated to select the one code that combines according to the location of the spine involved these conditions. Cleft lip and palate are such as cervical, dorsal, lumbar, lumbosacral, congenital defects caused when the bones and sacral, thoracic or thoracolumbar. The subterm tissues don’t fuse together in utero. The palate is for the location presumes that no hydrocephalus the roof of the mouth, and consists of the soft is present. If hydrocephalus is present, a subterm (back part near the throat) and the hard (front appears under the location for “with part behind the teeth) palates. Frequently cleft hydrocephalus.” This question states that lip and palate are both present. A cleft lip can be hydrocephalus is not present. either unilateral or bilateral. The unilateral cleft lip has a gap on one side of the lip under either 2. Coloboma of right eye iris the left or right nostril, but in a bilateral cleft lip, Q13.0 Coloboma (iris) the gap is on both side of the lip. The default code for coloboma is for the ICD-10-CM classifies the condition by hard, anatomic site of iris. A coloboma may also soft, hard with soft, uvular and unspecified. occurs on the eyelid, fundus, lens and optic lens ICD-10-CM uses the terms bilateral, median, or and subterms for these sites appear under the unilateral. Cleft lip and palate in ICD-10-CM is main term of coloboma. classified according to hard versus soft palate with unilateral versus bilateral cleft lip. 3. Z38.00 Newborn (infant) (liveborn) (singleton), born in hospital 7. Q54.0 Hypospadias, coronal Q86.0 Syndrome, fetal, alcohol In ICD-10-CM codes are available for (dysmorphic) hypospadias balanic, penile, penoscrotal, According to ICD-10-CM coding guidelines, a perineal, congenital chordee, other hypospadias, code from Z38 is assigned as the principal/first and unspecified. Hypospadias refers to a listed diagnosis. When the coder reviews code congenital condition in which the urethral Q86.0, there is an Excludes2 statement that meatus lies in an abnormal location on the penis refers to a possible use of code P04.-. However, and may be located as far down as in the when code P04.3 (that with use of alcohol) is scrotum or perineum. referenced, it specifically excludes that with fetal alcohol syndrome. 8. Z38.01 Newborn (infant) (liveborn) (singleton), born in hospital, by cesarean 4. Fragile X Syndrome Q20.3 Transposition (congenital) vessels, Q99.2 Fragile, X chromosome; Syndrome, great (complete) (partial) fragile X In this case, the newborn code is listed first The Index entry for the main term of fragile because it is the admission for the birth. includes subterm for X chromosome. It is Transposition of the great vessels (TGV) is a classified to category X99, chromosome congenital heart defect in which the aorta and abnormalities, not elsewhere classified as it is a the pulmonary artery are transposed. Because genetic condition involving changes in the X this is a cyanotic heart defect (too little oxygen) chromosome that causes the most common form the cyanosis is inherent and not separately of inherited intellectual disability (mental coded. retardation) in males. 9. Cri-du-chat Syndrome 5. Q01.0 Encephalocele, frontal Q93.4 Syndrome, cri-du-chat The Index provides a straight-forward entry for term “patent” is the direction—see also the syndrome which is a deletion of the short Imperfect, closure. Under imperfect, closure the arm of chromosome 4. subterms of ductus and arteriosus appear also with code Q25.0 10. Polycystic kidney disease, autosomal recessive Q61.19 Polycystic (disease), kidney, 16. PROCEDURE: Open Blalock-Hanlon procedure autosomal recessive with excision of the atrial septal opening for Main terms could be polycystic, kidney or palliative treatment of transposition of great vessels disease, polycystic, kidney with subterm for Character Code Explanation autosomal recessive type which is assigned to Section 0 Medical and Surgical code of polycystic kidney, infantile type. Body 2 Heart and Great Vessels System 11. Duplicate ureter, left kidney Root B Excision Q62.5 Duplication , no entry for ureter— Operation see also Accessory, ureter Body Part 5 Atrial Septum Coding question is an example of following the Approach 0 Open directional notes in the Index. The main term of Device Z No Device accessory is the word used in ICD-10-CM for a Qualifier Z No Qualifier duplicate or double anatomic site. INDEX: Excision, septum, atrial 02B5; Blalock- 12. Tetralogy of Fallot congenital defect with Hanlon is an open procedure to excise the atrial ventricular septal defect, pulmonary stenosis, septum to create an opening in the atrial septum. dextroposition of aorta with hypertrophy of right ventricle 17. PROCEDURE: Laparoscopic Heller Q21.3 Tetralogy of Fallot which is described in the operative report of cutting Main term of “Tetralogy of Fallot” clearly into the muscle at the lower end of esophageal provides the code of Q21.3. In the Tabular, the sphincter to release it. inclusion terms listed below the title of the code Character Code Explanation Q21.3 are “ventricular septal defect with Section 0 Medical and Surgical pulmonary stenosis or atresia, dextroposition of Body D Gastrointestinal System aorta and hypertrophy of right ventricle. No System additional codes are required as the stated Root N Release conditions are the definition of Tetralogy of Operation Fallot Body Part 3 Esophagus, Lower

13. Hirschsprung’s congenital megacolon disease Approach 4 Percutaneous Endoscopic Q43.1 Hirschsprung’s disease or Device Z No Device megacolon Qualifier Z No Qualifier Main term in the Index can be Hirschsprung’s or megacolon, with subterms megacolon or INDEX: Release esophagus lower, 0DN3. Procedure Hirschsprung’s. No entry if found under the done to treat achalasia by cutting the muscles of the main term of “disease” for the condition. lower esophageal sphincter to release the constriction to allow food and liquids to reach the 14. Acoustic neurofibromatosis stomach. The main term “myotomy” leads the code Q85.02 Neurofibromatosis, acoustic to table “OK8” which is a division of muscle but Main term is neurofibromatosis with subterm of does not include the body part for the esophagus. acoustic. The main term acoustic states see This question is an example of the importance of condition, which for this example is the reviewing the operative report for the objective of neurofibromatosis. the procedure and not rely on the procedure title exclusively. 15. Patent ductus arteriosus Q25.0 Patent, ductus arteriosus The only main term that can be used for this condition is “patent.” The term ductus is followed by “see condition.” After the main 18. PROCEDURE: Reopening of chest wall to control bleeding after thoracic surgery, bleeding controlled and incision closed Character Code Explanation Section 0 Medical and Surgical Body W Anatomical Regions, System General Root 3 Control Operation Body Part 8 Chest wall Approach 0 Open Device Z No Device Qualifier Z No Qualifier

INDEX: Control postprocedural bleeding in, chest wall, 0W38.

19. PROCEDURE: Frenulotomy to treat ankyloglossia and speech dysfunction Character Code Explanation Section 0 Medical and Surgical Body C Mouth and Throat System Root N Release Operation Body Part 7 Tongue Approach X External Device Z No Device Qualifier Z No Qualifier

INDEX: Frenulotomy—see Release, mouth and throat, 0CN (tongue) Approach is external as the physician can reach inside the mouth to perform the procedure

20. PROCEDURE: Repair of cerebral artery aneurysm by restriction with bioactive intravascular coil Character Code Explanation Section 0 Medical and Surgical Body 3 Upper Arteries System Root V Restriction Operation Body Part G Intracranial artery Approach 3 Percutaneous Device B Bioactive Intraluminal Device Qualifier Z No Qualifier

INDEX: Restriction, artery, intracranial (cerebral) 03VG

Chapter 21 example is the high blood pressure readings. Symptoms, Signs and Abnormal 5. C56.1 Neoplasm, malignant primary, Clinical and Laboratory Findings, Not ovary, right Elsewhere Classified R18.0 Ascites, malignant There is a “code first” note appearing under Review Exercises code R18.0 to code first the malignancy, such as, malignant neoplasm of ovary. This 1. R10.821 Tenderness, rebound, right upper question is also an example of coding a quadrant symptom (ascites) that is not routinely ICD-10-CM provides subcategory R10.81 for associated with the underlying condition abdominal tenderness and subcategory R10.82 (malignant neoplasm of ovary). for rebound abdominal tenderness. 6. R19.7 Diarrhea 2. R40.2111 Coma, with opening of eyes (never) R50.9 Fever (with chills) R40.2211 Coma, with verbal response (none) R56.00 Seizure, febrile R40.2311 Coma, with motor response (none) The patient had two conditions when she R40.2134 Coma, with opening of eyes, in presented to the ER and then developed response to sound another symptom in the ER, all conditions are R40.2234 Coma, with verbal response, coded. No definitive diagnosis is documented inappropriate words so all symptoms are coded. R40.2344 Coma, with motor response, flexion withdrawal 7. R07.9 Pain, chest In order to report the scale, all three categories R68.84 Pain, jaw must be identified. The first set of codes R61 Sweating, excessive identified the condition as reported by the EMT. The diagnosis of “acute myocardial The second set of codes corresponds to the infarction” is not coded as it is qualified as a neurologist’s assessment on day 2. It is “rule out” condition In the outpatient setting, appropriate to report more than one set of codes only the conditions that are known for certain if desired. The seventh character for the first set are coded, for this question, those known of codes (1) identifies that this was done by the conditions are symptoms. EMT in the field, and the second set (4) 24 hours or more after hospital admission. This case is 8. R6.02 Shortness, breath used to illustrate the coma scale codes, but they R53.83 Fatigue would not be used alone. Two symptoms are coded separately as neither includes the other condition. 3. R92.0 Microcalcifications, breast ICD-10-CM has individual codes for 9. R97.2 Elevated, prostate specific antigen mammographic microcalcification found on (PSA) diagnostic imaging of the breast and Elevated PSA is an abnormal tumor marker. In mammographic calcification found on some patients there may be a known condition diagnostic imaging of breast. as well such as malignant neoplasm of prostate No conclusive diagnosis was documented, or non-malignant conditions such as prostatitis therefore the symptoms are coded. or enlarged prostate.

4. R03.0 High, blood pressure reading 10. R63.1 Polydipsia without diagnosis of hypertension R35.8 Polyuria The diagnosis of rule out hypertension is not At conclusion of physician office visit, doctor coded according to ICD-10-CM Diagnostic wrote the final diagnosis in the record as “Rule Coding and Reporting Guidelines for out diabetes.” Patient complained of polydipsia Outpatient Services (IV.H.) that specifies that and polyuria for several weeks. Again the “rule qualified diagnosis with such terms as rule out” diagnosis is not coded for an outpatient out, possible, probable or similar terms are visit. not coded. Instead, the condition that is known for certain is coded, which in this 11. R09.82 Postnasal drip R51 Headache 17. PROCEDURE: Control of epistaxis by R59.0 Lymphadenopathy, localized electrocautery At the conclusion of the physician office visit, Character Code Explanation the physician documented “Postnasal drip, Section 0 Medical and Surgical headache and localized lymphadenopathy, Body System 9 Ear, Nose, Throat possible seasonal allergies.” The possible Root Operation 5 Destruction allergies is not coded as it is qualified as Body Part K Nose possible. Approach X External Device Z No Device 12. R73.02 Elevated, glucose tolerance Qualifier Z No Qualifier The main term of “elevated” or “abnormal” can be used as well as the main term “findings, INDEX: Electrocautery—see Destruction, nose. abnormal, inconclusive, without diagnosis with Approach is external because the procedure can be the subterm of glucose (tolerance test). performed by reaching into the nose

13. R97.0 Elevated carcinoembryonic antigen 18. PROCEDURE: EGD with mid-esophageal (CEA) biopsy The main term “Elevated” with the subterm Character Code Explanation carcinoembryonic antigen (CEA) Section 0 Medical and Surgical

Body D Gastrointestinal System 14. R93.2 Nonvisualization, gallbladder. System The diagnosis of possible chronic cholecystitis is Root B Excision not coded as if it exists as this is an outpatient Operation visit. Another Index entry can be used to locate Body Part 2 Esophagus, Middle this code using the main term of “findings, abnormal, inconclusive, without diagnosis” with Approach 8 Via Natural or Artificial the subterm radiologic (x-ray) and biliary tract. Opening Endoscopic Device Z No Device 15. R91.1 Lesion, lung (coin) Qualifier X Diagnostic R05 Cough, chronic . The 2012 Draft version of ICD-10-CM has the INDEX: Excision, esophagus, middle 0DB2 The main term of lesion, lung (coin) has the code of qualifier of X is used to identify the procedure R91.2 listed which is an invalid code as it codes (biopsy) is a diagnostic procedure. not exist. Using the Tabular, the code of R91.1 is the correct code. 19. PROCEDURE: Maxillary sinusoscopy Character Code Explanation 16. PROCEDURE: Bronchoscopy with biopsy of Section 0 Medical and Surgical left main bronchus Body 9 Ear, Nose, Sinus Character Code Explanation System Section 0 Medical and Surgical Root J Inspection Body System B Respiratory System Operation Root B Excision Body Part Y Sinus Operation Approach 4 Percutaneous Endoscopic Body Part 7 Main Bronchus, Left Device Z No Device Approach 8 Via Natural or Artificial Qualifier Z No Qualifier Opening Endoscopic INDEX: Sinusoscopy 09JY4ZZ. No other procedure Device Z No Device is performed through the sinusoscope so the root Qualifier X Diagnostic operation is an inspection procedure.

INDEX: Excision, bronchus, main, left 0BB7 The qualifier of X is used to acknowledge the excision if a diagnostic procedure.

20. PROCEDURE: Cystoscopy with bladder biopsy Character Code Explanation Section 0 Medical and Surgical Body System T Urinary System Root B Excision Operation Body Part B Bladder Approach 8 Via Natural or Artificial Opening Endoscopic Device Z No Device Qualifier X Diagnostic INDEX: Excision, bladder 0TBB The qualifier of X is used to identify the bladder procedure as a biopsy, that is, a diagnostic procedure.

Chapter 22A second. Review the Tabular for correct seventh Injury character. Seventh character “S”, sequela, is used for

complications or conditions that arise as a direct Review Exercises result of an injury. When using seventh

character “S” it is necessary to use both the 1. Foreign body, cornea, right, initial encounter in injury code that precipitated the sequela and the Emergency Department (Do not assign the code for the sequela itself. The S is added only external cause code) to the injury code, not the sequela code. The T15.01xA Foreign body, cornea, right, specific type of sequela (paraplegia) is placeholder x for sixth character, initial sequenced first, followed by the injury code. encounter A for seventh character. The cornea is

part of the external eye. Any foreign body in a 5. S68.120D Amputation, traumatic, finger, penetrating wound of the orbit or eyeball would partial, index, right through the be coded to an open wound. metacarpophalangeal joint, subsequent visit

The seventh character “D” is used for the 2. S82.852K Nonunion, fracture—see Fracture, subsequent encounter for the visit for checking by site. Fracture, traumatic (abduction) of the healing progress for the traumatic (adduction) (separation), ankle, trimalleolar amputation. The fracture is an open fracture and (displaced). Review the Tabular for complete coded as displaced because when a fracture is code assignment as well as correct seventh not specified as displaced or nondisplaced it is character. coded as displaced. Aftercare Z codes should not be used for

aftercare of fractures. For aftercare of a fracture, 6. S67.22xA Crush, hand, left, initial encounter assign the acute fracture code with the correct S62.232B Fracture, metacarpal, first, base, seventh character indicating the type of open, initial encounter aftercare. Coding guidelines specify that if There is a “use additional code” for all displaced versus nondisplaced is not indicated, associated injuries, such as fracture of wrist or the default is displaced. hand (S62.-) that appears under category S67 for

Crushing injury or wrist, hand and fingers. The 3. S52.351B Fracture, traumatic (abduction) seventh character of B is used for the fracture (adduction) (separation), radius, shaft, code as it is an open fracture initial treatment. comminuted (displaced). Review the Tabular for

complete code assignment, including the seventh 7. S37.041A Laceration, kidney, minor (less than character. 1 cm), right, initial encounter A compound fracture is an open fracture and The main term is laceration with subterms for this is stated as a type II open fracture in the kidney and minor type as it is stated as small or documentation. The seventh character of B less than 1 cm in size. This is the initial indicates the initial treatment for a type II open encounter so the seventh character of A was fracture. The coder has to refer back to the list of used. There is no mention of an open wound so applicable seventh characters that appears none is coded. There is a note under category directly under category heading S52, Fracture of S37 to code also any associate open wound forearm. There are other lists of seventh (S31.-) but this is not necessary for this case. characters that are applicable to certain

subcategories or codes in category S52, for 8. S33.6xxA Sprain, sacroiliac, joint, initial example, a shorter list of seventh character encounter codes appears under code S52.01, S52.22, The main term is sprain, sacroiliac joint that S52.21, S52.31 and so on. gives code S33.6. Two placeholder “x”

characters are used to fill in the fifth and sixth 4. G82.21 Paraplegia (lower), complete positions before the seventh character of A can S32.029S Fracture, traumatic (abduction) be applied for the initial encounter. (adduction) (separation), vertebra, vertebral

(arch) (body) (column) (neural arch) (pedicle) 9. S61.211D Laceration, finger, index, left (spinous process) (transverse process), lumbar, S64.491D Injury, nerve, digital, finger, index, left Main term is laceration, finger, index finger, left 14. T24.299A, , lower limb, multiple sites side. Seventh character of D is used as this is a except ankle and foot, left, second degree, initial subsequent visit. The laceration to the nerve is encounter accessed in the Index under laceration, nerve – This case is an example of a less specified see Injury, nerve, finger – see Injury, nerve, diagnosis that is only described as first and digital, finger, index second degree of multiple sites on the leg or lower limb. The burn is described as two 10. S01.02xA Laceration, scalp, with foreign body. depths – first and second degree. According to Placeholder x used for 6th character. Seventh the coding guideline for burns of the same local character of A for initial encounter site, the second degree burn is coded as it is the The main term is laceration with subterms for highest degree. No code is assigned for the first the location of scalp and the fact a foreign body degree burn. is present. A placeholder of “x” is used to fill the sixth character position in order to add the 15 . S45.111A Injury, blood vessel, brachial, artery, seventh character of A for the initial encounter laceration, right, initial encounter of care. Category code S01 includes “code also” S41.121A Laceration, arm (upper) with foreign notes for any associated injury to cranial nerve, body, right, initial encounter muscle, tendon, or wound infection. None of According to coding guideline 19.b.2, when a these conditions were present in this case. primary injury results in minor damage to peripheral nerves or blood vessels, the primary 11. S43.012A Dislocation, shoulder, humerus, injury is sequenced first with additional code(s) anterior, left, initial encounter for injuries to nerves and spinal cord (such as This diagnosis is very specific to include the category S04), and/or injury to blood vessels type of dislocation being the anterior portion of (such as category S15). When the primary injury the left humerus. Other terminology for this is to the blood vessels or nerves, that injury condition may include dislocation of the should be sequenced first. In this scenario, the glenohumeral joint. The seventh character of A primary injury is to the brachial artery so that is applied as this is the initial encounter of care. injury sequenced first. Under category S45 is a “code also any associated open wound, S41.- 12. S83.421D Sprain, knee, collateral ligament, statement. Open wound or laceration code lateral, right, subsequent encounter sequenced in second position. This case is described as a subsequent encounter in the physician’s office for care of a knee 16. PROCEDURE: Closed reduction, fracture distal sprain. The type of sprain is specific to the radius, right with cast application lateral collateral ligament of the right knee. The Character Code Explanation main term used in the Index is sprain with Section 0 Medical and Surgical subterms of knee, collateral ligament, lateral Body P Upper Bones side. A “code also” any associated open wound System appears under the category heading of S83, Root S Reposition dislocation and sprain of joints and ligaments of Operation the knee does not apply to this case. Body Part H Radius, Right Approach X External (closed reduction) 13. T22.211A Burn, forearm, right, second degree, Device Z No Device initial encounter Qualifier Z No Qualifier This is an example of coding for a burn of a single site (forearm) with two different depths of INDEX: Reduction, fracture, see Reposition, radius, burn (first and second degree.) The coding right 0PSH guideline 19.d.2, burns of the same local site, to Application of a cast or splint in conjunction with classify the burn of the same local site but of the reposition procedure is not coded separately. The different degrees to the subcategory identifying main term is reposition, radius, right side. The code the high degree (second) recorded in the is built using table for 0PS with the body part for the diagnosis. The first degree burn is not coded for right radius with the approach for closed reduction the same site. as external. On this row for the body part and approach there is not option for device or qualifier so character “Z” is used for the sixth and seventh 19. PROCEDURE: Suture laceration repair, characters. laceration of forehead Character Code Explanation 17. PROCEDURE: Open reduction with internal Section 0 Medical and Surgical fixation, right femur shaft Body H Skin and Breast Character Code Explanation System Section 0 Medical and Surgical Root Q Repair Body Q Lower Bones Operation System Body Part 1 Skin, Face Root S Reposition Approach X External Operation Device Z No Device Body Part 8 Femoral Shaft, Right Qualifier Z No Qualifier Approach 0 Open Device 4 Internal Fixation Device INDEX: Suture laceration repair—see Repair, skin Qualifier Z No Qualifier for laceration, face, 0HQ1XZZ The coder must know to use the root operation of INDEX: Reduction, fracture, see Reposition, “repair” for the suture repair of a skin laceration. femoral shaft, left 0QS9 Under the main term repair, the subterm of skin, face The main term for the procedure is reposition, is use for the site of forehead specified in this femoral shaft, left. The code is built using table 0QS example. There is not a body part for the site of with the body part for femoral shaft, right side. The forehead so the value of “1” is used for skin of face. approach is specified as open. The device is only The approach is external as the repair can be described as an internal fixation device but not performed directly on the skin. There are no options specifically intramedullary, monoplanar, right or for a device or qualifier for skin repairs. hybrid so the sixth character of 4 is used. There is no option for a qualifier on this row. 20. PROCEDURE: Removal of foreign body, bullet from open wound, right neck muscle by incision 18. PROCEDURE: Application of left lower arm Character Code Explanation cast for nondisplaced fracture of ulna Section 0 Medical and Surgical Character Code Explanation Body K Muscles Section 2 Placement System Body W Anatomical Regions Root C Extirpation System Operation Root 3 Immobilization Body Part 2 Neck Muscle, Right Operation Approach 0 Open Body D Lower Arm Left Device Z No Device Region Qualifier Z No Qualifier Approach X External Device 2 Cast INDEX: Removal of foreign body see Extirpation, Qualifier Z No Qualifier muscle, neck, right 0KC2 The code must translate the removal of a foreign to INDEX: Casting see Immobilization, arm, lower, the root operation of extirpation as the root operation 2W3DX. Casting of a nondisplaced fracture is coded “removal” refers to the removal of a device from a to the root operation of immobilization in the body part, not a foreign body. The main term placement section. The main term is immobilization extirpation is used with the body part where the with the subterm for the location of ulna which is in foreign body is found—muscle of the right neck. the lower arm on the left side. Using the table for Table 0KC is used to construct the code. The body 2W3, the body part character is D for lower arm, part is “2” for right neck muscle. The approach is left. The only choice for the approach is external for open as described being performed by incision. the application of a cast. The device character is “2” There is not options for a device or qualifier on this for the cast that was applied. There is no option for table. the qualifier.

Chapter 22B , Z91.13-) or complication of care (Y63.6, Poisoning and Certain Other Y63.8–Y3.9) codes are to be used with an underdosing code to indicate intent, if known. Consequences of External Causes Codes for underdosing should never be assigned as principal or first-listed codes.” There is also a Review Exercises "code first underdosing of medication..." note under code Z91.13. The combination code for 1. T39.1x1A Poisoning (acute) —see also Table heart and kidney disease is used in this situation of Drugs and Chemicals, Acetaminophen, because both heart and renal disease exist along Poisoning, Accidental (unintentional). Review with the hypertension. According to the Official the Tabular for the correct seventh character. Coding Guidelines for hypertensive heart The seventh character is used with the poisoning disease, the causal relationship is implied with codes in ICD-10-CM. the word “hypertensive.” An additional code from category I50 is used to identify the type of 2. R11.2 Nausea, with vomiting heart failure. The “use additional code” R53.83 Fatigue statement under code I13.2 indicates the use of T46.0x5A Table of Drugs and Chemicals, the N18.5 code to identify the stage of the Digoxin, adverse effect, initial encounter chronic kidney disease. The Index directs the coder to T46.0x5 in the Tabular. The seventh character must be assigned 4. I49.5 Syndrome, sick, sinus to indicate the initial encounter. The Official T82.110A Complication(s) (from) (of), Coding Guidelines state that “a code for adverse cardiovascular device, graft, or implant, effect is assigned when the drug was correctly electronic, electrode, mechanical, breakdown. prescribed and properly administered.” Review the Tabular for assignment of seventh character. 3. I13.2 Disease, diseased, heart (organic), Z53.8 Canceled procedure (surgical), hypertensive—see Hypertension, heart. because of, specified reason NEC Hypertension, hypertensive (accelerated) The complication code, for the broken (benign) (essential) (idiopathic) (malignant) pacemaker electrode, is assigned as a secondary (systemic), heart (disease) with kidney disease diagnosis because the sick sinus syndrome was (chronic) —see Hypertension, cardiorenal the reason for admission. The Z code for the (disease), with heart failure, with stage 5 or end canceled procedure should also be added. stage renal disease I50.9 Failure, heart (acute) (sudden), 5. T84.020A Dislocation, prosthesis, congestive (compensated) (decompensated). The internal—see Complications, prosthetic device, "use additional code" statement under code I13.2 by site, mechanical. Complications, prosthetic indicates the use of this code to identify the type device, joint—see Complications, joint of heart failure. prosthesis, internal, dislocation N18.5 Disease, diseased, kidney The main term dislocation of a prosthesis refers (functional) (pelvis), chronic, stage 5. The "use the coder to the main term of complications, additional code" statement under code I13.2 prosthetic device. These types of mechanical indicates the use of this code to identify the complications are coded according to the type of stage of the chronic kidney disease complication, the specific type of device and T50.1x6A Refer to Table of Drugs and laterality for the side of the body affected. The Chemicals, Lasix, underdosing seventh character A is used as this was the initial Z91.130 Noncompliance, with, medication episode of care. regimen, underdosing, unintentional, due to patient’s age-related debility 6. T39.311A Table of Drugs and Chemicals, In ICD-10-CM, underdosing of medication can Naproxen, poisoning, accidental now be identified. The coding guidelines state: T51.0X1A Table of Drugs and Chemical, “Underdosing refers to taking less of a alcohol, beverage, poisoning, accidental medication than is prescribed by a provider or a R40.0 Drowsiness manufacturer’s instruction. For underdosing, According to coding guidelines and note assign the code from categories T36–T50 (fifth appearing under the block of codes T51–T65, or sixth character “6”). Noncompliance (Z91.12- when no intent is indicated, code to accidental for the poisoning. When two or more drugs or substance, poisoning, rattlesnake (venom), chemicals are reported, each is coded (naproxen accidental, initial episode and alcohol). The additional code for drowsiness S51.812A Laceration, forearm, left, initial is assigned for the manifestation of the encounter poisoning. 15. T76.12xD Abuse, child—see Maltreatment, 7. R42 Dizziness child, physical abuse, suspected, subsequent T44.7X5D Table of Drugs and Chemicals, encounter atenolol, adverse effect, subsequent visit S52.531D Fracture, Colles’ see Colles’ fracture, I10 Hypertension right, subsequent encounter for fracture with This is the patient’s second visit to address the routine healing dizziness that is a side effect or adverse effect of his medication and for the management of his 16. PROCEDURE: Open Thrombectomy right hypertension. The seventh character of D is used brachial artery with the adverse effect code to recognize it is a Character Code Explanation subsequent encounter of care for it. Section 0 Medical and Surgical Body 3 Upper Arteries 8. T86.23, Complication, heart, infection System I30.1 Pericarditis, viral Root C Extirpation B97.6 Infection, parvovirus, as cause of diseases Operation classified elsewhere B97.6 Body Part 7 Brachial Artery, Right Use additional code to specify infection appears Approach 0 Open after code T86.23. Use additional code (B95– Device Z No Device B97) to identify infectious agent Qualifier Z No Qualifier

9. H66.001 Otitis, suppurative, acute, right, INDEX: Thrombectomy—see Extirpation, artery, subsequent encounter, underdosing brachial right 03C7 T36.0x6A Table of drugs and chemicals, The thrombus is a solid or formed matter that is amoxicillin, underdosing, initial episode. being removed from the artery. The Index in ICD- Z91.128 Noncompliance with medication 10-PCS directs the coder to the root operation of regimen, underdosing, intentional extirpation when the title of the procedure thrombectomy is used. The subterm under 10. T48.0x1A Table of drugs and chemicals, extirpation is the location of artery, brachial. Using oxytocin, accidental, initial encounter table 03C, the fourth character for the body part of right brachial artery is 7. The approach is an open 11. R23.2, Flushing, thrombectomy. There is no option for a device or L29.9, Itching—see pruritus qualifier value other than Z on the table. T50.8x5A Table of drugs and chemicals, contrast medium for radiology, adverse effect, 17. PROCEDURE: Right kidney transplant from initial episode living donor Character Code Explanation 12. T82.868A Complication, catheter, dialysis Section 0 Medical and Surgical (vascular), thrombosis, initial encounter Body T Urinary System N18.6 Disease, end stage renal disease System (ESRD) Root Y Transplantation Z99.2 Status, renal dialysis Operation

Body Part 0 Kidney, Right 13. T42.4x2A Table of drugs and chemicals, valium, self harm, initial encounter Approach 0 Open T51.0x2A Table of drugs and chemicals, alcohol Device Z No Device beverage, self harm, initial encounter Qualifier 0 Allogeneic

14. T63.011A Venomous rattlesnake bite—see INDEX: Transplant, Kidney, Right 0TY00Z venom, bite, snake—see venom, venomous—see The root operation is transplantation, organ is right Table of Drugs and Chemicals, by animal or kidney and the Index refers the coder to the six character code of 0TY00Z. Using table 0TY, the catheter and there is no option for a qualifier so seventh character for the qualifier that must be value Z is used. selected is the type of organ transplanted— allogeneic, syngeneic or zooplastic. Because the 20. PROCEDURE: Angioplasty of left renal artery patient is receiving a kidney from a living donor with insertion of a vascular stent (another human) the qualifier is 0 for allogeneic Character Code Explanation Section 0 Medical and Surgical 18. PROCEDURE: Hemodialysis single episode Body 4 Lower Arteries Character Code Explanation System Section 5 Extracorporeal Assistance Root 7 Dilation and Performance Operation Body Sys A Physiological Systems Body Part A Renal Artery, Left Root 1 Performance Approach 3 Percutaneous Operation Device D Intraluminal Device Body D Urinary Qualifier Z No Qualifier System INDEX: Angioplasty—see Dilation, Lower Arteries Duration 0 Single 047 Function 0 Filtration The purpose of an angioplasty is to dilate a vessel. Qualifier Z No Qualifier The main term of angioplasty refers the coder to the root operation of dilation. The location of the INDEX: Dialysis, hemodialysis 5A1D00Z angioplasty is the left renal artery which is a lower The main term in the Index, dialysis, hemodialysis artery. The coder is referred to table 047. The body refers the coder to the complete seven character code part is value A for renal artery, left. The approach is 5A1D00Z. Referring to the table 5A1 in the typically percutaneous for an angioplasty not Extracorporeal Assistance and Performance section otherwise stated to be open. The device value is D of ICD-10-PCS, the coder confirms the code with for the stent which is an intraluminal device. There the fourth character of D for urinary system that is no option for a qualifier so value Z is used. would include kidney dialysis, fifth character for the duration being a single episode or 0, the only choice for the sixth character is filtration which is the function of dialysis and there is no option for the qualifier.

19. PROCEDURE: Percutaneous removal of PICC venous catheter from upper arm Character Code Explanation Section 0 Medical and Surgical Body 5 Upper Veins System Root P Removal Operation Body Part Y Upper Vein Approach 3 Percutaneous Device 3 Infusion Device Qualifier Z No Qualifier

INDEX: Removal of device from, vein, upper 05PY The root operation is removal when a device is removed from the body. Index entry is removal of device from, vein, upper (arm). When the coder refers to the table 05P, the remaining choices are the approach being percutaneous, the device identified as infusion which is the purpose of a PICC venous Chapter 23 3. W54.0XXA Index to External Causes. External Causes of Morbidity Bite, bitten by, dog Y92.71 Index to External Causes. Place of

occurrence, barn Review Exercises Y93.K9 Index to External Causes. Activity

(involving) (of victim at time of event), animal 1. V43.53XA Index to External Causes. Accident, care NEC car – see Accident, transport, car occupant. Y99.0 Index to External Causes. External Accident, transport, car occupant, driver, cause status, civilian activity done for income or collision (with) pickup truck (traffic) pay Y92.411 Index to External Causes. Place of

occurrence, highway (interstate) In this case it is possible to report the place of Y93.C2 Index to External Causes, Activity occurrence, the activity and status in addition to (involving) (of victim at time of event), cellular, the external cause code for bite. When adding telephone the seventh character if the code does not

contain six characters, the “x” is used before The transport accident codes have been greatly placing the seventh character. expanded in ICD-10-CM with much more detail.

It takes experience to get used to the External 4. W39.XXXA Index to External Causes, Causes Index and Tabular sections. Just getting fireworks familiar with both is a help to coding these Y92.830 Index to External Causes, place of conditions correctly. An appropriate seventh occurrence, park (public) character is to be added to each code from Y99.8 Index to External Causes, status of category V43. If the code does not contain six external cause, student activity characters, the “x” is used before placing the

seventh character. No Status code was selected 5. W03.XXXA Index to External Causes, because this information was not documented. tackle in sports

Y92.321 Index to External Causes, place of 2. Y37.230A Index to External Causes. Military occurrence, football field operations (injuries to military and civilians Y93.61 Index to External Causes, activity, occurring during peacetime on military property football (American) tackle and during routine military exercises and Y99.8 Index to External Causes, status of operations) (by) (from) (involving) explosion external cause, student activity (of) improvised explosive device [IED] (person-

borne) (roadside) (vehicle-borne) 6. X10.1XXA Index to External Causes, burned, Y92.139 Index to External Causes. Place of hot, food occurrence, military base—see Place of Y92.511 Index to External Causes, place of occurrence, residence, institutional, military base occurrence, restaurant Y99.1 Index to External Causes. External Y99.0 Index to External Causes, status of cause status, military activity external cause, civilian activity done for

financial or other compensation There is no activity code assigned here because

none of the categories is specific to this case. 7. W20.8XXA Index to External Causes, Even though Y93.89 (other activity) is available, falling, tree it is not assigned in this case because of this Y92.821 Index to External Causes, place of note: “They are also appropriate for use with occurrence, forest external cause codes for cause and intent if Y93.01 Index to External Causes, activity, identifying the activity provides additional hiking information on the event.” In this case, there is Y99.8 Index to External Causes, status of no kind of activity involved. The fact that the external cause, recreation person was military personnel injured by an IED

is not an activity—it is captured by the Y37 8. V80.010A Index to External Causes, accident, code. transport, animal-rider, noncollision, specified as

horse rider Y92.39 Index to External Causes, place of shooting—see also discharge, firearm, by occurrence, riding school type (handgun), homicide Y93.52 Index to External Causes, activity, horseback riding 17. Y02.0XXD Index to External Causes, Y99.8 Index to External Causes, status of Assault, pushing, before moving object, external cause, student activity motor vehicle

9. V91.37XA Index to External Causes, accident, 18. W85.XXXD Exposure, electric current, transport, watercraft, hit by falling object, transmission lines unpowered craft, water skis Y92.828 Index to External Causes, place of 19. W89.1XXS Radiation, ultraviolet , tanning occurrence, lake bed Y93.17 Index to External Causes, activity, water, skiing 20. W06.XXXA Fall, from (off)(out of ), bed Y99.8 Index to External Causes, status of Y92.032 Place of occurrence, residence, external cause, recreation apartment, bedroom Y93.84 Activity, sleeping 10. W21.03XS Index to External Causes, struck by, Y99.8 External cause status, student ball, baseball

11. W17.2XXA Index to External Causes, fall, into, hole Y92.017 Index to External Causes, place of occurrence, residence, house, single, yard Y93.H2 Index to External Causes, activity, gardening Y99.8 Index to External Causes, status of external cause, recreation

12. W42.9XXS Index to External Causes, noise

13. Y65.53 Index to External Causes, wrong, procedure Y92.530 Index to External Causes, place of occurrence, ambulatory surgery center

14. V03.90XA Index to External Causes, accident, transport, pedestrian, on foot, collision, car Y92.481 Index to External Causes, place of occurrence, parking lot Y93.01 Index to External Causes, activity, walking Y99.8 Index to External Causes, status of external cause, specified

15. X99.1XXA Index to External Causes, assault, stab—see cutting or piercing, knife Y92.310 Index to External Causes, place of occurrence, basketball court Y93.67 Index to External Causes, activity, basketball Y99.8 Index to External Causes, status of external cause, recreation

16. X93.XXXD Index to External Causes, Chapter 24 Main term of counseling for alcohol abuser is Factors Influencing Health Status and correct Index entry for alcohol dependence as well. Use additional code for alcohol abuse Contact with Health Services and dependence is found under code Z71.41 to make sure both facts are identified with ICD-10-CM Review Exercises codes.

1. Z38.00 Newborn (infant) (liveborn) 5. Z85.118 History, personal, malignant (singleton) born in hospital neoplasm, lung P55.0 Incompatibility, Rh (blood group) Z87.891 History, personal, tobacco (factor), newborn dependence Z67.10 Blood, type, A (Rh positive) The main term of history is used in the The newborn code would be listed first, Alphabetic Index to code the previous lung followed by the Rh incompatibility. The blood carcinoma. A use additional code note type of the baby is A+. The mother’s blood type appears under category Z85 reminds the is not coded on the newborn’s record. coder to use additional code to identify the history of tobacco use as documented. 2. Z02.0 Examination (for) (following) (general) (of) (routine), medical (adult) (for) (of) 6. Z95.1 Status (post), pacemaker, cardiac preschool children, for admission to school Z95.0 Status (post), aortocoronary ICD-10-CM provides specificity for the type of bypass administrative examinations performed. Two postprocedural statuses are identified in this patient. The main term of status (post) Is 3. Z44.121 Encounter (with health service) (for) used to find two entries for pacemaker and fitting (of)—see Fitting (and adjustment) (of). aortocoronary bypass Fitting (and adjustment) (of) artificial, leg—see Admission, adjustment, artificial, leg. 7. Z12.11 Screening, neoplasm (malignant) Admission (for), adjustment (of), artificial, leg, (of), colon partial Z 80.0 History, family (of), malignant Z89.511 Absence (of) (organ or part) neoplasm (of), (colon) (complete or partial) leg (acquired) (above Note under the category Z12, Encounter for knee), below knee (acquired) screening for malignant neoplasm states Category Z44 is used for fitting and adjustment “Screening is the testing for disease or disease of external prosthetic devices, including the precursors in asymptomatic individuals so that removal or replacement of external prosthetic early detection and treatment can be provided devices. This category is not used for for those who test positive for the disease. An malfunction or other complications of the “use additional code to identify any family device. In this case, the acquired absence of the history of malignant neoplasm(Z80.-)” limb was added as an additional code. See instruction is included under category Z12. coding guideline I.C.21.7, which references that a status code should not be used when the 8. Z51.11 Encounter (with health service) aftercare code indicates the type of status, such (for), chemotherapy for neoplasm as using Z43.0, Encounter for attention to C25.1 Neoplasm, pancreas, body, tracheostomy, with Z93.0, Tracheostomy status. malignant primary This is the same type of situation, but the A note appears under category Z51, Encounter aftercare code indicates that the artificial leg is for other aftercare, states code also condition partial, but not specifically where the amputation requiring care. Another note appears under occurred. The status code can provide greater category C25, Malignant neoplasm of pancreas specificity about the site, for example, foot, to “use additional code to identify: alcohol abuse ankle, below knee, above knee. In this case. it and dependence.” There was no mention of was felt that the additional code provided alcohol abuse or dependence in this patient additional information. therefore no code was required.

4. Z71.41 Counseling, alcohol abuser 9. Z33.2 Abortion, induced (encounter for) F10.20 Dependence, alcohol An Excludes1 note appears under code Z33.2 14. Z01.419 Examination (for) cervical that identifies conditions that cannot be code Papanicolaou smear, as part of routine with Z33.2, Encounter for elective termination gynecological examination of pregnancy. These excluded conditions are (1) Instructions appear under Z01.41, Encounter for early fetal death with retention of dead fetus routine gynecological examination to use (O02.1), (2) late fetal death (O36.4) and (3) additional code: spontaneous abortion (O03) (1) for screening for human papillomavirus, if applicable (Z11.51) 10. Z38.62 Newborn, triplet, born in hospital, (2) for screening vaginal pap smear, if by cesarean applicable (Z12.72) P07.17 Low, birthweight, with weight of (3) to identify acquire absence of uterus, if 1769 grams applicable (Z90.71-) P07.38 Preterm, newborn, gestational age, An Excludes1 note also appears to state what is 35 completed weeks not coded with Z01.41 codes, that is, The note that appears under category Z38, gynecological examination status-post Liveborn infants according to place of birth and hysterectomy for malignant condition (Z08) and type of delivery, states this category is for use as screening cervical pap smear not a part of a the principal code on the initial record of a routine gynecological examination (Z12.4) newborn baby. It is to be used for the initial birth record only. It is not to be used on the mother’s 15. Z08 Examination, following, treatment record. (for), chemotherapy, malignant neoplasm Z85.3 History, personal (of), malignant 11. Z20.2 Exposure (to) gonorrhea or neoplasm (of), breast Exposure (to) sexually-transmitted disease Z92.21 History, personal (of), The main term of exposure (to) is used in the chemotherapy for neoplastic condition Alphabetic Index to locate the condition of the The purpose of the visit is to perform a follow- patient being exposed to or having contact with up examination on a patient who has completed a partner who was known to have gonorrhea, a chemotherapy for . Main term in sexually transmitted disease the Alphabetic Index is an follow-up examination for the chemotherapy treatment. 12. Z30.2 Encounter (for), sterilization Under code Z08, Encounter for follow-up Z87.59 History, personal (of) obstetric examination is a note to use additional codes for complications the personal history malignant neoplasm of the Two reasons for the encounter are identified in breast. In addition, the coder must recognize the this case. First the patient seeks permanent need for a code for the personal history of sterilization so the main reason for the encounter chemotherapy for the neoplastic condition the is for sterilization. The underlying reason for the patient had. sterilization is the fact that the patient had complications during her previous pregnancies. 16. Z04.2 Examination, following, accident at Main term of history, personal, obstetric work complications is the approach to find the The category is used to describe an encounter underlying reason for the sterilization in the when a patient is brought to the health care ICD-10-CM codes. setting for examination following an accident at work but does not have signs or symptoms of an 13. Z52.3 Donor, bone, marrow injury or condition. Includes note under category Z52, Donors of Organs and Tissues, states the codes include 17. Z18.81 Retained, foreign body fragments, autologous and other living donors. The glass Excludes1 instructs the coder that cadaveric This category, Z18, Retained foreign body donor—omit code so that none is assigned and fragments, is used when a patient is examined examination of potential donor is classified with when an embedded fragment or splinter of a Z00.5 foreign body such as metal, glass, plastic, organic or other foreign matter is found in the patient’s body.

18. Z28.3 Delinquent immunization status This code is used when a patient is examined for an under immunization status or what also might be described as delinquent immunization status or lapsed immunization schedule status.

19. Z63.0 Problems, marital The code Z63.0, Problems in relationship with spouse or partner is used to identify the reason for a health care encounter. 20. Z34.83 Prenatal care, normal pregnancy, see Pregnancy, normal, specified (this was not her first pregnancy The Tabular List must be reviewed in order to find the correct code for the supervision of the pregnancy that occurred in the third trimester of pregnancy. Chapter 25 hospital’s net revenue from state and other Coding and Reimbursement local government sources for indigent care. 9. Medicare designates a hospital as a sole

community hospital if it is: Review Exercises  located at least 35 miles from other

similar acute care IPPS hospital; 1. Goal is to significantly improve Medicare's ability to recognize severity of illness in its  located in a rural setting located Inpatient hospital payments. The new system is between 25 and 35 miles for another projected to increase payments to hospitals for similar hospital and must meet one other services provided to sicker patients and decrease criteria related to the admission patterns payments for treating less severely ill patients. of the community residents; and must meet one other criteria related to the 2. Hospital payment = MSDRG relative weight admission patterns of the community multiplied by the hospital base rate residents.  located in a rural setting that 3. Additional payments may be made to (1) experiences severe weather conditions disproportionate share hospitals, (2) for Indirect that makes travel to similar hospitals medical education, (3) for new technologies and inaccessible for at least 30 days in two (4) for an outlier case. out of three years; or  located in a rural setting and because of 4. Principal and secondary surgical procedure distances, roads or weather conditions (codes) that requires a travel time of at least 45 minutes between like hospital. 5. The QIO’s programs are designed to: 10. Post acute care settings are health care  Review beneficiary complaints as well as settings where patients receive services after serving as an advocate for beneficiaries and discharge from hospitals, for example, in their families through quality improvement long-term care hospitals, rehabilitation or activities psychiatric hospitals, units in acute care  Use evidence-based performance hospitals, skilled nursing facilities, home improvement tools to promote health care health agencies, cancer hospitals, or services children's hospitals.

 Work with nursing homes to reduce the occurrence of pressure ulcers  Work with hospitals to reduce central line catheter bloodstream infections  Promote the use of electronic health records for care management  Increase preventive services like flu and pneumococcal immunizations as well as colorectal and breast cancer screenings  Help reduce readmissions to hospitals for Medicare beneficiaries by promoting community based services to provide follow up care for the hospitals. 6. Recovery audit contractors (RACs) 7. By reviewing all the ICD-10-CM diagnosis codes assigned to explain the reasons the services were provided. 8. A hospital qualifies for a disproportionate share hospital adjustment if the hospital treats a high percentage of low-income patients and if a hospital is located in an urban setting with more than 100 beds and receives more than 30 percent of the Coding Self-Test (Descending Colon = M), approach (open=0) Device (none = Z) and Qualifier (Cutaneous = 4) 1. Complete elective abortion, first trimester, 8 weeks, due to maternal rubella, with suspected 4. Paranoid schizophrenia damage to fetus affecting management of F20.0 Schizophrenia, paranoid pregnancy; abortion by laminaria Z33.2 Abortion, induced (encounter for) 5. Obstructive hydrocephalus; cerebral ventricle to O35.3XX0 Rubella, maternal, suspected atrium shunt using synthetic substitute by open damaged to fetus affecting management of approach pregnancy. G91.1 Hydrocephalus, obstructive Note: the seventh character of "0" is used for a 00160J2 Shunt creation, see Bypass, cerebral single gestation as this case does not describe it ventricles (to atrium) to be a twin or other multiple gestation. Because seventh character is required for code O35.3 two PROCEDURE: Cerebral ventriculoatrial shunt using placeholder XX characters are used to complete synthetic substitute by open approach the code before the seventh character Is added. Character Code Explanation Z3A.08 Pregnancy, weeks of gestation, 8 weeks Section 0 Medical and Surgical 10A07ZW Abortion, Laminaria Body 0 Cerebral Nervous System System 2. Postpartum abscess of breast; patient discharged Root 1 Bypass 5 days ago following spontaneous delivery of Operation live triplets Body Part 6 Cerebral ventricle O91.22 Abscess, breast, puerperal, postpartum, Approach 0 Open gestational—see Mastitis, obstetric, purulent, Device J Synthetic substitute associated with puerperium Qualifier 2 Atrium

3. Adenocarcinoma of descending colon with INDEX: Shunt creation, See Bypass, cerebral extension to mesenteric lymph nodes; permanent ventricles (to atrium) Bypass, Cerebral ventricles = descending colon colostomy colostomy, open 001, Code is constructed based body part (Cerebral procedure with colostomy brought to the skin ventricle =6), approach (open=0) Device (Synthetic level substitute= J) and Qualifier (Atrium=2 . ICD-10- C18.6 Adenocarcinoma - see Neoplasm, PCS Guideline B3.6a: Bypass procedures are coded malignant, by site. Go to Table of Neoplasms, by identifying the body part bypassed "from" and the intestine, large, colon, descending, malignant, body part bypassed "to". The fourth character body primary part specified the body part bypassed from, and the C77.2 Table of Neoplasms, lymph, gland, qualifier specifies the body part bypassed to. mesenteric, malignant, secondary 6. Parkinsonism secondary to haloperidol 3. PROCEDURE: Permanent descending colon neuroleptic drug therapy, initial encounter; drug was colostomy, open, colostomy brought to the skin discontinued (cutaneous) level G21,11 Parkinsonism, due to, drugs, neuroleptic Character Code Explanation T43.4X5A Table of Drugs and Chemicals, Section 0 Medical and Surgical Haloperidol, Adverse Effect, Initial encounter. Body D Gastrointestinal System Seventh character of "A" for initial encounter System Root 1 Bypass 7. Gangrene of lower leg due to uncontrolled Operation type I diabetes Body Part M Descending colon E10.52 Diabetes, type 1, with, gangrene Approach 0 Open E10.65 Diabetes, out of control - code to Diabetes, Device Z No Device by type, with hyperglycemia: Diabetes, type 1, with Qualifier A Cutaneous hyperglycemia. Note: Diagnosis of diabetes uncontrolled should be coded as diabetes out of INDEX: Colostomy, see Bypass, Gastrointestinal control or poorly controlled or inadequately System = 0D1, Code is constructed based body part controlled. . 8. Newborn twin, male, delivered by cesarean I67.2 Arteriosclerosis, cerebral delivery (in hospital) with syndrome of infant of F09 Syndrome, organic, brain. Note: There is a diabetic mother "code first" note under category F09: Code first Z38.31 Newborn, twin, born in hospital, by the underlying physiological condition. In this cesarean case, the underlying condition is the cerebral P70.1 Syndrome, infant, of diabetic mother arteriosclerosis.

9. History of allergic reaction to penicillin 20. Fracture of frontal bone with subarachnoid Z88.0 History, personal, allergy, penicillin hemorrhage and concussion with no loss of consciousness due to motor vehicle accident 10. Chronic kidney disease, ESRD, dependence on collision with another car (patient driver of car) renal dialysis. ; hemodialysis single session initial encounter N18.6 Disease, end stage renal (ESRD) S02.0xxA Fracture, frontal (bone) Z99.2 Dependence, on, renal dialysis S06.5x0A Hemorrhage, subdural—see 5A1D00Z Hemodialysis Hemorrhage, intracranial, subdural, traumatic— see Injury, intracranial, subdural, hemorrhage, 11. COPD with asthma traumatic J44.9 Disease, lung, obstructive, with asthma Note: A separate code for the concussion is not See the instructional "Code Also" note under assigned. See the Excludes1 note at subcategory category J44: Code also type of asthma if S06.0, Concussion. Note states concussion with applicable (J45.-) other intracranial injuries classfied to category J45.909 Asthma S06—code to specified intracranial injury V43.52xA Index to External Causes, Accident, 12. Unstable angina motor vehicle, see also Accident, transport, car I20.0 Angina, unstable occupant, driver, collision (with) car

13. Unexplained dizziness 21. Infiltrative tuberculosis of both lungs R42 Dizziness A15.0 Tuberculosis, lungs—see Tuberculosis, pulmonary 14. Hypertensive heart and kidney disease with chronic kidney disease, stage 3 22. Ovarian retention cyst; laparoscopic partial I13.10 Hypertensive, heart, with, kidney oophorectomy, left side disease—see hypertension, cardiorenal, without N83.29 Cyst, retention (ovary) or Cyst, ovary, heart failure, with stage 1 through stage 4 retention chronic kidney disease 0UB14ZZ Oophorectomy, see Excision, Female N18.3 Disease, kidney, chronic, stage 3 Reproductive System, ovary, left

15. Iron deficiency anemia due to chronic blood loss PROCEDURE: Laparoscopic partial oophorectomy, D50.0 Anemia, iron deficiency, secondary left side to blood loss (chronic) Character Code Explanation Section 0 Medical and Surgical 16. Cystic pancreatitis Body U Gastrointestinal System K86.1 Pancreatitis, cystic System Root B Excision 17. Reye’s syndrome Operation G93.7 Syndrome, Reye's Body Part 1 Ovary, left Approach 4 Percutaneous endoscopic 18. Third-degree burn of chest and second-degree Device Z No device burn of right leg, Initial encounter Qualifier Z No qualifier T21.31xA Burn, chest, third degree T24.201A Burn, leg, see Burn, lower limb, right, INDEX: Oophorectomy—see Excision or Resection, second degree Female Reproductive System. Because this was a partial oophorectomy (the entire ovary was not 19. Organic brain syndrome due to cerebral removed) this is an "excision" root operation arteriosclerosis procedure. Excision, ovary, left = 0UB1 Code is chair constructed based body part (Ovary, left =1), Y92.010 Index to External Causes, Place of approach (laparoscopic=4 for percutaneous occurrence, residence, house, single family, endoscopic) Device (none = Z) and Qualifier kitchen (none=Z) Y93.G3 Index to External Causes, Activity, cooking 23. Lyme disease with associated arthritis Y99,8 Status of external cause, specified A69.23 Arthritis, due to or associated with, (retirement) Lyme Disease Note: See Coding Guideline 1.C.20.d Place of occurrence, activity, and status codes used with 24. Abnormal prothrombin time, cause to be other external cause code. When applicable, determined place of occurrent, activity, and external cause R79.1 Abnormal, prothrombin time status codes are sequenced after the main external cause code(s). Regardless of the number 25. Newborn born in community hospital transferred of external cause codes assigned, there should be to university medical center. Code for the infant only one place of occurrence code, one activity at the university medical center treated for code, and one external cause status code hypoplastic left heart syndrome. assigned to an encounter. Q23.4 Syndrome, hypoplastic left heart 0PSCXZZ Reduction, fracture, see Reposition, Note: See coding guideline 1.C.16.a.2: Principal humeral head, right diagnosis for birth record. When coding the birth episode In a newborn record, assign a code from PROCEDURE: Closed reduction, humeral head with category Z38, Liveborn infants according to immobilization place of birth and type of delivery, as the Character Code Explanation principal diagnosis. A code from category Z38 Section 0 Medical and Surgical Is assigned only once, to a newborn at the time Body P Upper Bones of birth. If a newborn is transferred to another System institution, a code from category Z38 should not Root S Reposition be used at the receiving hospital. Operation Body Part C Humeral Head, Right 26. Ingestion of 30 doxepin (Sinequan) tablets Approach X External resulting in an overdose, determined to be a Device Z No Device suicide attempt; tachycardia [Doxepin is a Qualifier Z No Qualifier tricyclic antidepressant drug] Initial episode of care INDEX: Reduction, fracture—see Reposition, T43.012A Table of Drugs and Chemicals, humeral head, right 0PSC. Code is constructed based Sinequan, Poisoning, Intentional Self Harm, body part (Humeral head, right = C), approach initial episode (closed = external=X) Device (none = Z, there is no See the instructional note under section T36– option for a device when the approach is external) T50, Poisoning, adverse effects of and and Qualifier (none=Z) underdosing of drugs, medicaments and biological substances. "Use additional code" to 28. Inflamed of right face ; specify manifestations of poisoning. cryotherapy of lesion on right temple R00.0 Tachycardia L82.0 Keratosis, seborrheic, inflamed 0H51XZ Z Cryotherapy—see Destruction, skin, 27. Fracture, right shoulder, humerus upper end face (head), as the result of a fall from a chair she was standing on to reach a high shelf, occurred at her single family residence , kitchen while cooking; closed reduction, , humeral head, with immobilization, initial episode of care. Patient is retired. S42.201A Fracture, traumatic, humerus, upper end (right) W07.xxxA Index to External Causes, Fall, from PROCEDURE: Cryotherapy of lesion on right 32. Diabetic hypoglycemic coma in a patient with temple uncontrolled type 1 diabetes Character Code Explanation E10.641 Diabetes, type 1, hypoglycemic, with Section 0 Medical and Surgical coma Body H Skin and Breast E10.65, Diabetes, type 1, out of control System (uncontrolled) —see Diabetes, by type (1), with Root 5 Destruction hyperglycemia Operation Body Part 1 Skin, face 33. Secondary thrombocytopenia due to Approach X External hypersplenism; total splenectomy, open Device Z No Device D69.59 Thrombocytopenia, secondary Qualifier Z No Qualifier D73.1 Hypersplenism Note: The sequencing of the diagnoses depends INDEX: Cryotherapy—see Destruction, skin, face, on the circumstances of the admission. 0H51XZ. A keratosis lesion on the temple would be However, the secondary thrombocytopenia was a skin lesion. Code is constructed based body part chosen to be principal in this case as it was the (Skin, face = 1), approach (external = X) Device condition that was being treated by the surgery; (none = Z) and Qualifier (none = Z) This is a it is possible that a splenectomy would not destruction root operation for a cryotherapy necessarily be done simply for hypersplenism but each case should be reviewed independently 29. Moderate mental retardation as the sequela of with the physician to determine the reason for acute bacterial meningitis 10 years ago admission after study. F71 Retardation, mental—see Disability, 07TP0ZZ Splenectomy, see Resection, intellectual, moderate Lymphatic and Hemic System, spleen See "Code first" note under section F70–F79, Intellectual Disabilities: Code first any PROCEDURE: Total splenectomy (open) associated physical or developmental disorders. Character Code Explanation But this instruction is over-ridden by the "code Section 0 Medical and Surgical first" note under category G09, Sequela, Body 7 Lymphatic and Hemic meningitis, bacterial. This patient's mental System Systems retardation is the sequela of a previous acute Root T Resection bacterial meningitis that occurred ten years ago. Operation The G09 category code cannot be used first Body Part P Spleen because of the note below it: the condition that Approach 0 Open resulted from the meningitis of ten years ago has Device Z No device to be coded first, therefore, the F71 code for the Qualifier Z No qualifier condition the patient has today is coded first. G09 Sequela, meningitis, bacterial INDEX: Splenectomy—see Excision or Resection, See "Code first" note under category G09, Lymphatic and Hemic System. Because the Sequelae of inflammatory diseases of central procedure was described as "total" splenectomy nervous system: Code first condition resulting which means the entire spleen was removed, the root from (sequela) of inflammatory diseases of operation Is a resection. Code is constructed based central nervous system. body part (Spleen = P), approach (open = 0) Device (none = Z) and Qualifier (none = Z) 30. Chlamydial vaginitis A56.02 Vaginitis, chlamydial 34. Pneumonia due to Staphylococcus aureus; fiberoptic bronchoscopy, tracheobronchial tree 31. Infiltrating duct breast carcinoma, , right upper J15.211 Pneumonia, in (due to) Staphylococcus, outer quadrant, with metastases to bone (female aureus patient) 0BJ08ZZ Bronchoscopy C50.411 Neoplasm, breast, upper outer quadrant, malignant, primary, female, right 35. Peptic ulcer of the lesser curvature of the breast stomach, acute, with hemorrhage; C79.51 Neoplasm, bone, malignant, secondary esophagogastroduodenoscopy (EGD) with closed biopsy of stomach 37. Coronary artery disease with previous K25.0 Ulcer, stomach (peptic), acute, with, autologous vein bypass grafts in the left anterior hemorrhage descending, left circumflex, and right posterior The site of stomach is used to code the ulcer descending arteries. Procedure performed are condition. The code for "peptic" ulcer, K27 is coronary artery bypass grafts with double (left for peptic ulder, site unspecified so the site of and right) internal mammary bypass to the left the ulcer is more important for coding purposes anterior descending and the left circumflex and a than the type of ulcer being peptic single aortocoronary bypass to the right 0DB68ZX Biopsy, see Excision, stomach with posterior descending artery using saphenous qualifier of diagnostic vein graft with cardiopulmonary bypass I25.810 Disease, coronary artery—see Disease, PROCEDURE: Esophagogastroduodenoscopy with heart ischemic, atherosclerotic, coronary artery closed biopsy of stomach bypass graft, see Arteriosclerosis, coronary Character Code Explanation (artery), bypass graft, autologous vein Section 0 Medical and Surgical 021009W Bypass, artery, coronary, one site Body D Gastrointestinal System (single aortocoronary bypass) System Root B Excision PROCEDURE: Coronary artery bypass graft, single Operation aortocoronary bypass to the right posterior Body Part 6 Stomach descending artery using saphenous vein graft, open Approach 8 Via Natural or Artificial approach Opening Endoscopic Character Code Explanation Device Z No Device Section 0 Medical and Surgical Qualifier X Diagnostic Body 2 Heart and Great Vessels System INDEX: Biopsy—see Excision, Stomach 0DB6, Root 1 Bypass Code is constructed based body part (Stomach = 6), Operation approach (EGD=via natural or artificial opening Body Part 0 Coronary Artery, One Site endoscopic = 8) Device (none = Z) and Qualifier Approach 0 Open (Diagnostic=X) Device 9 Autologous Venous Tissue Qualifier W Aorta 36. Rapidly progressive glomerulonephritis; percutaneous renal biopsy, right kidney INDEX: Bypass, artery, coronary, one site 0210 N01.9 Glomerulonephritis, rapidly progressive Code is constructed based body part (Coronary 0TB03ZX Biopsy, see Excision kidney with artery, one site = 0), approach (open=0) Device qualifier of diagnostic (saphenous veing graft = autologous venouse tissue =9) and Qualifier (Aorta=W, the saphenous vein is PROCEDURE: Percutaneous renal biopsy, right sewn from the aorta to the coronary artery specified, kidney in this case the right posterior descending artery) Character Code Explanation See Coding Guideline B3.6b for Coronary arteries Section 0 Medical and Surgical are classified by number of distinct sites treated, Body T Urinary System rather than number of coronary arteries or anatomic System name of a coronary artery. Coronary artery bypass Root B Excision procedures are coded differently than other typass Operation procedures as described in the previous guidelines. Body Part 0 Kidney, Right Rather than identifying the body part bypassed from, Approach 3 Percutaneous the body part identifies the number of coronary Device Z No Device artery sites bypassed to, and the qualifier specifies Qualifier X Diagnostic the vessel bypassed from. 02100Z9 Bypass, artery, coronary, one site (internal INDEX: Biopsy—see Excision, Kidney, right 0TB0 mammary left) Code is constructed based body part (Kidney, right = 0), approach (percutaneous=3) Device (none = Z) PROCEDURE: Coronary artery typass graft, one and Qualifier (Diagnostic= X) site using internal mammary artery, left to left anterior descending artery, open approach PROCEDURE: Bypass, cardiopulmonary Character Code Explanation Character Code Explanation Section 0 Medical and Surgical Section 5 Extracorporeal Assistance Body 2 Heart and Great Vessels and Performance System Body A Physiological System Root 1 Bypass System Operation Root 1 Performance Body Part 0 Coronary Artery, One Site Operation Approach 0 Open Body Part 2 Cardiac Device Z No Device Approach 2 Continuous Qualifier 9 Internal Mammary, Left Device 1 Output Qualifier Z No Qualifier INDEX: Bypass, artery, coronary, one site 0210 Code is constructed based body part (Coronary INDEX: Bypass, cardiopulmonary 5A1221Z artery, one site = 0), approach (open=0) Device (None=0, the internal mammary artery is not 38. Patient with a history of bladder carcinoma seen considered a device as one end of the internal for a follow-up examination related to his past mammary artery Is detached and sewn into the partial cystectomy treatment; no recurrence specific coronary artery) and Qualifier (Internal found; cystoscopy with biopsy of bladder Mammary, Left=9, the left internal mammary artery Z08 Examination, follow-up, malignant is the vessel that is bypassed from to the left anterior neoplasm descending) Z85.51 History, personal, malignant neoplasm, 02100Z8 Bypass, artery, coronary, one site (internal bladder mammary right) 0TBB8ZX Biopsy, see excision, bladder with qualifier diagnostic PROCEDURE: Coronary artery bypass graft, one site using internal mammary artery, right to left PROCEDURE: Cystoscopy with biopsy of bladder circumflex ,open approach Character Code Explanation Character Code Explanation Section 0 Medical and Surgical Section 0 Medical and Surgical Body T Urinary System Body 2 Heart and Great Vessels System System Root B Excision Root 1 Bypass Operation Operation Body Part B Bladder Body Part 0 Coronary Artery, One Site Approach 8 Via Natural or Artificial Approach 0 Open Opening Endoscopic Device Z No Device Device Z No Device Qualifier 8 Internal Mammary, Right Qualifier X Diagnostic

INDEX: Bypass, artery, coronary, one site 0210 INDEX: Biopsy—see Excision, Bladder Kidney, Code is constructed based body part (Coronary right 0TBB Code is constructed based body part artery, one site = 0), approach (open=0) Device (Bladder= B), approach (cystoscopy =via natural or (None=0, the internal mammary artery is not artificial opening endoscopic=8) Device (none = Z) considered a device as one end of the internal artery and Qualifier (Diagnostic= X) Is detached and sewn into the specific coronary artery) and Qualifier (Internal Mammary, Right = 8, 39. Degenerative joint disease, bilateral knees ; total the right internal mammary artery is the vessel that knee replacement using metal prosthesis is bypassed from to the left circumflex) cemented, left knee 5A1221Z Bypass, cardiopulmonary M17.0 Disease, joint, degenerative—see Osteoarthritis, knees bilateral 0SRD0J9 Replacement, joint, knee, left

PROCEDURE: Total knee replacement using metal prosthesis cemented, left knee Character Code Explanation is coded first. Section 0 Medical and Surgical 0TND8ZZ Release, urethra Body S Lower Joints System PROCEDURE: Cystoscopic release of urethral Root R Replacement stricture in female patient. Operation Character Code Explanation Body Part D Knee Joint, Left Section 0 Medical and Surgical Approach 0 Open Body T Urinary System Device J Synthetic Substitute System Qualifier 9 Cemented Root N Release Operation INDEX: Replacement, joint, knee, left 0SRD. Code Body Part D Urethra is constructed based body part (Knee Joint, left = D), Approach 8 Via Natural or Artificial approach (open = 0) Device (metal prosthesis= Opening Endoscopic synthetic substitute = J) and Qualifier (Cemented = Device Z No Device 9) Qualifier Z No Qualifier

40. Malignant lymphoma, undifferentiated Burkitt INDEX: Release, urethra 0TND Code is constructed type, Intrathoracic; percutaneous bone marrow based body part (Urethra = D), approach biopsy , iliac (cystoscopy = via natural or artificial opening C83.72 Lymphoma, Burkitt endoscopic=8) Device (none = Z) and Qualifier 07DR3ZX Bone marrow biopsy is an Extraction (none=Z) of bone marrow 42. Chronic hidradenitis suppurativa. subcutaneous PROCEDURE: Bone marrow biopsy, iliac, tissue, right axilla; wide excision of hidradenitis percutaneous of right axilla; partial-thickness skin graft. Character Code Explanation Patient's own skin excised and grafted from Section 0 Medical and Surgical patient's back to right axilla Body 7 Lymphatic and Hemic L73.2 Hidradenitis System System 0JBD0ZZ Excision, subcutaneous tissue and Root D Extraction fascia, upper arm, right Operation Body Part R Bone Marrow, Iliac PROCEDURE: Wide excision of hidradenitis, Approach 3 Percutaneous subcutaneous tissue, right axilla Device Z No Device Character Code Explanation Qualifier X Diagnostic Section 0 Medical and Surgical Body J Subcutaneous Tissue and INDEX: Bone marrow biopsy—see Extraction as the System Fascia root operation. Extraction, bone marrow, iliac 07DR. Root B Excision Code is constructed based body part (Bone marrow, Operation Iliac=R), approach (percutaneous=3) Device (none = Body Part D Subcutaneous Tissue and Z) and Qualifier (Diagnostic= X) Fascia, Right upper arm Approach 0 Open 41. Postprocedural stricture of urethra with urinary Device Z No Device retention ; cystoscopic release of urethral Qualifier Z No Qualifier stricture (female patient) N99.12 Stricture, urethra, postprocedural, INDEX: Excision, subcutaneous tissue, axilla is female upper arm, right, 0JBD. Code is constructed based R33.8 Retention, urine, specified (with stricture) body part (Right upper arm (axilla) =D), approach Note: There is a "Code first" note under code (open excision=0) Device (none = Z) and Qualifier R33.8 to code first, if applicable, any causal (No qualifier= Z) condition, such as enlarged prostate N40.1. In 0HRBX74 Graft, see Replacement, skin, right upper this question, the underlying condition is the arm (own skin, partial thickness) postprocedural stricture of the urethra so N99.12 PROCEDURE: Partial thickness skin graft from 45. Gunshot wound of chest with massive back to right axilla intrathoracic injury to right lung with laceration; Character Code Explanation shot by another person with a handgun who was Section 0 Medical and Surgical charged with attempted homicide; injury Body H Skin and Breast occurred on a local residential street; patient System died during an exploratory thoracotomy to Root R Replacement examine right lung Operation S27.331A Gunshot wound, internal organs—see Body Part B Skin, Right Upper Arm Injury, by site. Injury, lung—see Injury, Approach X External intrathoracic, laceration, unilateral (right) Device 7 Autologous Tissue Substitute X93.xxxA Index to External Causes, Shooting, Qualifier 4 Partial thickness homicide (attempt) —see Discharge, firearm, by INDEX: Graft—see Replacement, skin, right upper type (handgun), homicide arm (own skin, partial thickness, axilla is upper arm, Y92.414 Index to External Causes, Place of right, 0HRB. Code is constructed based body part Occurrence, street, local residential (Right upper arm (axilla) =D), approach (external (No statement of activity of patients or status, (can reach skin directly=X) Device (autologous therefore, no codes assigned) tissue, ow skin = 7) and Qualifier (Partial thickness= 4) PROCEDURE: Exploratory thoracotomy to examine right lung PROCEDURE: Excision of skin from back to use Character Code Explanation for partial thickness graft to axilla Section 0 Medical and Surgical 0HB6XZZ Excision, skin, back Body B Respiratory System Character Code Explanation System Section 0 Medical and Surgical Root J Inspection Body H Subcutaneous Tissue and Operation System Fascia Body Part K Lung, Right Root B Excision Approach 0 Open Operation Device Z No Device Body Part 6 Skin, back Qualifier Z No Qualifier Approach X External Device Z No Device INDEX: Exploration—see Inspection. No Index Qualifier Z No Qualifier entry for thoracotomy. Inspection, lung right 0BJK. Code is constructed based body part (Lung, INDEX: Excision, skin, back, 0HB6XZ. Code is right=K), approach (open thoracotomy=0) Device constructed based body part (Skin, back=6), (none = Z) and Qualifier (No qualifier= Z) approach (external=X) Device (none = Z) and Qualifier (No qualifier= Z) 46. Patient admitted for her first round of antineoplastic chemotherapy after a total 43. Heroin poisoning, accidental overdose; acute abdominal hysterectomy and salpingo- lung edema; multiple drug dependence including oophorectomy for right ovarian carcinoma with heroin and barbiturates, initial encounter known metastases to intrapelvic lymph nodes; T40.1X1A Table of Drugs and Chemicals, administration of antineoplastic chemotherapy Heroin, poisoning, accidental by central vein infusion J81.0 Edema, lung, acute Z51.11 Chemotherapy (session) (for) neoplasm F11.20 Dependence, drug, heroin—see C56.1 Neoplasm, ovary, malignant, primary dependence, drug, opioid C77.5 Neoplasm, lymph, intrapelvic, malignant, F13.20 Dependence, drug, barbiturate—see secondary dependence, drug, sedative PROCEDURE: Administration of antineoplastic 44. Positive tuberculosis skin test chemotherapy by central vein infusion R76.11 Positive, skin test, tuberculin (without 3E04305 Chemotherapy, infusion for cancer—see active tuberculosis) Introduction of substance in or on, vein, central, antineoplastic PROCEDURE: Arthroscopy, right knee Character Code Explanation 0SJC4ZZ Arthroscopy, lower joints—see Section 3 Administration Inspection, lower joints Body E Physiological Systems and Character Code Explanation System Anatomical Regions Section 0 Medical and Surgical Root 0 Introduction Body S Lower Joints Operation System Body 4 Central Vein Root J Inspection Region Operation Approach 3 Percutaneous Body Part C Knee Joint, Right Substance 0 Antineoplastic Approach 4 Percutaneous Endoscopic Qualifier 5 Other antineoplastic Device Z No Device Qualifier Z No Qualifier INDEX: Chemotherapy, infusion for cancer—see Introduction of substance in or on, vein, central, INDEX: Arthroscopy, lower joints (knee) —see antineoplastic 3E04. Code is constructed based body Inspection, lower joints. Code is constructed based part (Central vein=4), approach (percutaneous - body part ( Knee joint, right =C), approach puncture into central vein=3) Substance (percutaneous endoscopic (arthroscopic) =4) Device (antineoplastic=0) and Qualifier (Other (type) (none = Z) and Qualifier (No qualifier= Z) antineoplastic= 5) 49. Traumatic arthritis of left wrist secondary to old 47. Congenital hypertrophic pyloric stenosis fracture-dislocation of lower end of radius, left corrected by open to dilate the M12.532 Arthritis, traumatic—see Arthropathy, pylorus of stomach in a 4-week-old infant traumatic, wrist Q40.0 Stenosis, pylorus, congenital S52.502S Sequelae (of) fracture—code to Injury 0D77 Dilation, stomach, pylorus with seventh character S. Fracture, traumatic, radius, lower end, PROCEDURE: Open pyloromyotomy to dilate the pylorus of stomach 50. Pregnancy, preterm labor with preterm delivery Character Code Explanation at 35 weeks, single liveborn infant; postpartum Section 0 Medical and Surgical fever of unknown origin; patient with known Body D Gastrointestinal System continuous marijuana drug dependence; System spontaneous vaginal delivery Root 7 Dilation O60.14x0 Pregnancy, complicated by preterm Operation labor, third trimester, with third term preterm Body Part 7 Stomach, pylorus (35 weeks) delivery. Seventh character of zero Approach 0 Open (0) is for single infant. Device Z No Device Z3A.35 Pregnancy, weeks of gestation, 35 Qualifier Z No Qualifier weeks Z37.0 Outcome of delivery, single, liveborn INDEX: There is no Index entry for O86.4 Postpartum—see Puerperal, fever (of pyloromyotomy. Root operation or objective of unknown origin) procedure is to dilate the pylorus of the stomach. O99.323 Pregnancy, complicated by drug use, Index: Dilation, stomach, pylorus 0D77. Code is third trimester constructed based body part (Stomach, pylorus =7), F12.20 Dependence, drug, marihuana—see approach (open=0) Device (none = Z) and Qualifier Dependence, drug, cannabis (No qualifier= Z)

48. Internal derangement of lateral meniscus, old tear, posterior horn, right knee ; arthroscopy, right knee M23.251 Derangement, joint, knee—see Derangement, knee, due to old tear, lateral, posterior horn, right PROCEDURE: Manually assisted delivery 10E0XZZ Delivery, manually assisted Character Code Explanation Section 1 Obstetrics Body 0 Pregnancy System Root E Delivery Operation Body Part 0 Products of Conception Approach X External Device Z No Device Qualifier Z No Qualifier

INDEX: Delivery, manually assisted 10E0XZZ Code is verified in code table 10E. Code is constructed based body part (Products of conception=0), approach (baby can be reached directly, external=0) Device (none = Z) and Qualifier (No qualifier= Z)